Sunteți pe pagina 1din 306

Mestrado Profissional

em Matemática em Rede Nacional

Iniciação à Matemática

Autores:

Krerley Oliveira Adán J. Corcho

Unidade I:

Capítulos I e II
.

Dedicamos este livro as nossas esposas e lhos, que compreenderam

os sábados sacricados em função de escrevê-lo e a nossos pais, por

tudo o que eles representam.

Tente! E não diga que a vitória está perdida. Se é de batalhas que se

vive a vida. Tente outra vez! (Raul Seixas)


vi
Sumário

Prefácio xi
1 Primeiros Passos 1
1.1 Organizando as Ideias . . . . . . . . . . . . . . . . . . 1

1.2 Verdadeiro ou Falso? . . . . . . . . . . . . . . . . . . . 5

1.3 Teoremas e Demonstrações . . . . . . . . . . . . . . . . 9

1.3.1 Métodos de Demonstração . . . . . . . . . . . . 10

1.4 Algumas Dicas para Resolver Problemas . . . . . . . . 15

1.5 Soluções dos Problemas da Seção 1.4 . . . . . . . . . . 18

1.6 Exercícios . . . . . . . . . . . . . . . . . . . . . . . . . 26

2 Equações e Inequações 31
2.1 Equações do Primeiro Grau . . . . . . . . . . . . . . . 33

2.1.1 Problemas Resolvidos . . . . . . . . . . . . . . . 37

2.2 Sistemas de Equações do Primeiro Grau . . . . . . . . 42

2.2.1 Problemas Resolvidos . . . . . . . . . . . . . . . 46

2.3 Equação do Segundo Grau . . . . . . . . . . . . . . . . 49

2.3.1 Completando Quadrados . . . . . . . . . . . . . 50

2.3.2 Relação entre Coecientes e Raízes . . . . . . . 55

2.3.3 Equações Biquadradas . . . . . . . . . . . . . . 59

2.3.4 O Método de Vièti . . . . . . . . . . . . . . . . 60

vii
viii SUMÁRIO

2.4 Inequações . . . . . . . . . . . . . . . . . . . . . . . . . 62

2.5 Inequação do Primeiro Grau . . . . . . . . . . . . . . . 63

2.6 Inequação do Segundo Grau . . . . . . . . . . . . . . . 69

2.6.1 Máximos e Mínimos das Funções Quadráticas . 75

2.7 Miscelânea . . . . . . . . . . . . . . . . . . . . . . . . . 77

2.7.1 Equações Modulares . . . . . . . . . . . . . . . 77

2.7.2 Um Sistema de Equações Não lineares . . . . . 80

2.8 Exercícios . . . . . . . . . . . . . . . . . . . . . . . . . 81

3 Divisibilidade 89
3.1 Conceitos Fundamentais e Divisão Euclidiana . . . . . 90

3.2 Bases Numéricas . . . . . . . . . . . . . . . . . . . . . 99

3.3 Máximo Divisor Comum e Mínimo Múltiplo Comum . 106

3.3.1 Máximo Divisor Comum . . . . . . . . . . . . . 106

3.3.2 Algoritmo de Euclides . . . . . . . . . . . . . . 111

3.3.3 Mínimo Múltiplo Comum . . . . . . . . . . . . 115

3.3.4 Equações Diofantinas Lineares . . . . . . . . . . 120

3.4 Números Primos e Compostos . . . . . . . . . . . . . . 123

3.5 Procurando Primos . . . . . . . . . . . . . . . . . . . . 127

3.5.1 O Crivo de Eratóstenes . . . . . . . . . . . . . . 127

3.5.2 Primos de Mersenne . . . . . . . . . . . . . . . 129

3.5.3 O Teorema Fundamental da Aritmética . . . . . 133

3.6 Exercícios . . . . . . . . . . . . . . . . . . . . . . . . . 139

4 O Princípio da Casa dos Pombos 143


4.1 Primeiros Exemplos . . . . . . . . . . . . . . . . . . . . 145

4.2 Uma Versão mais Geral . . . . . . . . . . . . . . . . . . 146

4.3 Aplicações na Teoria dos Números . . . . . . . . . . . . 149

4.4 Aplicações Geométricas . . . . . . . . . . . . . . . . . . 151


SUMÁRIO ix

4.5 Miscelânea . . . . . . . . . . . . . . . . . . . . . . . . . 153

4.6 Exercícios . . . . . . . . . . . . . . . . . . . . . . . . . 157

5 Contagem 161
5.1 Princípio Aditivo da Contagem . . . . . . . . . . . . . 162

5.2 Princípio Multiplicativo de Contagem . . . . . . . . . . 170

5.3 Uso Simultâneo dos Princípios Aditivo e Multiplicativo 178

5.4 Permutações Simples . . . . . . . . . . . . . . . . . . . 181

5.5 Arranjos Simples . . . . . . . . . . . . . . . . . . . . . 184

5.6 Combinações Simples . . . . . . . . . . . . . . . . . . . 188

5.7 O Binômio de Newton . . . . . . . . . . . . . . . . . . 193

5.8 Contagem e Probabilidades . . . . . . . . . . . . . . . 195

5.9 Exercícios Propostos . . . . . . . . . . . . . . . . . . . 197

6 Indução Matemática 203


6.1 Formulação Matemática . . . . . . . . . . . . . . . . . 204

6.2 Aplicações . . . . . . . . . . . . . . . . . . . . . . . . . 206

6.2.1 Demonstrando Identidades . . . . . . . . . . . . 206

6.2.2 Demonstrando Desigualdades . . . . . . . . . . 210

6.2.3 Indução e Problemas de Divisibilidade . . . . . 212

6.3 Indução na Geometria . . . . . . . . . . . . . . . . . . 215

6.4 Miscelânea . . . . . . . . . . . . . . . . . . . . . . . . . 220

6.4.1 Cuidados ao Usar o Princípio da Indução . . . . 222

6.5 Indução e Recorrências . . . . . . . . . . . . . . . . . . 222

6.6 Exercícios . . . . . . . . . . . . . . . . . . . . . . . . . 229

7 Desigualdades 233
7.1 Desigualdade Triangular . . . . . . . . . . . . . . . . . 234

7.2 Desigualdade das Médias . . . . . . . . . . . . . . . . . 238


x SUMÁRIO

7.3 Desigualdade de Cauchy-Schwarz . . . . . . . . . . . . 245

7.4 Desigualdade de Jensen . . . . . . . . . . . . . . . . . . 246

7.5 Exercícios . . . . . . . . . . . . . . . . . . . . . . . . . 250

8 Polinômios 255
8.1 Operações com Polinômios . . . . . . . . . . . . . . . . 255

8.2 Algoritmo de Euclides . . . . . . . . . . . . . . . . . . 263

8.3 Sempre Existem Raízes de um Polinômio? . . . . . . . . 268

8.3.1 Números Complexos e Raízes de Polinômios . . 269

8.4 Exercícios . . . . . . . . . . . . . . . . . . . . . . . . . 272

A Apêndice: Funções 279


Referências 285
Prefácio
Imaginação é mais importante que onhe imento.

Albert Einstein

Leo, vo ê tem uma religião? Assim, uma religião, omo judaísmo,

ou ristianismo, ou Matemáti a...?

Alon Peres, 6 anos, lho do Matemáti o Yuval Peres

Neste livro pretendemos oferecer ao leitor uma introdução à Mate-

mática Elementar. Juntando as experiências didáticas vividas pelos

autores individualmente no Brasil e em Cuba, e mais alguns anos

juntos como treinadores de projetos de introdução à Matemática no

estado de Alagoas, esperamos tornar para o leitor a Matemática mais

interessante, mostrando um pouco do imenso brilho e beleza que ela

esconde.

O livro foi escrito em capítulos, cada um deles detalhando um

tema central e trazendo alguns teoremas fundamentais. Com muitos

exemplos e aplicações dos conceitos introduzidos, pretendemos mos-

trar ao leitor a importância do assunto abordado. A organização dos

exemplos tenta seguir uma linha em ordem crescente de diculdade e,

para o melhor aproveitamento do livro, o trabalho com os exercícios

é parte fundamental. Ler o enunciado e resolver o maior número pos-

xi
xii Prefácio

sível de exercícios é imperativo. Como já disse o Prof. Elon Lima,

Matemática não se aprende passivamente.

Os exemplos e aplicações dos conceitos, bem como os teoremas,

devem ser lidos com cuidado e muita atenção. Para os estudantes

que desejem treinar para olimpíadas de Matemática, sugerimos que

formem grupos de estudo para trabalhar os temas individualmente,

sob a orientação de um professor. Acreditamos que o texto pode ser

utilizado em uma disciplina elementar num curso de licenciatura ou

bacharelado em Matemática.

O primeiro capítulo é para introduzir o leitor no espírito do livro

e dar uma amostra do tipo de problemas e material que seguirá nos

demais capítulos. São propostos alguns problemas, muitos deles com

soluções, e discutimos alguns métodos importantes para uso no dia a

dia dos estudantes. Nesta discussão incluímos o estudo de proposições

matemáticas, provas por contraposição, o método de redução ao absur-

do e algumas outras regras básicas e cuidados que devemos ter ao

resolver problemas em Matemática.

Em seguida, estudamos as equações do primeiro e do segundo grau.

Estudamos os métodos de resolução dessas equações, sistemas de equa-

ções, relações entre raízes e coecientes, bem como alguns problemas

interessantes que podem ser solucionados via essas equações. Em se-

guida, estudamos inequações do primeiro e do segundo grau.

O capítulo seguinte trata do conceito de divisibilidade. Tentamos

introduzir o leitor nos principais aspectos básicos, incluindo-se a divisi-

bilidade com resto, máximo divisor comum e mínimo múltiplo comum,

números primos e compostos, e um pouco de equações diofantinas li-

neares.

Um capítulo útil para o estudante que deseja participar de Olim-


Prefácio xiii

píadas de Matemática é o que trata do princípio da casa dos pombos.

Este capítulo é um belo exemplo de como algo aparentemente ingênuo

pode gerar consequências interessantes. Alguns dos exemplos estão

conectados com os capítulos anteriores e aparentemente aplicam o

princípio de modo inusitado, em problemas de geometria, teoria dos

números e em áreas diversas.

No capítulo de contagem, começamos com noções úteis sobre con-

juntos e princípios básicos para contar os elementos de um conjunto.

Nesse capítulo, estamos mais preocupados com as aplicações imediatas

do assunto, sugerindo alguns problemas para o estudante iniciante.

Seguimos discutindo os tipos de agrupamento de elementos e suas

consequências. Obtemos o binômio de Newton e introduzimos a no-

ção de probabilidade de um conjunto, resolvendo alguns problemas

relacionados.

Em seguida, estudante se depara com uma arma poderosa do mate-

mático. O método da indução nita é estudado procurando conectar

esta noção com os capítulos anteriores, reobtendo com o auxílio do

método da indução algumas coisas que já foram deduzidas por outros

métodos. Vários exemplos e problemas são resolvidos, alguns deles de

modo surpreendente e inesperado.

No próximo capítulo, introduzimos algumas desigualdades popula-

res para o uso do estudante. Algumas dessas desigualdades são muito

importantes no estudo mais profundo da Matemática e não apare-

cem em cursos introdutórios, apesar de suas provas e aplicações serem

elementares. Todas as desigualdades aparecem com demonstrações,

em muito dos casos utilizando-se álgebra elementar e o método de

indução nita. São apresentados vários exemplos que mostram a uti-

lidade dessas desigualdades em alguns problemas práticos. Para xar


xiv Prefácio

o conhecimento, propomos vários exercícios complementares. Alguns

deles, cuja solução é mais elaborada, são sugeridos. No último ca-

pítulo, estudamos um pouco as propriedades gerais dos polinômios.

Para complementar a formação do leitor menos experiente, incluímos

um apêndice sobre funções.

Somos gratos a muitas pessoas que colaboraram com a elaboração

deste livro com sugestões e correções em versões iniciais. Entre eles,

citamos: Carlos Gustavo Moreira, Ali Tahzibi, Feliciano Vitório, Edu-

ardo Teixeira, Chico Potiguar e vários de nossos alunos de Iniciação

Cientíca e mestrado, que por várias ocasiões deram sugestões para

a melhoria do texto. Um agradecimento especial vai para Fernando

Echaiz, que nos ajudou ativamente nas notas do Capítulo 5 que origi-

naram este texto. Finalmente, agradecemos aos revisores pela leitura

cuidadosa e ao comitê editorial da SBM, na pessoa da profa. Helena

Lopes, pelo excelente trabalho de editoração.

Maceió, Abril de 2010

Krerley Oliveira

Adán J. Corcho
1
Primeiros Passos
Redu tio ad absurdum, que Eu lides gostava tanto, é uma das mais

nas armas do matemáti o. É muito mais no que um movimento

de xadrez: o jogador de xadrez pode ofere er o sa rifí io de uma

peça, mas o matemáti o ofere e o jogo inteiro.

G. H. Hardy

Neste capítulo, discutiremos algumas ideias gerais e convenções

que servirão como base para os diferentes métodos de resolução de

problemas que trataremos nos capítulos seguintes. Alguns dos exem-

plos que abordamos serão úteis para orientar quanto ao cuidado que

devemos ter quando discutimos problemas em Matemática.

1.1 Organizando as Ideias

Para resolver problemas matemáticos precisamos ter bem claro o que

devemos provar e o que estamos assumindo como verdade. É sobre

isso que falaremos agora. Começaremos observando as seguintes ar-

mações:

1
2 1 Primeiros Passos

(a) A soma de dois números pares é sempre um número par.

(b) Todo brasileiro é carioca.

(c) A terra é um planeta.

(d) Se c é o comprimento da diagonal de um retângulo de lados a e

b, então c 2 = a2 + b 2 .

(e) Se a < 1, então a2 > a.

Todas as armações acima se encaixam no conceito de proposição,

que damos a seguir.

Uma proposição ou sentença é uma frase armativa em forma de

oração, com sujeito, verbo e predicado, que ou é falsa ou é verdadeira,

sem dar lugar a uma terceira alternativa.

Por exemplo, as proposições (a) e (c) são claramente verdadeiras;

mais adiante nos convenceremos da veracidade da proposição (d). Por

outro lado, as proposições (b) e (e) são falsas. Com efeito, para cons-

tatar a veracidade da sentença (b) teríamos que checar o registro de

nascimento de cada brasileiro e vericar se nasceu no Rio de Janeiro,

mas isto é falso pois o conhecido escritor Graciliano Ramos é um

brasileiro nascido em Alagoas. Analogamente, para convencer-nos de

que a proposição (e) é falsa basta tomar a = 1/2 e checar que (1/2)2 =
1/4 não é maior do que 1/2 como a sentença arma. Em ambos os

casos temos vericado que as proposições (b) e (e) são falsas apre-

sentando casos particulares onde as mesmas deixam de valer. Estes

casos particulares são chamados de contraexemplos e são muito úteis

para vericar a falsidade de algumas proposições.

Notemos que as proposições (d) e (e) são do tipo:


1.1 Organizando as Ideias 3

Se P, então Q,

onde P e Q também são sentenças. Por exemplo, na proposição (e)

temos que:

P: c é o comprimento da diagonal de um retângulo de lados a e b,

Q: c2 = a2 + b2 ,

ou seja, estamos assumindo que P é verdade e usando este fato deve-

mos vericar se P é verdade ou não.

Uma proposição condicional ou implicativa é uma nova proposição

formada a partir de duas proposições P e Q, que é escrita na forma:

Se P, então Q ou P implica Q,

onde para o último caso usamos a notação: P =⇒ Q. Chamaremos


a proposição P de hipótese e a proposição Q de tese. A hipótese
também é chamada de proposição antecedente e a tese, de proposição

consequente.

Por exemplo, na proposição condicional (f ) a hipótese é: a<1 e a


2
tese é: a > a.
A partir de uma de uma proposição condicional podem-se gerar

novas proposições que são de especial interesse para os matemáticos.

Vamos chamar o modo em que apresentamos uma proposição de forma

positiva. Por exemplo, quando enunciamos a proposição

Se como laranja, então gosto de frutas,

assumimos esta armação como sua forma positiva. Vamos descrever

agora como podemos obter novas proposições a partir desta.


4 1 Primeiros Passos

Forma recíproca de uma proposição condicional: para cons-

truirmos a forma recíproca, temos que trocar na forma positiva a hi-

pótese pela proposição consequente e vice-versa. Vejamos em nosso

exemplo:

Forma da proposição Hipótese Tese


Positiva como laranja gosto de frutas

Recíproca gosto de frutas como laranja

Assim, a recíproca de proposição de nosso exemplo é então:

Se gosto de frutas, então como laranja

Forma contrapositiva de uma proposição condicional: Para

obtermos a forma contrapositiva a partir da forma positiva de uma

proposição condicional podemos fazer primeiro sua forma recíproca e

em seguida negamos as sentenças antecedente e consequente da recí-

proca ou, também, podemos primeiro negar as sentenças antecedente

e consequente da forma positiva e imediatamente fazer a forma recí-

proca desta última. A forma contrapositiva também é conhecida como

forma contrarrecíproca. Usando novamente nosso exemplo temos que:

Forma da Proposição Hipótese Tese


Positiva como laranja gosto de frutas

Recíproca gosto de frutas como laranja

Contrapositiva não gosto de frutas não como laranja

Portanto, a forma contrapositiva escreve-se assim:


1.2 Verdadeiro ou Falso? 5

Se não gosto de fruta, então não como laranja

Em particular, a forma contrapositiva de uma proposição poderá ser,

eventualmente, uma forma indireta muito ecaz de vericar resultados

em Matemática.

1.2 Verdadeiro ou Falso?

Uma das coisas que distingue a Matemática das demais ciências natu-

rais é o fato de que um tema de Matemática é discutido utilizando-se

a lógica pura e, por conta disso, uma proposição em Matemática, uma

vez comprovada sua veracidade, é aceita como verdade irrefutável e

permanecerá assim através dos séculos. Por exemplo, até hoje usamos

o teorema de Tales do mesmo modo que foi usado antes de Cristo e

este fato continuará valendo eternamente.

Vamos ilustrar melhor essa diferença com um exemplo em Geo-

graa. Hoje, todos nós sabemos que a Terra tem aproximadamente

o formato de uma laranja, um pouco achatada nos polos. Porém,

na época de Pitágoras, um dos grandes temores dos navegadores era

encontrar o m do mundo. No pensamento de alguns destes aventu-

reiros, a Terra tinha o formato de um cubo, e uma vez chegando em

um dos seus extremos, o navio despencaria no vazio. Esse é um dos

muitos exemplos de como a concepção da natureza mudou ao longo

do tempo, transformando uma concepção verdadeira num período da

humanidade em algo completamente falso em outra época. Porém,

para nossa felicidade, isso não acontece na Matemática. Uma propo-

sição matemática ou é verdadeira ou é falsa e permanecerá assim para

sempre.
6 1 Primeiros Passos

Mas como saber se uma proposição é verdadeira ou falsa? A pri-

meira coisa que devemos fazer é tomar muito cuidado. As aparências

enganam ou, como diziam nossos avós, nem tudo que reluz é ouro.

O leitor, avisado disso, pense agora na seguinte pergunta:

Pergunta 1: Qual é a chance de que pelo menos duas pessoas num

ônibus com 44 passageiros façam aniversário no mesmo dia do ano?

Como já avisamos, o leitor deve ter cuidado ao responder à per-

gunta acima, pois podemos nos enganar muito facilmente. Por exem-

plo, podemos formular o seguinte argumento errado: o ano tem 365

dias e, como estou escolhendo um grupo de 44 (número muito pequeno

com respeito a 365) pessoas ao acaso, é claro que podemos responder

à pergunta com a seguinte armação:

Resposta intuitiva: A chance de que num grupo de 44 pessoas pelo

menos duas delas façam aniversário no mesmo dia do ano é pequena.

À primeira vista a resposta dada pode até parecer verdadeira, mas

com uma análise mais cuidadosa veremos que é completamente falsa.

Na verdade, a chance de que pelo menos duas pessoas do ônibus façam

aniversário no mesmo dia do ano é de cerca de 93%!

Quem não acreditar nisto pode fazer duas coisas: primeiro, ir a

sua sala de aula ou no seu ônibus escolar, que deve ter pelo menos

44 pessoas, e fazer o experimento ao vivo. Muito provavelmente você

deve conseguir duas pessoas que fazem aniversário no mesmo dia do

ano. Se você verica que existem duas pessoas que fazem aniversário

no mesmo dia do ano, não é por acaso, pois a chance de isso acontecer

é muito alta. Mas, cuidado! Isso não é uma prova matemática para

este fato. Para provar que este fato é verdadeiro você deve vericar

que se escolhermos ao acaso um grupo de 44 pessoas então com aproxi-


1.2 Verdadeiro ou Falso? 7

madamente 93% de chance, pelo menos duas delas fazem aniversário

no mesmo dia do ano!

Porém, se você faz o experimento e não encontra duas pessoas que

fazem aniversário no mesmo dia do ano (você seria muito azarado!),

não se desespere. Lembre-se de que se trata de algo que acontece com

chance de 93% e que pode não acontecer quando fazemos um teste.

Em qualquer um dos casos, para ter a certeza de que a proposição

é verdadeira o leitor deve demonstrá-la. Faremos isso no nal do

Capítulo

Vamos analisar agora outro fato aparentemente óbvio.

Pergunta 2: Num campeonato de futebol onde cada time joga a

mesma quantidade de jogos, cada vitória vale três pontos, o empate

vale um ponto e a derrota nenhum ponto. Em caso de empate, o

critério de desempate entre as equipes era o seguinte:

• A melhor equipe é aquela que tem mais vitórias.

Os organizadores decidiram passar a adotar o critério a seguir:

• A melhor equipe é aquela que tem mais derrotas.

Você acha que este último critério adotado é justo?

Com respeito a esta pergunta, o leitor deve ter respondido do se-

guinte modo:

Resposta: Um time que perdeu mais é pior que um que perdeu me-

nos; portanto, a mudança de critério é totalmente injusta. Acertamos

a sua resposta?

Na verdade, não houve mudança nenhuma de critério, ou seja,

ambos os critérios nos conduzem ao mesmo ganhador.


8 1 Primeiros Passos

Para ver isso rapidamente, lembre-se de que se a equipe A perdeu

mais que a equipe B e ainda assim empataram, então ela deve ter

ganho mais, para que no m do campeonato a equipe A ainda assim

conseguisse empatar com a equipe B. Vamos mostrar isso precisa-

mente. Sejam d1 , e1 , v1 o número de derrotas, empates e vitórias,

respectivamente, da equipe A. Do mesmo modo, sejam d2 , e2 , v2 o

número de derrotas, empates e vitórias, respectivamente, da equipe

B. Suponhamos que a equipe A obteve mais vitórias do que a equipe

B, ou seja, que v1 > v2 . Como cada equipe jogou o mesmo número de

jogos, temos que

d1 + e1 + v1 = d2 + e2 + v2 . (1.1)

Por outro lado, note que o número de pontos obtidos pela equipe A é

e1 + 3v1 . Do mesmo modo, o número de pontos obtidos pela equipe

B é igual a e2 + 3v2 . Como as duas empataram, temos que:

e1 + 3v1 = e2 + 3v2 .

Ou ainda,

e2 − e1
3(v1 − v2 ) = e2 − e1 ou v2 − v1 = − .
3
Como v1 −v2 > 0, temos que e2 −e1 > 0. Reescrevendo a equação (1.1),

temos que:

e2 − e1 2
d1 − d2 = e2 − e1 + (v2 − v1 ) = e2 − e1 − = (e2 − e1 ).
3 3
Logo, temos que d1 − d2 > 0, pois e2 − e1 > 0. Isso signica que
A teve mais derrotas que B ; logo, qualquer um dos dois critérios de

desempate usado nos leva à equipe vencedora.


1.3 Teoremas e Demonstrações 9

Assim, como estes dois exemplos mostram, ao depararmos com um

problema em Matemática, devemos ter cuidado ao tirar conclusões

apressadas para evitar que cometamos algum engano. Pode acontecer

que uma situação que é claramente falsa para um observador menos

atento, se mostre verdadeira quando fazemos uma análise mais crite-

riosa.

1.3 Teoremas e Demonstrações

Agora denimos o que entendemos por demonstração matemática de

uma proposição.

Uma demonstração em Matemática é o processo de raciocínio ló-

gico e dedutivo para checar a veracidade de uma proposição condici-

onal. Nesse processo são usados argumentos válidos, ou seja, aqueles

que concluam armações verdadeiras a partir de fatos que também

são verdadeiros.

Como exemplo de demonstração citamos a argumentação usada

para mostrar na segunda pergunta da seção anterior que os critérios

de desempate eram similares.

Sempre que, via uma demonstração, comprovemos a veracidade de

uma proposição passamos então a chamar esta de teorema. Assim, um

teorema é qualquer armação que possa ser vericada mediante uma

demonstração.

Alguns teoremas se apresentam na forma de uma proposição con-

dicional, isto é, uma sentença do tipo SeP , então Q ou implicativa


da forma  P =⇒ Q. Nesse caso, a sentença P é chamada de hipótese

e a sentença Q é denominada de tese. Ou seja, a validade da hipótese

nos implica a veracidade da tese.


10 1 Primeiros Passos

Um exemplo de teorema é o famoso teorema de Pitágoras, cujo

enunciado diz o seguinte:

Teorema 1.1 (Teorema de Pitágoras) . Num triângulo retângulo a

soma dos quadrados dos catetos é igual ao quadrado da hipotenusa.

Notemos que o teorema de Pitágoras não está enunciado na forma

condicional, mas pode ser reescrito nessa forma como:

Teorema 1.2 (Teorema de Pitágoras). Se T é um triângulo retângulo

de catetos a e b e hipotenusa c, então c = a + b2 .


2 2

Observação 1.3. Em geral, é mais comum usar a palavra teorema

apenas para certas proposições que são de grande importância mate-

mática, chamando-se simplesmente de proposição ao resto das propo-

sições verdadeiras que admitem uma demonstração. Para uma discus-

são mais detalhada, recomendamos [8].

1.3.1 Métodos de Demonstração


Quando realizamos uma demonstração não existe um caminho único.

Dependendo do problema em questão podemos usar métodos dife-

rentes. A seguir ilustramos os seguintes três métodos:

• Demonstração direta.

• Demonstração por contraposição.

• Demonstração por redução ao absurdo.


1.3 Teoremas e Demonstrações 11

Demonstração Direta
A demonstração direta é aquela em que assumimos a hipótese como

verdadeira e através de uma série de argumentos verdadeiros e dedu-

ções lógicas concluímos a veracidade da tese.

a b
β a
b γ
Q α
c b
a

Figura 1.1: Figura auxiliar para a demonstração do teorema de Pitágoras

Um exemplo de demonstração direta é a que daremos a seguir,

para o teorema de Pitágoras enunciado anteriormente no Teorema

1.1. Com efeito, usando a gura acima temos que a área do quadrado

de lado a+b é a soma das quatro áreas dos triângulos retângulos

congruentes pelo critério lado-ângulo-lado (de catetos a e b) mais a

área do quadrilátero Q, o qual é um quadrado visto que cada um dos

seus lados coincide com a hipotenusa c dos triângulos retângulos de

catetos a
b e, além disso, cada um dos seus ângulos internos
e mede
o
γ = 180 − (α + β) = 180◦ − 90◦ = 90◦ (veja a Figura 1.1).
Portanto,
ab
(a + b)2 = 4 · + c2 ,
2
de onde

a2 + 2ab + b2 = 2ab + c2 ,
e consequentemente

a2 + b 2 = c 2 ,
12 1 Primeiros Passos

como queríamos.

Demonstração por Contraposição


Este método é baseado no fato de que a veracidade de forma positiva

de uma proposição é equivalente à veracidade de sua forma contraposi-

tiva, podendo ser esta última, eventualmente, mais fácil de se provar.

Por exemplo, a armação

Se sou alagoano, então sou brasileiro

é equivalente à armação

Se não sou brasileiro, então não sou alagoano

Por exemplo, provemos a seguinte proposição:

Proposição 1.4. Se N2 é par, então N é par.

• Hipótese: N2 é par.

• Tese: N é par.

Desaamos o leitor a tentar mostrar esta proposição partindo da hipó-

tese e tentando concluir a tese. Note que podemos vericar que nossa

proposição é verdadeira para vários valores de N2 como na tabela a

seguir, mas isso não é uma prova matemática da nossa proposição.

N2 4 16 36 64 100 144

N 2 4 6 8 10 12
1.3 Teoremas e Demonstrações 13

Mesmo vericando para um bilhão de valores de N 2, sempre nos

restariam números para serem vericados. Como nossas tentativas

de provar a forma positiva dessa proposição estão sendo frustradas,

apelaremos para mostrar a forma contrapositiva da mesma, isto é:

Proposição 1.5. Se N não é par, então N2 não é par.

Neste caso, temos:

• Hipótese: N não é par.

• Tese: N2 não é par.

Demonstração. Como estamos assumindo que N N


não é par, logo

tem que ser ímpar, ou seja, existe p, número inteiro, tal que N = 2p+1.

Logo,

N 2 = (2p + 1)(2p + 1)
= 4p2 + 2p + 2p + 1
= 4p2 + 4p + 1
= 2(2p2 + 2p) + 1
= 2q + 1,
onde q = 2p2 + 2p. Logo, N 2 = 2q + 1 é ímpar e concluímos assim

nossa prova.

Demonstração por Redução ao Absurdo


Este método é uma das ferramentas mais poderosas da Matemática.

O nome provém do latim reductio ad absurdum e também é conhecido

como método do terceiro excluído devido ao mesmo estar baseado na


14 1 Primeiros Passos

lei do terceiro excluído que diz o seguinte: uma armação que não

pode ser falsa, deverá ser consequentemente verdadeira.

De um modo geral, o roteiro que segue uma demonstração por

redução ao absurdo é o seguinte:

• Assumimos a validade da hipótese.

• Supomos que nossa tese é falsa.

• Usando as duas informações anteriores concluímos, através de

argumentos verdadeiros, uma armação falsa; como tal fato não

poderá ocorrer, então nossa tese deverá ser verdadeira.

Vamos mostrar como o método funciona na prática provando a

seguinte proposição:

Proposição 1.6. Seja x um número positivo, então x + 1/x ≥ 2.

Destaquemos primeiramente a nossa hipótese e a nossa tese.

• Hipótese: x é um número positivo.

• Tese: x + 1/x ≥ 2.

Demonstração. Seja x um número positivo e suponhamos que a tese


1
é falsa, isto é, x+ x
< 2. Usando que x > 0 e multiplicando por este

a desigualdade anterior, obtemos que

x2 + 1 < 2x.

Daí segue-se quex2 − 2x + 1 < 0 é equivalente a (x − 1)2 < 0, já que


x2 − 2x + 1 = (x − 1)2 , o que é impossível. Portanto, x + 1/x ≥ 2,
como desejávamos.
1.4 Algumas Dicas para Resolver Problemas 15

1.4 Algumas Dicas para Resolver Proble-

mas

Nesta seção, damos algumas regras gerais que consideramos impor-

tante ter em mente na hora de resolver um problema de Matemática.

Aplicaremos estas regras a alguns problemas interessantes para ilus-

trar a sua importância. Elas são:

R1) Ler bem o enunciado do problema e utilizar todas as informações

disponíveis.

R2) Fazer casos particulares ou casos mais simples de problemas si-

milares, para adquirir familiaridade com o problema.

R3) Mudar a representação do problema, transformando-o em um

problema equivalente.

R4) Usar a imaginação pesquisando caminhos alternativos. Extra-

polar os limites!

A seguir propomos vários problemas onde as regras anteriores são

muito úteis. O leitor deve tentar resolvê-los; mas se não conseguir

achar solução depois de muito tentar poderá então passar para a pró-

xima seção onde os solucionamos.

Problema 1.7. Ao encontrar uma velha amiga (A), durante uma

viagem de trem, um matemático (M) tem a seguinte conversa:

(M)  Como vão os três lhos da senhora?

(A)  Vão bem, obrigada!


16 1 Primeiros Passos

(M)  Qual a idade deles mesmo?

(A)  Vou lhe dar uma dica. O produto das idades deles é 36.

(M)  Só com essa dica é impossível!

(A)  A soma das idades deles é igual ao número de janelas deste

vagão.

(M)  Ainda não sei!

(A)  O mais velho toca piano!

(M) Agora eu sei!

Você é capaz de descobrir as idades dos três lhos da senhora?

Problema 1.8. Numa cesta encontram-se 9 moedas idênticas, sendo

que 8 delas têm o mesmo peso e uma moeda é mais leve que as demais.

Usando duas vezes uma balança de dois pratos, encontrar a moeda

mais leve.

Problema 1.9. Numa pequena ilha existem 5 pessoas de olhos azuis

e 5 pessoas de olhos verdes. Existe um grande tabu nesta ilha que é o

seguinte: se uma pessoa descobre que possui olhos azuis ela se suicida

à meia-noite do dia em que descobriu, pulando do alto da prefeitura.

Por conta disso, ninguém conversa sobre o assunto, olha para espelhos

ou vê seu reexo na água. Todos se cruzam diariamente e conhecem

os olhos de seus amigos. Numa manhã, um estrangeiro chegou à ilha

e reuniu as 10 pessoas para o seguinte pronunciamento:

Nesta ilha, existe uma pessoa de olhos azuis.

Pergunta-se:
1.4 Algumas Dicas para Resolver Problemas 17

(a) O que aconteceu com os habitantes da ilha?

(b) Que informação nova o estrangeiro trouxe?

Problema 1.10. Um viajante deseja se hospedar durante 31 dias num


hotel. Entretanto, percebe que está sem dinheiro e que a única coisa

que possui é uma corrente com 31 elos de ouro. Para pagar sua conta,

ele acertou com o gerente pagar um elo por dia, sem atrasar ou a-

diantar o pagamento, durante os 31 dias. O gerente pode dar troco em

elos. Depois ele deseja recuperar a corrente e por isso ele quer pagar

a conta cortando a corrente no menor número de pedaços. Quantos

cortes você conseguiria dar e pagar a conta?

Problema 1.11. Sabendo que em cada jogada o movimento do cavalo

consiste em se deslocar duas casas na horizontal e uma na vertical

ou duas na vertical e uma na horizontal, decidir se é possível sair

da conguração apresentada no tabuleiro (a) e chegar à conguração

apresentada no tabuleiro (b) da Figura 1.2 sem que em algum momento

existam dois cavalos na mesma casa.

(a) (b)

Figura 1.2: Cavalos de xadrez


18 1 Primeiros Passos

Problema 1.12. Mostre que podemos cobrir os 9 pontos no reticulado


da Figura 1.3 traçando 4 segmentos de reta sem tirar o lápis do papel.

• • •
• • •
• • •

Figura 1.3: Reticulado de 9 pontos

Sugerimos seguir as dicas abaixo para obter sucesso na solução dos

problemas:

• Para os problemas 1.7 e 1.8 use a primeira regra.

• Para os problemas 1.9 e 1.10 use a segunda regra. Por exemplo,

no problema 1.9 fazer primeiro o caso: uma pessoa com olhos

azuis e uma com olhos verdes e depois fazer o caso: duas pessoas

de olhos azuis e duas de olhos verdes; generalize.

• Para os problema 1.11 use a terceira regra.

• Para o problema 1.12 use a quarta regra.

1.5 Soluções dos Problemas da Seção 1.4

A seguir apresentamos soluções para os problemas enunciados na seção

anterior.

Solução do Problema 1.7. É muito importante neste problema tirar

o máximo de informação das dicas da senhora. Vamos à primeira dica:

o produto das idades é 36.


1.5 Soluções dos Problemas da Seção 1.4 19

Suponhamos que as idades dos lhos sejam 0 6 x 6 y 6 z 6 36.


Como xyz = 36, temos as seguintes possibilidades para os números x,

y e z:
x y z xyz
1 1 36 36
1 2 18 36
1 3 12 36
1 4 9 36
1 6 6 36
2 2 9 36
2 3 6 36
3 3 4 36
A segunda dica dada pela senhora é a soma das idades. Assim,

vamos agora calcular todas as possíveis somas de acordo com as fato-

rações de 36 dadas na tabela anterior:

x y z x+y+z
1 1 36 38
1 2 18 21
1 3 12 16
1 4 9 14
13
1 6 6
13
2 2 9
2 3 6 11
3 3 4 10
Sabemos que após a segunda dica, o matemático ainda não conse-

guiu deduzir as idades das crianças.


20 1 Primeiros Passos

Por que ele não conseguiu? Imagine que o número da casa fosse

14. Ora, de acordo com nossa tabela, só existe um terno de números

cujo produto é 36 e a soma é 14, que é o terno (1,4,9). Assim, se o

número da casa fosse 14 o matemático teria dado a resposta após a

segunda dica. Como ele cou em dúvida, olhando a tabela 2, chegamos

à conclusão de que o número da casa só pode ser igual a 13.

Lembremos a última dica: o mais velho toca piano. No início essa

dica parecia inútil, mas agora ela é fundamental para resolvermos o

problema. De fato, como o mais velho toca piano, isso signica que

existe um mais velho, o que descarta o caso (1,6,6). Assim, as idades

são 2, 2, e 9.

Solução do Problema 1.8. Este é o tipo de problema que a primeira

vista pode parecer difícil, mas que quando usamos todas as informa-

ções do seu enunciado se torna fácil. A ideia é dividir as moedas em

três grupos de três moedas cada, que chamaremos grupos A, B e C.


Colocaremos na balança os grupos A e B e deixaremos o grupo C fora.

Podem acontecer duas coisas:

(a) Os pratos cam equilibrados.

(b) Os pratos cam desequilibrados.

A e B têm o mesmo peso. Logo,


No caso (a), temos que os grupos

a moeda mais leve deve estar no grupo C . No caso (b), um dos grupos

cou mais leve, o que signica que a moeda mais leve está neste grupo.

Assim, utilizando a balança apenas uma vez conseguiremos descobrir

qual é o grupo em que a moeda mais leve está. Digamos que este grupo

seja o grupo A. Para achar a moeda mais leve, procedemos de modo

semelhante ao que zemos anteriormente: separamos as três moedas


1.5 Soluções dos Problemas da Seção 1.4 21

do grupo A colocando uma em cada prato e deixando a terceira de

fora. Podem acontecer duas coisas:

(a) Os pratos cam desequilibrados e assim a moeda mais leve está

no prato mais leve.

(b) Os pratos cam equilibrados, logo a moeda mais leve foi a que

cou fora.

No nal, usamos a balança exatamente duas vezes.

Solução do Problema 1.9. Como em muitos problemas de Mate-

mática, abordar casos mais simples do problema pode ajudar bastante

na solução. Assim, vamos imaginar o seguinte caso mais simples:

na ilha existe somente uma pessoa de olhos azuis e a outra de olhos

verdes. Pensando neste caso, a pessoa que tinha olhos azuis só via as

que tinham olhos verdes. Quando o estrangeiro armou que existia

uma pessoa de olhos azuis, ela descobriu que tinha olhos azuis, pois as

outras pessoas tinham olhos verdes. Assim, à meia-noite ela subiu na

prefeitura e pulou. Com isso, a pessoa que tinha olhos verdes descobriu

que tinha olhos verdes, pois se ela tivesse olhos azuis sua companheira

não se suicidaria no dia anterior.

Vamos agora dar um passo crucial na solução do nosso problema

original, considerando o caso onde existem duas pessoas de olhos azuis

e duas pessoas de olhos verdes na ilha. Vamos chamar as pessoas de

olhos azuis de A e B e as pessoas de olhos verdes de C e D. No dia

em que o estrangeiro fez o seu pronunciamento, nada aconteceu, pois

as pessoas C e D viam as pessoas A e B com olhos azuis e a pessoa

A via a pessoa B com olhos azuis e vice-versa. Já no segundo dia, a

pessoa A teve o seguinte pensamento:


22 1 Primeiros Passos

Se eu tivesse olhos verdes, a pessoa B teria descoberto que

tinha olhos azuis ontem, pois ela veria três pessoas de olhos

verdes. Como ela não se suicidou ontem, eu tenho olhos

azuis.

Pensando da mesma forma, a pessoa B descobriu que também tinha

olhos azuis. Por isso, à meia-noite do segundo dia, as pessoas A e B


se suicidaram.

O que aconteceu depois? As pessoas C e D ainda tinham a dúvida

da cor de seus olhos. Para chegar à conclusão de que seus olhos são

verdes, no terceiro dia, a pessoa C pensou assim:

Bem, se eu tivesse olhos azuis, as pessoas A e B veriam

cada uma duas pessoas com olho azul. Logo, elas não te-

riam se suicidado no segundo dia, pois não conseguiriam

deduzir a cor de seus olhos. Logo, tenho olhos verdes.

Ufa!

Do mesmo modo, a pessoa D conseguiu descobrir a cor de seus olhos.

Analisando de modo semelhante, conseguiremos deduzir que no

problema original as cinco pessoas de olhos azuis descobrirão que pos-

suem olhos azuis e juntas se suicidarão no quinto dia após o pronun-

ciamento do estrangeiro.

Agora vamos descobrir a resposta da segunda pergunta do enun-

ciado: que informação nova o estrangeiro trouxe? Aparentemente

nada de novo foi acrescentado pela frase do estrangeiro, pois cada

pessoa estava vendo alguma pessoa com olhos azuis. Mas isso não é

verdade.

Para ver isso e descobrir qual é a nova informação que o estrangeiro

trouxe, vamos voltar ao caso de somente duas pessoas na ilha, uma


1.5 Soluções dos Problemas da Seção 1.4 23

de olhos azuis e outra de olhos verdes. Neste caso, a pessoa de olhos

azuis somente vê uma pessoa de olhos verdes. Com a informação de

que existe uma pessoa de olhos azuis ela pode descobrir a cor de seus

olhos. Note que a pessoa de olhos verdes já sabia que existia pelo

menos uma pessoa de olhos azuis. Mas ela não sabia que a pessoa

de olhos azuis tinha conhecimento de que na ilha existia alguém com

olhos azuis. Essa é a nova informação que o estrangeiro trouxe.

Solução do Problema 1.10. Uma primeira solução é cortar a cor-

rente 30 vezes, separando todos os elos. Porém, essa não é a melhor so-

lução, como veremos a seguir. Vamos iniciar nossa análise observando

que para pagar o primeiro dia precisamos dar um corte na corrente.

Assim, o gerente receberá um elo. O pulo do gato do problema vem



agora: para pagar o 2 dia, vamos cortar a corrente de modo a separar

dois elos de uma vez. Assim, daremos dois elos ao gerente e ele de-

volverá um elo de troco. Com este elo pagaremos o terceiro dia. Note

que pagamos três dias fazendo dois cortes na corrente, como mostra a

tabela:

Gerente Viajante

Elos 1, 2 28

Note que o número 2 denota o pedaço que contém 2 elos. Para



pagar o 4 dia, cortaremos a corrente de modo a obter um pedaço

com quatro elos. Entregamos ao gerente este pedaço e recebemos

de troco um elo solto e um pedaço com dois elos. Com o elo solto,
◦ ◦
pagamos o 5 dia. Assim, no 5 dia teremos os seguintes grupos de

elos:

Gerente Viajante

Elos 1, 4 2, 24
24 1 Primeiros Passos


Assim, pagamos o 6 dia com o pedaço que contém dois elos e

receberemos o elo solto de troco. Finalmente pagaremos o 7 dia com

o elo solto. Note que foi possível pagar 7 dias com apenas três cortes na

corrente. A continuação do procedimento está quase revelada. Para



pagar o 8 dia, cortaremos um pedaço com oito elos. Daremos este

pedaço e receberemos de troco 7 elos, sendo um elo solto, um pedaço

com 4 e um pedaço com dois elos. Repetindo o procedimento anterior,



pagaremos os 7 dias seguintes, pagando até o 15 dia sem precisar de

cortes adicionais. Ou seja, para pagar os 15 primeiros dias, precisamos

de 4 cortes na corrente. Neste momento, a corrente está distribuída

do seguinte modo:

Gerente Viajante

Elos 1, 2, 4, 8 16


Para pagar o 16 dia, entregaremos ao gerente o pedaço com os 16

elos restantes, recebendo 15 elos divididos em pedaços de 1, 2, 4 e 8



elos. Se repetirmos o processo, pagaremos o hotel até o 31 dia sem

precisar de novos cortes. Assim, o mínimo número de cortes é 4.

Solução do Problema 1.11. Para resolver este problema vamos usar

a estratégia de mudar a representação. O que signica isso? Vamos

reescrever o problema com outros ingredientes, porém sem alterar em

nada sua essência. Primeiramente, enumere as casas do tabuleiro com

os números 1, 2, . . . , 9, como na Figura 1.4.

Vamos agora associar ao tabuleiro, um conjunto de nove pontos

também enumerados com os números 1, 2, . . . , 9. Se for possível sair

de uma casa i e chegar à casa j com apenas uma jogada do cavalo,

colocaremos um segmento ligando os pontos i e j. Por exemplo, é


1.5 Soluções dos Problemas da Seção 1.4 25

1 2 3

4 5 6

7 8 9

Figura 1.4: Tabuleiro de 9 casas

possível, saindo da casa 1 chegar à casa 6 e a casa 8. Desse modo,

o ponto com número 1 está ligado com o ponto com número 8. Se

analisarmos todas as possíveis ligações entre os pontos obteremos um

esquema com o mostrado na Figura 1.5

9 9
• 2• •
2• •4 •4
5 5
7 • • • 3 7 • • • 3

• • • •
6 • 8 6 • 8
1 1

Figura 1.5: Conexões das casas Figura 1.6: Tabuleiro (a)

Assim, se colocarmos os cavalos como no tabuleiro (a), teremos

a situação descrita na Figura 1.6. Deste modo, ca evidente que não

podemos trocar a posição dos cavalos branco e preto sem que em algum

momento eles ocupem a mesma casa.


26 1 Primeiros Passos

1.6 Exercícios

1. Uma sacola contém meias cujas cores são branca, preta, amarela

e azul. Sem olhar para a sacola, qual é a quantidade mínima de

meias que precisamos retirar da mesma para garantir pelo menos

um par de meias da mesma cor?

2. O pai do padre é lho único de meu pai. O que eu sou do padre?

3. Numa mesa há 5 cartas:

Q T 3 4 6

Cada carta tem de um lado um número natural e do outro lado

uma letra. João arma: Qualquer carta que tenha uma vogal

tem um número par do outro lado. Pedro provou que João

mente virando somente uma das cartas. Qual das 5 cartas foi a

que Pedro virou?

4. A polícia prende 4 homens, um dos quais é culpado de um furto.

Eles fazem as seguintes declarações:

• Arnaldo: Bernaldo é o culpável.

• Bernaldo: Cernaldo é o culpável.

• Dernaldo: eu não sou culpável.

• Cernaldo: Bernaldo mente ao dizer que eu sou culpável.

Se se sabe que só uma destas declarações é a verdadeira, quem

é culpável pelo furto?


1.6 Exercícios 27

5. Numa cidade existe uma pessoa X que sempre mente terças,

quintas e sábados e é completamente sincera o resto dos dias

da semana. Felipe chega um certo dia na cidade e mantém o

seguinte diálogo com a pessoa X:


 Felipe: Que dia é hoje?

 X: Sábado.
 Felipe: Que dia será amanhã?

 X: Quarta-feira.
Em qual dia da semana foi mantido este diálogo?

6. Divida o relógio de parede abaixo em 6 partes iguais de forma tal

que a soma das horas que cam em cada parte seja a mesma.n

12
11 1
10 2
9 • 3
8 4
7 5
6

7. João adora Gabriela, que é uma aluna excelente em Matemática.

João armou um plano para dar um beijo nela, e descobriu que

poderá fazer isso apenas dizendo uma frase. Que frase é essa?

8. No plano se colocam 187 rodas dentadas do mesmo diâmetro,

enumeradas de 1 até 187. A roda 1 é acoplada com a roda 2, a 2

com a 3, ..., a 186 com a 187 e esta última com a roda 1. Pode

tal sistema girar?


28 1 Primeiros Passos

9. Um canal, em forma quadrada, de 4 metros de largura rodeia um

castelo. A ponte do castelo está fechada e um intruso quer entrar

no castelo usando duas pranchas de 3,5 metros de comprimento.

Será que o intruso consegue?

10. Os números 1, 2, 3, . . . , 99 são escritos no quadro-negro e é permi-


tido realizar a seguinte operação: apagar dois deles e substituí-

los pela diferença do maior com o menor. Fazemos esta operação

sucessivamente até restar apenas um último número no quadro.

Pode o último número que restou ser o zero?

11. Alguém elege dois números, não necessariamente distintos, no

conjunto de números naturais 2, . . . , 20. O valor da soma destes

A) e o valor do produto dos


números é dado somente a Adriano (

números é dado unicamente a Karla (K).

 Pelo telefone A diz a K: Não é possível que descubras minha


soma.

 Uma hora mais tarde, K lhe diz a A: Ah! sabendo disso, já

sei quanto vale tua soma!

 Mais tarde A chama outra vez a K e lhe informa: Poxa,

agora eu também conheço teu produto!

Quais números foram eleitos?

12. É possível cobrir um tabuleiro de xadrez com 31 dominós onde

removemos as casas dos vértices superior esquerdo e inferior di-

reito?

13. Num saco encontram-se 64 moedas leves e 64 moedas pesadas.


1.6 Exercícios 29

É possível separar duas moedas de pesos diferentes com 7 pesa-

gens?

14. Quantas vezes precisamos dobrar um papel de 1mm de espessura

para que a altura da pilha chegue da Terra à Lua?

15. Descubra o erro da prova da armação abaixo:

Armação: Três é igual a dois.

Seja x um número diferente de zero. Temos que:

3x − 3x = 2x − 2x.

Colocando x−x em evidência, temos que:

3(x − x) = 2(x − x).

Cancelando x−x em ambos os lados, obtemos que 3 = 2.


30 1 Primeiros Passos
2
Equações e Inequações

Álgebra é generosa; ela geralmente nos dá mais do que lhe pedimos.

D'Alembert

Na antiguidade, todo conhecimento matemático era passado de

geração para geração através de receitas. A falta de símbolos e notação

adequada complicava substancialmente a vida de quem precisava usar

a Matemática e de quem apreciava sua beleza. Por exemplo, o uso de

letras (x, y, z etc.) para representar números desconhecidos não tinha

sido inventado ainda. Isso só veio ocorrer por volta dos meados do

século XVI, ou seja, a menos de 500 anos atrás.

Apesar disso, o conhecimento matemático das antigas civilizações

era surpreendente. Os egípcios, babilônios, gregos e vários outros po-

vos tinham o domínio de métodos e técnicas que são empregados hoje,

como soluções de equações do primeiro e segundo graus, inteiros que

são soma de quadrados e vários outros conhecimentos. Especialmente

os gregos, cuja cultura matemática resistiu aos tempos com a preser-

vação de Os Elementos de Euclides, tinham desenvolvido e catalisado

31
32 2 Equações e Inequações

muitos dos avanços da época.

Entretanto, todos os resultados tinham uma linguagem através dos

elementos de geometria, mesmo aqueles que só envolviam proprieda-

des sobre os números. Essa diculdade deve-se em parte aos sistemas

de numeração que eram utilizados pelos gregos e, posteriormente, pe-

los romanos, que eram muito pouco práticos para realizar operações

matemáticas.

Por volta de 1.100, viveu na Índia Bhaskara, um dos mais impor-

tantes matemáticos de sua época. Apesar de suas contribuições terem

sido muito profundas na Matemática, incluindo-se aí resultados sobre

equações diofantinas, tudo indica que Bhaskara não foi o primeiro a

descobrir a fórmula, que no Brasil chamamos de fórmula de Bhaskara,

assim como Pitágoras não deve ter sido o primeiro a descobrir o te-

orema que leva o seu nome, já que 3.000 a.c. os babilônios tinham

conhecimento de ternas pitagóricas de números inteiros bem grandes.

Apesar de ter conhecimento de como solucionar uma equação do

segundo grau, a fórmula que Bhaskara usava não era exatamente igual

a que usamos hoje em dia, sendo mais uma receita de como encontrar

as raízes de uma equação. Para encontrar essas raízes, os indianos

usavam a seguinte regra:

Multiplique ambos os membros da equação pelo número que vale

quatro vezes o coeciente do quadrado e some a eles um número igual

ao quadrado do coeciente original da incógnita. A solução desejada

é a raiz quadrada disso.

O uso de letras para representar as quantidades desconhecidas só

veio a se tornar mais popular com os árabes, que também desenvol-

veram um outro sistema de numeração, conhecido como indo-arábico.

Destaca-se também a participação do matemático francês François


2.1 Equações do Primeiro Grau 33

Vièti, que aprimorou esse uso dos símbolos algébricos em sua obra In

artem analyticam isagoge e desenvolveu um outro método para resol-

ver a equação do segundo grau.

Na seção seguinte estudaremos com detalhe a equação do primeiro

grau, e como podemos utilizá-la para resolver alguns problemas em

Matemática.

2.1 Equações do Primeiro Grau

Iniciamos nossa discussão resolvendo o seguinte problema:

Exemplo 2.1. Qual é o número cujo dobro somado com sua quinta

parte é igual a 121?

Solução: Vamos utilizar uma letra qualquer, digamos a letra x, para


designar esse número desconhecido. Assim, o dobro de x é 2x e sua
quinta parte é x/5. Logo, usando as informações do enunciado, obte-

mos que:
x
2x + = 121,
5
ou ainda,

10x + x = 605,
onde 11x = 605. Resolvendo, temos que x = 605/11 = 55.

Se você já teve contato com o procedimento de resolução do exem-

plo acima, notou que o principal ingrediente é a equação do primeiro

grau em uma variável.

Denição 2.2. Uma equação do primeiro grau na variável x é uma

expressão da forma

ax + b = 0,
34 2 Equações e Inequações

onde a 6= 0, b ∈ R e x é um número real a ser encontrado.

Por exemplo, as seguintes equações são do primeiro grau:

(a) 2x − 3 = 0.

(b) −4x + 1 = 0.
3
(b) x − π = 0.
2
Para trabalhar com equações e resolvê-las, vamos pensar no mo-

delo da balança de dois pratos. Quando colocamos dois objetos com

o mesmo peso em cada prato da balança, os pratos se equilibram.

Quando os pratos estão equilibrados, podemos adicionar ou retirar a

mesma quantidade de ambos os pratos, que ainda assim eles perma-

necerão equilibrados. Essa é uma das principais propriedades quando

estamos trabalhando com uma equação. Em geral, para resolver uma

equação, utilizamos as seguintes propriedades da igualdade entre dois

números:

Propriedade 1. Se dois números são iguais, ao adicionarmos a

mesma quantidade a cada um destes números, eles ainda permane-

cem iguais. Em outras palavras, escrevendo em termos de letras, se a


e b são dois números iguais, então a+c é igual a b + c, ou seja,

a=b =⇒ a + c = b + c.

Note que podemos tomar c um número negativo, o que signica

que estamos subtraindo a mesma quantidade dos dois números. Por

exemplo, se x é um número qualquer que satisfaz

5x − 3 = 6,
2.1 Equações do Primeiro Grau 35

somando-se 3 a ambos os lados da equação acima, obtemos que x deve


satisfazer:

(5x − 3) + 3 = 6 + 3, ou seja, 5x = 9.

Propriedade 2. Se dois números são iguais, ao multiplicarmos a

mesma quantidade por cada um destes números, eles ainda permane-

cem iguais. Em outras palavras, escrevendo em termos de letras, se a


e b são dois números iguais, então a·c é igual a b · c, ou seja,

a=b =⇒ ac = bc.

Por exemplo, se 5x = 9 podemos multiplicar ambos os lados da

igualdade por 1/5 para obter

5x 9
x= = ,
5 5
encontrando o número que satisfaz a equação 5x − 3 = 6.
Para nos familiarizarmos um pouco mais com a linguagem das

equações, vamos pensar no seguinte problema:

Exemplo 2.3. Para impressionar Pedro, Lucas propôs a seguinte

brincadeira:

- Escolha um número qualquer.

- Já escolhi, disse Pedro.

- Multiplique este número por 6. A seguir, some 12. Divida o que

você obteve por 3. Subtraia o dobro do número que você escolheu. O

que sobrou é igual a 4!

Pedro realmente cou impressionado com a habilidade de Lucas.

Mas não há nada de mágico nisso. Você consegue explicar o que Lucas

fez?
36 2 Equações e Inequações

Solução: Na verdade, Lucas tinha conhecimento de como operar com

equações. Vamos ver o que Lucas fez de perto, passo a passo, utili-

zando a linguagem das equações. Para isso, vamos chamar a quanti-

dade que Pedro escolheu de x:

• Escolha um número: x.

• Multiplique este número por 6: 6x.

• A seguir, some 12: 6x + 12.


6x + 12
• Divida o que você obteve por 3: = 2x + 4.
3
• Subtraia o dobro do número que você escolheu: 2x + 4 − 2x = 4.

• O que sobrou é igual a 4!

Observação 2.4. Devemos ter cuidado na hora de efetuar divisões em


ambos os lados de uma equação, para não cometer o erro de dividir

os lados de uma igualdade por zero. Por exemplo, podemos dar uma

prova (obviamente) falsa de que 1 = 2, utilizando o seguinte tipo de

argumento: sempre é verdade que

x + 2x = 2x + x.

Logo,

x − x = 2x − 2x
Colocando (x − x) em evidência:

1(x − x) = 2(x − x)

Dividindo por (x − x) os dois lados da igualdade acima, temos que

1 = 2. Qual o erro?
2.1 Equações do Primeiro Grau 37

Para encontrar a solução da equação ax + b = 0, procedemos do

seguinte modo:

• Somamos −b a ambos os lados da equação, obtendo

ax + b + (−b) = 0 + (−b) ⇐⇒ ax = −b.

Note que como somamos a mesma quantidade aos dois lados da

equação, ela não se alterou.

• Dividimos os dois lados da equação por a 6= 0. Isso também não


altera a igualdade e nos dá que o valor de x é:

ax −b b
= ⇐⇒ x = − .
a a a

Assim, a solução da equação ax + b = 0 é

b
x=− .
a

2.1.1 Problemas Resolvidos


Vamos ver agora alguns problemas que podem ser solucionados utili-

zando as equações do primeiro grau.

Problema 2.5. Se x representa um dígito na base 10 e a soma

x11 + 11x + 1x1 = 777,

quem é x?
38 2 Equações e Inequações

Solução: Para resolver este problema, precisamos nos recordar que se

abc é a escrita de um número qualquer na base 10, então esse número

é igual a 102 a + 10b + c. Assim, temos que

x11 = 100x + 11
11x = 110 + x
1x1 = 101 + 10x
Logo, temos a seguinte equação do primeiro grau:

100x + 11 + 110 + x + 101 + 10x = 777 ou 111x + 222 = 777

Logo,
777 − 222
x= = 5.
111

Problema 2.6. Determine se é possível completar o preenchimento

do tabuleiro abaixo com os números naturais de 1 a 9, sem repetição,

de modo que a soma de qualquer linha seja igual a de qualquer coluna

ou diagonal.

1 6

9
Solução: Primeiro, observe que a soma de todos os números naturais

de 1 a 9 é 45. Assim, se denotamos por s o valor comum da soma dos

elementos de uma linha, somando as três linhas do tabuleiro, temos

que:

45 = 1 + 2 + · · · + 9 = 3s,
Onde s deve ser igual a 15. Assim, chamando de x o elemento da

primeira linha que falta ser preenchido,


2.1 Equações do Primeiro Grau 39

1 x 6

temos que 1 + x + 6 = 15. Logo, x = 8. Assim, observando a coluna


que contém 8 e 9, temos que sua soma é maior que 15. Logo, não é

possível preencher o tabuleiro de modo que todas as linhas e colunas

tenham a mesma soma.

Os quadrados de números com essas propriedades se chamam qua-

drados mágicos. Tente fazer um quadrado mágico. Você já deve ter

percebido que no centro do quadrado não podemos colocar o número

9. De fato, vamos descobrir no exemplo abaixo qual é o número que

deve ser colocado no centro de um quadrado mágico.

Problema 2.7. Descubra os valores de x de modo que seja possível

completar o preenchimento do quadrado mágico abaixo:

Solução: Para descobrir x, vamos utilizar o fato de que a soma de

qualquer linha, coluna ou diagonal é igual a 15, já obtido no exemplo

anterior. Se somarmos todas as linhas, colunas e diagonais que contêm

x, teremos que a soma será 4 · 15 = 60, pois existem exatamente uma

linha, uma coluna e duas diagonais que contêm x. Note também que

cada elemento do quadrado mágico será somado exatamente uma vez,

exceto x que será somado quatro vezes. Assim:

1 + 2 + 3 + 4 + · · · + 9 + 3x = 60,

onde temos que 45 + 3x = 60 e consequentemente x = 5.


40 2 Equações e Inequações

O problema a seguir é um fato curioso que desperta nossa atenção

para como a nossa intuição às vezes é falha.

Problema 2.8. Imagine que você possui um o de cobre extrema-

mente longo, mas tão longo que você consegue dar a volta na Terra

com ele. Para simplicar a nossa vida e nossas contas, vamos supor

que a Terra é uma bola redonda (o que não é exatamente verdade)

sem nenhuma montanha ou depressão e que seu raio é de exatamente

6.378.000 metros.

O o com seus milhões de metros está ajustado à Terra, cando

bem colado ao chão ao longo do equador. Digamos agora que você

acrescente 1 metro ao o e o molde de modo que ele forme um círculo

enorme, cujo raio é um pouco maior que o raio da Terra e tenha o

mesmo centro. Você acha que essa folga será de que tamanho?

Nossa intuição nos leva a acreditar que como aumentamos tão

pouco o o, a folga que ele vai ter será também muito pequena, di-

gamos alguns poucos milímetros. Mas veremos que isso está comple-

tamente errado!

Solução. Utilizaremos para isso a fórmula que diz que o comprimento

C de um círculo de raio r é

C = 2πr,

onde π (lê-se pi ) é um número irracional que vale aproximadamente


3, 1416 (veja a observação a seguir).
De fato, o comprimento da Terra CT calculado com essa fórmula é

aproximadamente:

CT = 2πrT ∼
= 2 × 3, 1415 × 6.378.000 = 40.072.974 metros,
2.1 Equações do Primeiro Grau 41

onde rT é o raio da Terra.

Se chamamos de x o tamanho da folga obtida em metros e rf o raio


do o, temos que a folga será igual a x = rf − rT . Logo, basta calcular

rf . Por um lado, o comprimento do o é igual a CT + 1 = 40.072.975.


Logo,
40.072.975
40.072.975 = 2πrf onde rf = .

Fazendo o cálculo acima, temos que rf é aproximadamente igual

a 6.378.000, 16 metros. Assim, x é aproximadamente igual a x =


rf − rT = 0, 16 metros, ou seja, 16 centímetros!

Observação 2.9. Vale observar que a folga obtida aumentando o o

não depende do raio em consideração. Por exemplo, se repetíssemos

esse processo envolvendo a Lua em vez da Terra, obteríamos que ao

aumentar o o em um metro, a folga obtida seria dos mesmos 16

centímetros. Verique isso!

Observação 2.10. De fato, podemos denir (e calcular!) o número

π de várias maneiras práticas. Vamos considerar dois experimentos

(que se você não conhece π deve fazer):

Experimento 1: Pegar um cinto e fazer um círculo com ele. Calcule

o comprimento do cinturão e divida pelo diâmetro do círculo obtido.

Experimento 2: Pegar uma tampa de uma lata e medir o compri-

mento do círculo da tampa e dividir pelo diâmetro da tampa.

Se você efetuou os cálculos acima com capricho, deve ter notado

que o número obtido é aproximadamente o mesmo. Se nossos círculos

fossem perfeitos (eles nunca são: sempre têm algumas imperfeições)

obteríamos o número π. Uma aproximação para π é

π∼
= 3, 1415926535897932384626433832795.
42 2 Equações e Inequações

2.2 Sistemas de Equações do Primeiro Grau

Nesta seção iremos discutir situações onde queremos descobrir mais de

uma quantidade, que se relacionam de modo linear, ou seja, através

de equações do primeiro grau. Por exemplo, considere o seguinte pro-

blema:

Exemplo 2.11. João possui 14 reais e deseja gastar esse dinheiro em

chocolates e sanduíches para distribuir com seus 6 amigos, de modo

que cada um que exatamente com um chocolate ou um sanduíche.

Sabendo que cada chocolate custa 2 reais e cada sanduíche custa 3

reais, quantos chocolates e sanduíches João deve comprar?

Para resolver esse problema, vamos chamar de x a quantidade de

chocolates que João deve comprar e y o número de sanduíches. Assim,

como João deseja gastar 14 reais, temos que

2x + 3y = 14. (2.1)

Como João comprará exatamente 6 guloseimas, uma para cada amigo,

temos que

x + y = 6. (2.2)

Note que não encontramos uma equação do primeiro grau em uma

variável e sim duas equações do primeiro grau em duas variáveis. Esse

é um caso particular de um sistema de equações do primeiro grau em

várias variáveis.

Denição 2.12. Uma equação do primeiro grau nas variáveis x1 , x2 ,


. . . , xn é uma expressão da forma

a1 x1 + a2 x2 + · · · + an xn + b = 0,
2.2 Sistemas de Equações do Primeiro Grau 43

onde os números a1 , a2 , . . . , an são diferentes de zero e b é um número

real.

Por exemplo,

2x − 3y = 0
é uma equação do primeiro grau nas variáveis x e y. Assim como,
c
2a − b + =5
3
é uma equação do primeiro grau nas variáveis a, b e c.
Dizemos que os números (r1 , r2 , . . . , rn ) formam uma solução da
equação, se substituindo x1 por r1 , x2 por r2 , . . . , xn por rn , temos

que a equação acima é satisfeita, isto é, a1 r1 +a2 r2 +· · ·+an rn +b = 0.

Por exemplo, (3, 2) é uma solução da equação 2x − 3y = 0 acima,

pois

2 · 3 − 3 · 2 = 0.
Note que a ordem que apresentamos os números importa, pois (2, 3)
não é solução da equação 2x − 3y = 0, já que 2 · 2 − 3 · 3 = −5 6= 0.
c
Do mesmo modo, (2, 0, 3) é solução da equação 2a − b + = 5, pois
3
3
2 · 2 − 0 + = 5.
3
Denição 2.13. Um sistema de equações do primeiro grau em n
variáveis x1 , x2 , . . ., xn é um conjunto de k equações do primeiro

grau em algumas das variáveis x1 , x2 , . . . , xn , isto é, tem-se o seguinte

conjunto de equações



 a11 x1 + a12 x2 + · · · + a1n xn + b1 = 0,

a x + a x + · · · + a x + b = 0,

21 1 22 2 2n n 2
(2.3)
· · · · · · · · · · · · · · · · · · · · · · · · · · · · · · · · · · ·




ak1 x1 + ak2 x2 + · · · + akn xn + bk = 0,

44 2 Equações e Inequações

onde alguns dos elementos aij (1 ≤ i ≤ k, 1 ≤ j ≤ n) podem ser zero.

Porém, em cada uma das equações do sistema algum coeciente aij é

diferente de zero e, além disso, cada variável xj aparece em alguma

equação com coeciente distinto de zero.

Dizemos que os números (r1 , r2 , . . . , rn ) formam uma solução do


sistema de equações (2.3) se (r1 , r2 , . . . , rn ) é solução para todas as

equações simultaneamente.

Quando resolvemos um sistema de equações do primeiro grau, po-

dem acontecer três situações:

(a) o sistema tem uma única solução;

(b) o sistema tem uma innidade de soluções;

(c) o sistema não possui solução.

A seguir ilustramos com exemplos cada uma das situações acima.

Situação (a): Retomamos o sistema proposto no Exemplo 2.11, o

qual se encaixa neste caso.


2x + 3y = 14,
x + y = 6.

Isolamos o valor de uma das variáveis numa das equações. Por conveni-

ência nos cálculos isolamos o valor de x na segunda equação, obtendo:

x = 6 − y.

A seguir, substituímos esse valor na outra equação, obtendo uma equa-


2.2 Sistemas de Equações do Primeiro Grau 45

ção do primeiro grau. Resolvendo temos:

2(6 − y) + 3y = 14,
12 − 2y + 3y = 14,
y = 2.

Assim, y = 2. Imediatamente, encontramos o valor de x = 6 − 2 = 4.


Vamos agora resolver alguns problemas semelhantes.

Situação (b): Consideremos os sistema de primeiro grau nas variáveis

x, y e z dado por 
x + y − z − 1 = 0,
(2.4)
x − y − 1 = 0.

Da segunda equação segue-se que

x = y + 1. (2.5)

Substituindo esta expressão na primeira equação obtemos

(y + 1) + y − z − 1 = 0,
2y − z = 0,
z = 2y. (2.6)

Notemos que as variáveis xez são resolvidas em função da variável y,


a qual não possui nenhuma restrição, de modo que se y assumir um

valor real t então x e z cam automaticamente determinadas por este

valor t. Isto é, para todo t real, de (2.5) e (2.6) tem-se que

x = t + 1, y = t, z = 2t

é solução do sistema (2.4) e, portanto, temos innitas soluções para

este.
46 2 Equações e Inequações

Situação (c): Consideremos agora o sistema de primeiro grau nas

variáveis x, y e z dado por


x + y + 2z − 1 = 0,



x + z − 2 = 0, (2.7)


y + z − 3 = 0.

Neste caso, da segunda e da terceira equação segue-se que

x=2−z e y = 3 − z.

Substituindo estas expressões na primeira equação obtém-se

(2 − z) + (3 − z) + 2z − 1 = 0 ⇐⇒ 4 = 0,

o que é uma incompatibilidade. Logo, este sistema não tem solução.

Observação 2.14. Os sistemas de equações de primeiro grau são tam-


bém conhecidos como sistemas de equações lineares. Quando um sis-

tema de equações lineares envolve muitas variáveis não é tão fácil

resolvê-lo se não se organiza com cuidado seu processo de resolução.

Existe uma teoria bem conhecida e amplamente divulgada sobre mé-

todos de resolução para esse tipo de sistemas. Um dos métodos mais

usado e eciente para resolver sistemas lineares é o método de elimi-

nação gaussiana. O leitor interessado pode consultar [7].

2.2.1 Problemas Resolvidos


O problema a seguir foi proposto na primeira fase da Olimpíada Bra-

sileira de Matemática.
2.2 Sistemas de Equações do Primeiro Grau 47

Problema 2.15. Passarinhos brincam em volta de uma velha árvore.

Se dois passarinhos pousam em cada galho, um passarinho ca voando.

Se todos os passarinhos pousam, com três em cada galho, um galho ca

vazio. Quantos são os passarinhos?

Solução: Vamos chamar de p o número de passarinhos e g o número

de galhos da árvore. Temos que se dois passarinhos pousam em cada

galho, um passarinho ca voando, ou seja,

2g = p − 1.

Além disso, se todos os passarinhos pousam, com três em um mesmo

galho, um galho ca vazio:

3(g − 1) = p.

Substituindo na equação anterior, temos que 2g = 3g − 3 − 1, onde

segue-se que g=4 e p = 9.


Problema 2.16. Quanto medem as áreas A1 e A2 na gura abaixo,

sabendo que o quadrado tem lado 1 e as curvas são arcos de círculos

com centros nos vértices V1 e V2 do quadrado, respectivamente.

V2
A2

A1

Solução: Aplicando relações de áreas na gura temos que


(
A1 + A2 = π4 ,
A1 + 2A2 = 1,
48 2 Equações e Inequações

ou seja, chegamos a um sistema de equações do primeiro grau com

duas incógnitas A1 e A2 . Da primeira equação temos que

π
A1 = − A2 ;
4
substituindo esta na segunda equação obtemos

π
− A2 + 2A2 = 1,
4
de onde
π
+ A2 = 1.
4
π
A1 = π4 − 1 − π4 = π

Logo, A2 = 1 − 4
e
2
− 1.

Problema 2.17. Carlos e Cláudio são dois irmãos temperamentais

que trabalham carregando e descarregando caminhões de cimento. Para

Carlos e Cláudio tanto faz carregar ou descarregar o caminhão, o tra-

balho realizado por eles é o mesmo. Quando estão de bem, trabalham

juntos e conseguem carregar um caminhão em 15 minutos. Cláudio

é mais forte e trabalha mais rápido conseguindo carregar sozinho um

caminhão em 20 minutos.

(a) Um dia, Cláudio adoeceu e Carlos teve que carregar os caminhões

sozinho. Quanto tempo ele leva para carregar cada um?

(b) Quando os dois brigam, Carlos costuma se vingar descarregando

o caminhão, enquanto Cláudio o carrega com sacos de cimento.

Quanto tempo Cláudio levaria para carregar o caminhão com

Carlos descarregando?

Solução: Vamos chamar de x a quantidade de sacos que Cláudio car-

rega por minuto e y a quantidade de sacos que Carlos carrega por


2.3 Equação do Segundo Grau 49

minuto. Como Cláudio carrega mais que Carlos, sabemos que y < x.
Do enunciado, sabemos que os dois juntos carregam um caminhão em

15 minutos. Se um caminhão tem capacidade para c sacos, temos que:


15x + 15y = c.
Além disso, sabemos que Cláudio sozinho carrega o mesmo caminhão

em 20 minutos. Logo,

20x = c.
Assim, igualando as duas equações, temos que

15x + 15y = 20x, onde 15y = 20x − 15x = 5x.


Logo, dividindo ambos os lados por 5, temos que 3y = x. Assim, Cláu-

dio carrega três vezes mais sacos que Carlos e a resposta do primeiro

item é 20 × 3 minutos, já que 60y = 20 × 3y = 20x = c.


Para descobrir quanto tempo os dois levam para carregar o cami-

nhão quando estão brigados, observamos que a cada minuto eles car-

regam x−y sacos, ou seja, 3y − y = 2y sacos. Logo, precisam de 30


minutos, já que 30 × 2y = 60y = c.

2.3 Equação do Segundo Grau

Como já mencionamos em nossa introdução, o conhecimento de mé-

todos para solucionar as equações do segundo grau remonta às civi-

lizações da antiguidade, como os babilônios e egípcios. Apesar disso,

a fórmula que conhecemos por fórmula de Bhaskara, em homenagem

ao matemático indiano de mesmo nome e que determina as soluções

de uma equação do segundo grau, só veio a aparecer do modo que

usamos muito mais tarde, com o francês Vièti. Nesta seção iremos

deduzir esta fórmula e aplicá-la a alguns problemas interessantes.


50 2 Equações e Inequações

2.3.1 Completando Quadrados


Um modo de resolver uma equação do segundo grau é o método de

completar quadrados. Ele consiste em escrever a equação numa forma

equivalente que nos permita concluir quais são as soluções diretamente.

Vamos ilustrar isso com um exemplo, resolvendo a equação

x2 − 6x − 8 = 0.

Podemos escrever essa equação como:

x2 − 6x = 8.

Somando 9 ao lado esquerdo, obtemos x2 − 6x + 9 que é o mesmo que


2
(x − 3) . Assim, somando 9 a ambos os lados da equação, obtemos:

(x − 3)2 = 9 + 8 = 17.
√ √
Logo, x−3= 17 x − 3 = − 17. Logo, as soluções
ou são:

√ √
x1 = 3 + 17 e x2 = 3 − 17.

Denição 2.18. A equação do segundo grau com coecientes a, b e c


é uma expressão da forma:

ax2 + bx + c = 0, (2.8)

onde a 6= 0, b, c ∈ R e x é uma variável real a ser determinada.

Para encontrar as soluções desta equação, vamos proceder do se-

guinte modo: isolando o termo que não contém a variável x do lado

direito da igualdade na equação (2.8)

ax2 + bx = −c
2.3 Equação do Segundo Grau 51

e dividindo os dois lados por a, obtemos:

b −c
x2 + x = .
a a
Agora vamos acrescentar um número em ambos os lados da equa-

ção acima, de modo que o lado esquerdo da igualdade seja um qua-


b2
drado perfeito. Para isso, observe que é necessário adicionar aos
4a2
dois lados da igualdade. Assim, temos que:

2  2
b2 b2 − 4ac

b 2 b b c
x+ =x +2 x+ = 2− = .
2a 2a 2a 4a a 4a2

Em geral, chamamos a expressão b2 −4ac de discriminante da equação


(2.8) e denotamos pela letra maiúscula ∆ (lê-se delta ) do alfabeto

grego. Assim, podemos escrever a igualdade anterior como:

2
b2 − 4ac

b ∆
x+ = 2
= 2. (2.9)
2a 4a 4a

Por isso, para que exista algum número real satisfazendo a igual-

dade acima, devemos ter que ∆ ≥ 0, já que o termo da esquerda na

igualdade é maior ou igual a zero. Extraindo a raiz quadrada quando

∆ ≥ 0, temos as soluções:

√ √
b b2 − 4ac b b2 − 4ac
x+ = e x+ =− .
2a 2a 2a 2a
Assim, obtemos as seguintes soluções:

√ √
b b2 − 4ac −b + ∆
x1 = − + =
2a 2a 2a

e
52 2 Equações e Inequações

√ √
b b2 − 4ac −b − ∆
x2 = − − = .
2a 4a2 2a

Em resumo,

• Se ∆>0 existem duas soluções reais.

• Se ∆=0 só existe uma solução real (x1 = x2 = −b/2a).

• Se ∆<0 não existe solução real.

A seguir apresentamos alguns exemplos.

Exemplo 2.19. Encontre as soluções da equação 2x2 − 4x + 2 = 0.

Solução: Observe que a = 2, b = −4 e c = 2. Logo,

∆ = b2 − 4ac = (−4)2 − 4 · 2 · 2 = 0.
b 4
Assim, a única solução é x=− = = 1.
2a 4
Exemplo 2.20. Encontre as raízes da seguinte equação do segundo

grau:

x2 − x − 1 = 0.

Solução: Basta aplicarmos diretamente a fórmula que acabamos de

deduzir. Como a = 1, b = −1 e c = −1, calculando ∆ temos:

∆ = b2 − 4ac = (−1)2 − 4 · 1 · (−1) = 5.

Logo, as soluções são


√ √ √ √
−b + ∆ 1+ 5 −b − ∆ 1− 5
x1 = = e x2 = = .
2a 2 2a 2
2.3 Equação do Segundo Grau 53

Exemplo 2.21. Sabendo que x é um número real que satisfaz

1
x=1+ ,
1
1+
x
determine os valores possíveis de x.

Solução: A solução desse problema consiste numa simples manipula-

ção algébrica, que feita com cuidado nos levará a uma equação do

segundo grau. Com efeito,

1 x+1
1+ = .
x x
Logo,
1 x 1 + 2x
1+ =1+ = .
1 1+x 1+x
1+
x
1 + 2x
Então devemos ter x= , de onde segue-se que
1+x
x2 + x = 1 + 2x ⇐⇒ x2 − x − 1 = 0.

Resolvendo a equação tem-se


√ √
1+ 5 1− 5
x1 = e x2 = .
2 2


Observação 2.22. O número (1 + 5)/2 é chamado de razão áurea.

Este número recebe essa denominação pois, frequentemente, as pro-

porções mais belas e que a natureza nos proporciona estão próximas

da razão áurea. Por exemplo, no arranjo das pétalas de uma rosa, nas

espirais que aparecem no abacaxi, na arquitetura do Parthenon, nos

quadros de da Vinci e nos ancestrais de um zangão podemos encon-

trar a razão áurea.


54 2 Equações e Inequações

O problema a seguir está relacionado com a seqüência de Fibonacci

e com a razão áurea. Dizemos que uma seqüência de números an


satisfaz a relação de Fibonacci se, para todo n ≥ 0, temos que

an+2 = an+1 + an . (2.10)

Exemplo 2.23. Encontre todas as sequências an da forma an = x n


para algum x 6= 0 que satisfazem a relação de Fibonacci.

Solução: Sabendo que an satisfaz a relação de Fibonacci e que an é da


n
forma x , podemos concluir que para todo n≥0 tem-se

xn+2 − xn+1 − xn = 0.

Colocando xn em evidência na equação acima, temos que

xn (x2 − x − 1) = 0

Logo, temos duas possibilidades: xn = 0 ou x2 − x − 1 = 0. Como


2
x 6= 0, temos que xn 6= 0 e, portanto, x − x − 1 = 0. Observando a

solução do Exemplo 2.20 temos que as únicas sequências são

√ !n √ !n
1+ 5 1− 5
an = ou an = .
2 2

Observação 2.24. Se an e bn satisfazem a relação de Fibonacci (2.10),


então dados números reais α e β, qualquer sequência da forma αan +
βbn satisfaz a relação. Pode-se provar que as sequências dessa forma,

com an = xn1 e bn = xn2 calculados anteriormente, são as únicas

sequências que satisfazem a relação. Veja, por exemplo, [4].


2.3 Equação do Segundo Grau 55

2.3.2 Relação entre Coecientes e Raízes


Dada a equação ax2 + bx + c = 0, com a 6= 0, já calculamos explicita-
mente as suas raízes x1 e x2 . Vamos estabelecer agora as relações entre

a, b e c e as raízes x1 e x2 . Vamos supor ∆ ≥ 0. Como já sabemos,


temos que
√ √
−b + ∆ −b − ∆
x1 = e x2 = .
2a 2a
Assim, somando x1 com x2 tem-se

√ √
−b + ∆ −b − ∆ −2b b
x1 + x2 = + = =− . (2.11)
2a 2a 2a a
Por outro lado, fazendo o produto x1 x2 obtemos

√ ! √ !
−b + ∆ −b − ∆
x1 x2 = ·
2a 2a
√  √ 
−b+ ∆ −b− ∆ b2 − ∆ (2.12)
= =
4a2 4a2
4ac c
= 2 = .
4a a
Em particular, quando a = 1, temos o seguinte resultado.

Teorema 2.25. Os números α e β são as raízes da equação

x2 − sx + p = 0 (2.13)

se, e somente se,

α+β =s e αβ = p. (2.14)
56 2 Equações e Inequações

Demonstração. Com efeito, se α e β são as raízes de (2.13) então os

cálculos feitos em (2.11) e (2.12) nos dão (2.14). Reciprocamente, se

vale (2.14) então da igualdade

(x − α)(x − β) = x2 − sx + p

segue-se que α e β são as raízes de (2.13).

Observação 2.26. ax2 + bx + c = 0, com


Em geral, dada a equação

a 6= 0, podemos escrevê-la como a(x2 − sx + p) = 0, com s = −b/a e


p = c/a. Supondo que a equação x2 − sx + p = 0 tem raízes α e β , a
igualdade

ax2 + bx + c = a(x2 − sx + p) = a(x − α)(x − β) (2.15)

nos permite concluir que α e β são as raízes da equação de segundo


2
grau ax + bx + c = 0.
A equação (2.15) mostra que se α é raiz de um polinômio do se-

gundo grau, então a divisão desse polinômio pelo polinômio (x − α) é

uma divisão exata. Voltaremos a tratar desse assunto no Teorema 8.5.

Exemplo 2.27. Paulo cercou uma região retangular de área 28 m2


com 24 metros de corda. Encontre as dimensões dessa região.

Solução: Se chamamos de a e b os lados do retângulo construído por

Paulo, as condições sobre o perímetro e a área desse retângulo nos

levam às seguintes equações:


(
a + b = 12,
ab = 28.

Como já observamos, a e b x2 − 12x + 28 = 0.


são raízes da equação
2
Calculando o discriminante, obtemos ∆ = 12 −4·28 = 32. Utilizando
2.3 Equação do Segundo Grau 57

a fórmula, temos que as soluções são



12 + 32 √
a= =6+2 2
2
e √
12 − 32 √
b= = 6 − 2 2.
2

Exemplo 2.28. Mostre que a equação x2 + bx + 17 = 0 não possui

raiz inteira positiva, se b é um inteiro não negativo.

Solução: Suponhamos que a equação possui alguma raiz inteira n po-


sitiva e sejam a outra raiz (podendo ser m = n). Então, n + m = −b,
onde m = −n − b deverá ser necessariamente um número inteiro. Por

outro lado, m e n são números inteiros tais que m · n = 17, o que só

é possível se m = 1 ou n = 1, o que nos daria em qualquer um dos

casos que 1 + b + 17 = 0 (b = −18), sendo isto uma contradição com

o fato de b ser inteiro não negativo.

Exemplo 2.29. Numa reunião havia pelo menos 12 pessoas e todos

os presentes apertaram as mãos entre si. Descubra quantas pessoas

estavam presentes na festa, sabendo que houve menos que 75 apertos

de mão.

Solução: Vamos denotar por a o número de apertos de mão e enumerar


as pessoas com os números do conjunto P = {1, 2, . . . , n}. A cada

aperto de mão associaremos um par (i, j), signicando que a pessoa i

apertou a mão da pessoa j. Assim, os apertos de mão envolvendo a

pessoa 1 foram


A1 = (1, 2), (1, 3), . . . , (1, n) .
58 2 Equações e Inequações

Do mesmo modo, denimos os apertos de mão envolvendo a pessoa 2


que não envolvem a pessoa 1, como


A2 = (2, 3), (2, 4), . . . , (2, n) .

Note que o aperto (2, 1) é o mesmo que o aperto (1, 2), já que se 1
aperta a mão de 2, então 2 aperta a mão de 1. Analogamente,


Ai = (i, i + 1), (i, i + 2), . . . , (i, n) , para 1 ≤ i ≤ n − 1.

Note que Ai ∩ Aj = ∅ para i 6= j . Observe também que todos os

apertos aparecem em um dos conjuntos Ai . Assim, A1 ∪ · · · ∪ An−1


contém todos os apertos de mão. Logo, se |X| denota o número de

elementos do conjunto X e a o número de apertos de mão, temos

|(A1 ∪ A2 ∪ · · · ∪ An−1 )| = |A1 | + |A2 | + · · · + |An−1 |


= (n − 1) + (n − 2) + · · · + 2 + 1
(n − 1)n
= = a.
2
Portanto, n2 − n − 2a = 0 deve admitir admite uma raiz inteira, maior
ou igual a 12. Deste modo, basta descobrirmos para que valores de

a < 75 a equação acima admite alguma raiz inteira n ≥ 12. Denotemos


as raízes da equação por n1 e n2 e suponhamos que n1 ≥ 12. Das

relações (
n1 n2 = −2a,
n1 + n2 = 1,
concluímos que −n2 = n1 − 1 ≥ 11. Assim, podemos deduzir que

−n2 n1 ≥ 11 · 12 = 132, pois −n2 ≥ 11 e n1 ≥ 12.


Observe que o mesmo raciocínio nos leva a concluir que se n1 ≥ 13
então −n2 n1 = 2a ≥ 12 · 13 = 156, o que nos daria a ≥ 78, sendo
2.3 Equação do Segundo Grau 59

isto impossível pois a < 75. Assim, a raiz positiva para tal equação

não pode ser maior ou igual que 13, restando somente n1 = 12 como
solução. De fato, essa solução é possível, se considerarmos a = 66.

Logo, haviam 12 pessoas na festa.

2.3.3 Equações Biquadradas


A dedução da solução da equação do segundo grau nos permite resolver

equações de grau mais alto, desde que elas se apresentem numa forma

peculiar, que nos permita reduzi-las a uma equação do segundo grau.

Por exemplo,

Exemplo 2.30. Resolva a equação

x4 − 2x2 + 1 = 0. (2.16)

Apesar da equação acima ser de grau quatro, podemos solucioná-la

utilizando o que aprendemos até agora. O truque será denotar por y


2
o valor x .

Solução: Denote por y = x2 . Neste caso, temos que 0 = y 2 − 2y + 1 =


(y − 1)2 . Logo, y = 1. Assim, x2 = y = 1, de onde segue-se que x = 1

ou x = −1.

De modo geral consideremos a equação

ax2k + bxk + c = 0, k ∈ N, (2.17)

e façamos a mudança y = xk . Então, a equação se transforma na

seguinte;

ay 2 + by + c = 0, (2.18)
60 2 Equações e Inequações

a qual já sabemos resolver. Logo, se (2.18) não possui solução então

(2.17) também não terá solução e no caso em que y = α seja uma

raiz de (2.18) então as soluções para (2.17), correspondentes à raiz α,


podem ser encontradas resolvendo a equação simples

xk = α,

a qual tem as seguintes possibilidades:


• uma única solução: x= k
α se k é ímpar;

• nenhuma solução: se α<0 e k é par;


• duas soluções: x=±kα se α>0 e k é par.

2.3.4 O Método de Vièti


A maneira que François Vièti (1540-1603) descobriu para resolver a

equação do segundo grau baseia-se em relacionar a equação

ax2 + bx + c = 0 (2.19)

como uma equação do tipo

Ay 2 + B = 0, (2.20)

onde A e B são números que dependem de a, b, c, de modo que qualquer


solução da equação (2.20) determinará uma solução da equação (2.19).

Note que a última equação possui soluções

r r
B B B
y1 = − e y2 = − − , se − ≥ 0.
A A A
2.3 Equação do Segundo Grau 61

Para fazer isso, usamos o seguinte truque: escrevendo x = u+v como

a soma de duas novas variáveis u e v , a equação (2.19) se escreve como

a(u + v)2 + b(u + v) + c = 0,

a qual, desenvolvendo o quadrado, equivale a

au2 + 2auv + av 2 + bu + bv + c = 0.

Agrupando convenientemente, podemos escrever a expressão acima

como uma equação na variável v, isto é,

av 2 + (2au + b)v + au2 + bu + c = 0.

Assim, podemos obter uma equação do tipo da equação (2.20), esco-

lhendo o valor de u de modo que o termo (2au + b)v se anule. Esco-

lhendo u = −b/2a temos que


 2
2 −b b b2 b2
av + a − b + c = 0 ⇐⇒ av 2 + − + c = 0,
2a 2a 4a 2a
o que é equivalente a

−b2 + 4ac
av 2 + = 0.
4a
Observando que a equação assumiu a forma da equação (2.20), temos

que suas soluções são


r r
b2 − 4ac b2 − 4ac
v1 = e v2 = − , se ∆ = b2 − 4ac ≥ 0.
4a2 4a2
Lembrando que u = −b/2a e que x=u+v temos que as soluções da

equação (2.19) são

b b
x1 = − + v1 e x2 = − + v2 ,
2a 2a
como já obtivemos anteriormente.
62 2 Equações e Inequações

2.4 Inequações

Inequações aparecem de maneira natural em várias situações dentro

do contexto matemático, assim como no próprio dia a dia.

Exemplo 2.31. Numa loja de esportes as bolas de tênis Welson en-

traram em promoção, passando a custar cada uma três reais. Pedro

que é um assíduo jogador de tênis quer aproveitar ao máximo a oferta

da loja, mas ele só dispõe de cem reais. Qual é a maior quantidade

possível de bolas que Pedro pode comprar?

Solução. Se denotamos por x o número de bolas que Pedro compra,

então devemos achar o maior valor possível de x tal que

3x ≤ 100. (2.21)

Notemos que o problema se reduz a encontrar o maior múltiplo

positivo de 3 que seja menor ou igual a 100. Observe que 99 = 3 · 33


é o maior múltiplo de 3 menor ou igual a 100, pois 3 · 34 = 102 > 100
e Pedro não teria orçamento para efetuar a compra. Logo, a solução

é x = 33, ou seja, Pedro poderá comprar 33 bolas.

Observemos que no exemplo anterior o que zemos foi achar o

maior valor inteiro de x tal que 3x−100 < 0; porém note que qualquer
número x real menor que 100/3 satisfaz que 3x − 100 < 0. Isto é um

caso particular de resolução de uma inequação, chamada inequação do

primeiro grau.
2.5 Inequação do Primeiro Grau 63

2.5 Inequação do Primeiro Grau

Uma inequação do primeiro grau é uma relação de uma das formas

abaixo

ax + b < 0, ax + b > 0,
(2.22)
ax + b ≤ 0, ax + b ≥ 0,

onde a, b ∈ R e a 6= 0.
O conjunto solução de uma inequação do primeiro grau é o con-

junto S de números reais que satisfazem a inequação, isto é, o conjunto

de números que quando substituídos na inequação tornam a desigual-

dade verdadeira. Para achar tal conjunto será de vital importância

tomar em conta as seguintes propriedades das desigualdades entre dois

números

• Invariância do sinal por adição de números reais: sejam

a e b números reais tais que a ≤ b, então a+c ≤ b+c para

qualquer número real c. O mesmo vale com as desigualdades do

tipo: <, ≥ ou >.

• Invariância do sinal por multiplicação de números reais


positivos: sejam a e b números reais tais que a ≤ b, então
ac ≤ bc para qualquer número real positivo c. Resultados aná-

logos valem para as desigualdades do tipo: <, ≥ ou >.

• Mudança do sinal por multiplicação de números reais


negativos: sejam a e b números reais tais que a ≤ b, então ac ≥
bc para qualquer número real negativo c. Resultados análogos

valem para as desigualdades do tipo: <, ≥ ou >.


64 2 Equações e Inequações

Vejamos como solucionar as inequações estritas

ax + b < 0 e ax + b > 0.
Para isto, dividimos a análise em dois casos.

• Caso 1: a>0
Inequação ax + b < 0: neste caso, dividindo por a obtemos

que x + b/a < 0 e somando −b/a, em ambos os membros desta

última inequação, temos que x < −b/a. Portanto,

S = {x ∈ R; x < −b/a},
o qual representamos no seguinte desenho:

S

−b/a

Inequação ax + b > 0: procedendo do mesmo modo que o

caso anterior, obtemos que o conjunto solução vem dado por

S = {x ∈ R; x > −b/a},
representado no desenho abaixo:

S

−b/a

• Caso 2: a<0
Inequação ax + b < 0: neste caso, quando dividimos por ao
sinal da inequação se inverte, obtendo assim que x + b/a > 0,
logo temos que x > −b/a e, consequentemente,

S = {x ∈ R; x > −b/a},
cuja representação na reta é a seguinte:
2.5 Inequação do Primeiro Grau 65

S

−b/a

Inequação ax + b > 0: similarmente, o conjunto solução vem

dado por

S = {x ∈ R; x < −b/a},
cuja representação é a seguinte:

S

−b/a

Observação 2.32. Notemos que se queremos resolver as inequações

ax + b ≤ 0 e ax + b ≥ 0, então o conjunto solução S em cada um

dos casos acima continua o mesmo acrescentado apenas do ponto x=


−b/a.

Vejamos agora um exemplo simples.

Exemplo 2.33. Para resolver a inequação 8x − 4 ≥ 0, primeiramente

dividimos por 8 a inequação (prevalecendo o sinal da desigualdade)

e imediatamente adicionamos 1/2 em ambos os membros da mesma,

para obter x − 4/8 + 1/2 ≥ 1/2, ou seja,

S = {x ∈ R; x ≥ 1/2}.

A seguir damos alguns exemplos que podem ser resolvidos usando

inequações lineares.

Exemplo 2.34. Sem fazer os cálculos, diga qual dos números

a = 3456784 · 3456786 + 3456785 e b = 34567852 − 3456788

é maior?
66 2 Equações e Inequações

Solução. Se chamamos de x ao número 3456784 então das denições


2
de a e b temos que a = x · (x + 2) + (x + 1) e b = (x + 1) − (x + 4).
2 2
Logo, a = x + 3x + 1 e b = x + x − 3. Se supomos que a ≤ b, então

x2 + 3x + 1 ≤ x2 + x − 3,

e somando −x2 −x+3 a ambos os membros desta desigualdade obtemos

2x + 4 ≤ 0.

A solução desta inequação do primeiro grau é o conjunto dos x∈R


tais que x ≤ −2, mas isto é falso, desde que x = 3456784. Logo,
nossa suposição inicial de a ser menor ou igual a b é falsa, sendo então

a > b.

O próximo exemplo já foi tratado antes (ver Problema 2.7), porém

apresentamos a seguir uma solução diferente usando inequações do

primeiro grau.

Exemplo 2.35. Um quadrado mágico 3×3 é um quadrado de lado 3

dividido em 9 quadradinhos de lado 1 de forma tal que os números de 1

até 9 são colocados um a um em cada quadradinho com a propriedade

de que a soma dos elementos de qualquer linha, coluna ou diagonal é

sempre a mesma. Provar que no quadradinho do centro de tal quadrado

mágico deverá aparecer, obrigatoriamente, o número 5.

Solução. Primeiramente observamos que a soma 1+2+3+···+9 =


45, logo como há três linhas e em cada uma destas guram números

diferentes temos que a soma dos elementos de cada linha é 15. Logo,

a soma dos elementos de cada coluna ou diagonal também é 15.

Chamemos de x o número que aparece no centro do quadrado

mágico, como mostra o desenho a seguir.


2.5 Inequação do Primeiro Grau 67

Agora fazemos as seguintes observações:

• O número x não pode ser 9, pois nesse caso em alguma linha,

coluna ou diagonal que contém o quadrado central aparecerá

o número 8, que somado com 9 dá 17 > 15 e isto não pode

acontecer.

• O número x não pode ser 1, pois nesse caso formaria uma linha,

coluna ou diagonal com o número 2 e um outro número que

chamamos de y, então 1 + 2 + y = 15 ⇔ y = 12, o qual é

impossível.

Feitas as observações anteriores, temos então que o número x forma


uma linha, coluna ou diagonal com o número 9 e algum outro número

que chamamos de z, logo

z = 15 − (x + 9) ≥ 1 ⇔ 6 − x ≥ 1,

de onde segue que x ≤ 5.


x aparece numa linha, coluna ou diagonal
Por outro lado, o número

com o número 1 e algum outro número que chamamos de s, logo

s = 15 − (x + 1) = 14 − x ≤ 9, de onde temos que x ≥ 5. Finalmente,


como 5 ≤ x ≤ 5 segue-se que x = 5.

Exemplo 2.36. Num triângulo com lados de comprimento a, b e c


traçamos perpendiculares desde um ponto arbitrário P, sobre o lado

de comprimento c, até cada um dos lados restantes (ver a Figura 2.1).

Se estas perpendiculares medem x e y e a > b, então


68 2 Equações e Inequações

(a) Qual a posição onde deve ser colocado P de maneira que ` = x+y
seja mínimo?

(b) Qual a posição onde deve ser colocado P de maneira que ` = x+y
seja máximo?

C
a
b
x P y
B c A

Figura 2.1: No desenho, os segmentos que partem do ponto P são perpen-

diculares aos lados AC e BC

Solução. Denotemos por S a área do triângulo e notemos que divi-

dindo este em dois triângulos menores: um com base a e altura x e

outro com base b e altura y , temos que


ax by
+ = S,
2 2
de onde se segue que

ax = 2S − by
2S − by
x= .
a
Somando y em ambos os lados da última igualdade, obtemos

2S − by
x+y = +y
a
2S − by + ay
=
a
2S a − b
= + y,
a a
2.6 Inequação do Segundo Grau 69

logo

` = α + βy,

onde
2S a−b
α= e β = .
a a
Agora notemos que 0 ≤ y ≤ hb , onde hb denota a altura relativa ao

lado de comprimento b no triângulo dado. Como β é positivo, por ser

a > b, temos então que 0 ≤ βy ≤ βhb e, portanto, α ≤ α + βy ≤


α + βhb , de onde
0 ≤ ` ≤ α + βhb .

Resumindo, o valor mínimo de ` é atingido quando y = 0, portanto


P deve ser colocado no vértice A, e o valor máximo é obtido quando
y = hb , portanto P deve ser colocado no vértice B .

2.6 Inequação do Segundo Grau

Agora passamos a discutir a solução das inequações do segundo grau,

que possuem um maior grau de diculdade quando comparadas com

as inequações do primeiro grau. Será de vital importância o uso das

propriedades da função quadrática ax2 + bx + c, estudadas no capítulo


anterior.

Uma inequação do segundo grau é uma relação de uma das formas

abaixo 
ax2 + bx + c < 0, ax2 + bx + c > 0,
(2.23)
ax2 + bx + c ≤ 0, ax2 + bx + c ≥ 0,

onde a, b, c ∈ R e a 6= 0. Por simplicidade, chamaremos o número a de


2
coeciente líder da função quadrática ax + bx + c.
70 2 Equações e Inequações

Por exemplo, para resolver a inequação x2 − 3x + 2 > 0 fatoramos


2
o trinômio usando que as raízes da equação x − 3x + 2 = 0 são 1 e 2,

isto é,

x2 − 3x + 2 = (x − 1)(x − 2).

O trinômio toma valores positivos quando o produto (x − 1)(x − 2) for


positivo, ou seja, quando os fatores (x − 1) e (x − 2) tenham o mesmo

sinal:

• Ambos positivos:

x−1>0⇔x>1

x − 2 > 0 ⇔ x > 2,

logo x > 2.

• Ambos negativos:

x−1<0⇔x<1

x − 2 < 0 ⇔ x < 2,

logo x < 1.

Portanto, x2 − 3x + 2 > 0 se, e somente se, x<1 ou x > 2.


A seguir explicamos como podemos resolver a inequação do se-

gundo grau de forma geral.

Suponhamos primeiramente que queremos resolver a inequação

ax2 + bx + c > 0. (2.24)


2.6 Inequação do Segundo Grau 71

Notemos que valem as seguintes igualdades:

 
2 b 2 c
ax + bx + c = a x + x +
a a
b2 b2
 
2 b c
=a x + x+ 2 − 2 +
a 4a 4a a
 2
  2  (2.25)
2 b b b c
=a x + x+ 2 −a −
a 4a 4a2 a
 2
b ∆
=a x+ − ,
2a 4a

onde ∆ = b2 − 4ac. Considerando esta igualdade, dividimos em vários

casos:

Caso 1: ∆ = b2 − 4ac > 0. Nesta situação procedemos tomando

em conta o sinal de a.

• (a > 0). Usando (2.25) notamos que basta resolver a inequação

 2
b ∆
a x+ − > 0.
2a 4a

Como a > 0, multiplicando por 1/a em ambos os membros da

desigualdade anterior o sinal desta não muda, obtendo-se então

 2
b ∆
x+ − 2 > 0.
2a 4a

Agora usamos que ∆>0 para obtermos que


72 2 Equações e Inequações

 2  2 √ !2
b ∆ b ∆
x+ − 2 = x+ −
2a 4a 2a 2a
√ ! √ !
b+ ∆ b− ∆
= x+ x+
2a 2a
√ ! √ !
−b − ∆ −b + ∆
= x− x−
2a 2a
= (x − α)(x − β) > 0,
√ √
−b− ∆ −b+ ∆
onde α= 2a
e β= 2a
são as raízes de ax2 + bx + c = 0.
Agora notamos que (x − α)(x − β) > 0 se os fatores (x − α) e

(x − β) são ambos positivos ou ambos negativos. No primeiro

caso (ambos positivos) temos que x > α e x > β, mas como

α < β , então x > β . No segundo caso (ambos negativos), temos


que x < α e x < β , logo x < α, novamente por ser α < β .

Resumindo, a solução da inequação vem dada pelo conjunto

S = {x ∈ R; x < α ou x > β},

com a seguinte representação na reta:

S S
• •
α β

• (a < 0). Esta situação é bem similar à anterior, a única dife-

rença é que ao multiplicar por 1/a o sinal se inverte tendo então


que resolver a inequação
 2
b ∆
x+ − < 0,
2a 4a2
2.6 Inequação do Segundo Grau 73

a qual é equivalente a provar (seguindo os mesmos passos do

caso anterior) que

(x − α)(x − β) < 0,
√ √
−b− ∆ −b+ ∆
com α = 2a
e β = 2a
raízes de ax2 + bx + c = 0.
Notemos que a desigualdade acima é válida sempre que os sinais

dos fatores (x − α) e (x − β) forem diferentes. Por exemplo,


se x − α > 0 e x − β < 0 temos então que x deve satisfazer a

desigualdade α < x < β , mas isso é impossível considerando que

neste caso α > β , por ser a < 0. No caso restante, se x − α < 0

e x − β > 0 temos então que β < x < α, o que é possível.

Portanto, o conjunto solução, neste caso, é dado por

S = {x ∈ R; β < x < α},

cuja representação na reta é:

S
• •
β α

Caso 2: ∆ = b2 − 4ac = 0. Usando novamente (2.25), devemos

resolver a inequação
 2
b
a x+ > 0,
2a
b
a qual é válida para qualquer x 6= − 2a , se a > 0 e sempre falsa, se

a < 0.
Caso 3: ∆ = b2 − 4ac < 0. Neste caso, quando a é positivo todos

os valores de x reais são solução para (2.24), pois a desigualdade


 2
2 b ∆
ax + bx + c = a x + − > 0,
2a 4a
74 2 Equações e Inequações


é sempre satisfeita, dado que − 4a > 0. Por outro lado, se a é negativo
não temos nenhuma solução possível para a inequação (2.24) já que

 2
2 b ∆
ax + bx + c = a x + −
2a 4a

é sempre negativo, dado que − 4a < 0.

Observação 2.37. Para a desigualdade do tipo

ax2 + bx + c < 0

são obtidos resultados similares, seguindo o mesmo processo descrito

anteriormente. Além disso, para as inequações

ax2 + bx + c ≥ 0 e ax2 + bx + c ≤ 0

os resultados são os mesmos, acrescentados apenas dos pontos α, β


ou −b/2a, dependendo do caso.

Exemplo 2.38. Provar que a soma de um número positivo com seu

inverso é sempre maior ou igual que 2.

Solução. Seja x > 0, então devemos provar que

1
x+ ≥ 2.
x
Partimos da seguinte desigualdade, que sabemos vale para qualquer

x ∈ R:
(x − 1)2 ≥ 0

logo

x2 − 2x + 1 ≥ 0 ⇐⇒ x2 + 1 ≥ 2x.
2.6 Inequação do Segundo Grau 75

Se x é positivo, podemos dividir ambos os membros da última desi-

gualdade sem alterar o sinal da mesma, ou seja,

1
x+ ≥ 2,
x
conforme queríamos provar.

2.6.1 Máximos e Mínimos das Funções Quadráti-


cas
A função quadrática f (x) = ax2 + bx + c, como já foi observado ante-

riormente, satisfaz a identidade


 2
2 b ∆
ax + bx + c = a x + − , (2.26)
2a 4a
onde ∆ = b2 − 4ac. O valor mínimo (máximo ) da função quadrática

f (x) é o menor (maior) valor possível que pode assumir f (x) quando

fazemos x percorrer o conjunto dos reais.

Da igualdade (2.26) segue-se que, quando a > 0 o valor mínimo


b b ∆
do trinômio é obtido quando x = − 2a e este vale f (− 2a ) = − 4a .

Similarmente, quando a < 0 o valor máximo do trinômio é obtido


b b ∆
quando x= − 2a , valendo também f (− 2a ) = − 4a
Exemplo 2.39. Sejam a, b reais positivos tais que a + b = 1. Provar

que ab ≤ 1/4.
Solução. Notemos que ab = a(1 − a) = −a2 + a. Denindo f (a) =
−a2 + a, basta provar que f (a) ≤ 1/4 para qualquer 0 < a < 1.
Completando o quadrado a função f (a), obtemos

f (a) = −(a2 − a) = −(a2 − a + 1/4 − 1/4) = −(a − 1/2)2 + 1/4,

logo este assume seu valor máximo igual a 1/4, quando a = 1/2.
76 2 Equações e Inequações

Alguns problemas de máximos ou mínimos não parecem que pos-

sam ser resolvidos achando o máximo ou mínimo de funções quadrá-

ticas. Porém, estes problemas podem ser reformulados de forma tal

que isto seja possível. Vejamos um exemplo.

Exemplo 2.40. Na gura abaixo ABCD é um retângulo inscrito den-


tro do círculo de raio r. Encontre as dimensões que nos dão a maior

área possível do retângulo ABCD.

D C
r y
x

A B

Solução. A área do retângulo vem dada pela fórmula

A = 2x · 2y = 4xy.

Usando o teorema de Pitágoras, temos que


y= r 2 − x2 , (2.27)

logo, substituindo esta última igualdade na fórmula de área anterior,

obtemos

A = 4x r2 − x2 .

Não é muito difícil nos convencermos de que as dimensões, que nos

dão a maior área possível para o retângulo ABCD, são as mesmas

que nos dão o máximo para o quadrado desta área, ou seja, basta

encontrar as dimensões que maximizam A2 . A vantagem que tem esta


2.7 Miscelânea 77

reformulação do problema é que A2 tem uma expressão mais simples,

dada por

A2 = 16x2 (r2 − x2 ) = 16r2 x2 − 16x4 .

Agora fazemos a mudança z = x2 , para obter

 2
r2
A2 = −16z 2 + 16r2 z = −16 z − 2
+ 4r4 ,

r2
de onde segue que o menor valor de A2 é obtido quando z = 2
e

portanto quando x= √r . Usando agora a igualdade (2.27) temos que


2
r
r2 r
y= r2 − =√ .
2 2
Então, o retângulo de maior área possível é o quadrado de lado
2r


2
= r 2.

2.7 Miscelânea

Nesta seção combinamos a teoria desenvolvida nos tópicos anteriores

para resolver outros tipos de equações com um nível de complexidade

maior.

2.7.1 Equações Modulares


Uma equação modular é aquela na qual a variável incógnita aparece

sob o sinal de módulo. Por exemplo, são equações modulares

(a) |2x − 5| = 3;

(b) |2x − 3| = 1 − 3x;


78 2 Equações e Inequações

(c) |3 − x| − |x + 1| = 4.

Para resolver equações modulares se usam basicamente três méto-

dos:

(1) eliminação do módulo pela denição;

(2) elevação ao quadrado de ambos os membros da equação;

(3) partição em intervalos.

Ilustramos a seguir estes métodos com os exemplos dados em (a),

(b) e (c).

Exemplo 2.41. Resolver a equação |2x − 5| = 3.

Solução: O método (1) pode ser utilizado para resolver esta equação.

Para isto, usamos a denição de módulo:



 a se a ≥ 0,
|a| =
−a se a < 0.

de onde segue-se a propriedade: seja b um número não negativo, então

|a| = b ⇐⇒ a = b ou a = −b.

Logo, x é solução da equação se, e somente se, x satisfaz uma das

equações de primeiro grau a seguir:

2x − 5 = 3 ou 2x − 5 = −3.

Da primeira equação obtemos a solução x1 = 4 e da segunda obtemos

a solução x2 = 1.
2.7 Miscelânea 79

Exemplo 2.42. Resolver a equação |2x − 3| = 1 − 3x

Solução: Resolveremos esta equação pelos métodos (1) e (2).

Método (1): Aplicando a denição de módulo temos que resolver a


equação é equivalente a resolver os sistemas mistos

 
2x − 3 ≥ 0, 2x − 3 < 0,
(a) ou (b)
2x − 3 = 1 − 3x, −(2x − 3) = 1 − 3x.

O sistema (a) não tem solução visto que a solução da equação do

primeiro grau 2x − 3 = 1 − 3x ⇔ 5x = 4 é x = 4/5 a qual não satisfaz


a desigualdade 2x − 3 ≥ 0. Por outro lado, no sistema (b) a solução

da equação −(2x − 3) = 1 − 3x tem por solução x = −2 a qual satisfaz

a inequação 2x − 3 < 0. Logo, a única solução da equação é x = −2.

Método (2): Observemos que a equação é equivalente ao sistema


misto 
1 − 3x ≥ 0,
(2x − 3)2 = (1 − 3x)2 .

Resolvendo agora a equação de segundo grau (2x − 3)2 = (1 − 3x)2 a


2
qual é equivalente a 5x + 6x − 8 = 0, temos que as possíveis soluções
sãox1 = 4/5 e x2 = −2, mas x1 é descartada pois não satisfaz que
1−3x1 ≥ 0. Assim, a solução do sistema misto e, portanto, da equação
modular é apenas x2 = −2.

Exemplo 2.43. |3 − x| − |x + 1| = 4.
Solução. Neste caso usaremos o método de partição em intervalos que

consiste no seguinte: marcamos na reta real os valores onde |3 − x|


e |x + 1| se anulam, neste caso, x1 = 3 e x2 = −1. Com isto a reta
80 2 Equações e Inequações

numérica é dividida em 3 intervalos x < −1, −1 ≤ x ≤ 3 e x > 3.


Agora analisamos a equação em cada intervalo:

Intervalo x < −1: Neste caso a equação modular toma a forma

3 − x − (−x − 1) = 4 ⇐⇒ 4 = 4,

Portanto, todo o intervalo x < −1 é solução.

Intervalo −1 ≤ x ≤ 3: Neste caso a equação modular toma a forma

3 − x − (x + 1) = 4 ⇐⇒ 2 − 2x = 4,

de onde segue-se que x = −1. Portanto, neste intervalo a solução é

x = −1.
Intervalo x > 3: Neste caso a equação modular toma a forma

−3 + x − (x + 1) = 4 ⇐⇒ −4 = 4,

o que é uma contradição. Portanto, neste intervalo não temos solução.

Em resumo, a solução da equação modular é o intervalo x ≤ −1.

2.7.2 Um Sistema de Equações Não lineares


O seguinte exemplo nos mostra como podemos combinar a técnica de

resolução de sistemas lineares e de equações de segundo grau para

resolver sistemas mais complicados.

Exemplo 2.44. Resolva o sistema de equações:


√
 x2 + 3x − (x2 − 2)3 = 3,

 x2 + 3x + (x2 − 2)3 = 1.
2.8 Exercícios 81

Solução. Propomos a seguinte mudança de variáveis:


u= x2 + 3x e v = (x2 − 2)3 .

Assim, o sistema se converte no sistema de equações do primeiro grau



u − v = 3,
u + v = 1,

o qual tem como solução u=2 e v = −1. Verique! Assim,


x2 + 3x = 2 ⇐⇒ x2 + 3x = 4,

sendo x=1 e x = −4 as soluções desta equação do segundo grau.

Por outro lado

(x2 − 2)3 = −1,


de onde x2 − 2 = −1, sendo x = 1 e x = −1 as soluções desta equação.
Logo, a solução do sistema é x = 1, que é a única que satisfaz u = 2

e v = −1 simultaneamente.

2.8 Exercícios

1. Observe as multiplicações a seguir:

(a) 12.345.679 × 18 = 222.222.222


(b) 12.345.679 × 27 = 333.333.333
(c) 12.345.679 × 54 = 666.666.666

Para obter 999.999.999 devemos multiplicar 12.345.679 por quan-


to?
82 2 Equações e Inequações

2. Outro dia ganhei 250 reais, incluindo o pagamento de horas

extras. O salário (sem horas extras) excede em 200 reais o que

recebi pelas horas extras. Qual é o meu salário sem horas extras?

3. Uma torneira A enche sozinha um tanque em 10 h, uma torneira


B enche o mesmo tanque sozinha em 15 h. Em quantas horas

as duas torneiras juntas encherão o tanque?

4. O dobro de um número, mais a sua terça parte, mais a sua quarta

parte somam 31. Determine o número.

5. Uma certa importância deve ser dividida entre 10 pessoas em

partes iguais. Se a partilha fosse feita somente entre 8 dessas

pessoas, cada uma destas receberia R$5.000,00 a mais. Calcule

a importância.

6. Roberto disse a Valéria: Pense um número, dobre esse número,

some 12 ao resultado, divida o novo resultado por 2. Quanto

deu?  Valéria disse 15 ao Roberto, que imediatamente reve-

lou o número original que Valéria havia pensado. Calcule esse

número.

7. Por 2/3 de um lote de peças iguais, um comerciante pagou

R$8.000,00 a mais do que pagaria pelos 2/5 do mesmo lote. Qual

o preço do lote todo?

3a+6 2a+10
8. Determine um número real a para que as expressões 8
e
6
sejam iguais.

9. Se você multiplicar um número real x por ele mesmo e do resul-

tado subtrair 14, você vai obter o quíntuplo do número x. Qual

é esse número?
2.8 Exercícios 83

10. Eu tenho o dobro da idade que tu tinhas quando eu tinha a

tua idade. Quando tu tiveres a minha idade, a soma das nossas

idades será de 45 anos. Quais são as nossas idades?

11. Um homem gastou tudo o que tinha no bolso em três lojas. Em

cada uma gastou 1 real a mais do que a metade do que tinha ao

entrar. Quanto o homem tinha ao entrar na primeira loja?

12. Com os algarismos x, y e z formam-se os números de dois alga-

rismos xy yx, cuja soma


e é o número de três algarismos zxz .
Quanto valem x, y e z ?

13. Quantos são os números inteiros de 2 algarismos que são iguais

ao dobro do produto de seus algarismos?

14. Obter dois números consecutivos inteiros cuja soma seja igual a

57.

15. Qual é o número que, adicionado ao triplo do seu quadrado, vale

14?

16. O produto de um número positivo pela sua terça parte é igual a

12. Qual é esse número?

17. Determine dois números consecutivos ímpares cujo produto seja

195.

18. A diferença entre as idades de dois irmãos é 3 anos e o produto

de suas idades é 270. Qual é a idade de cada um?

19. Calcule as dimensões de um retângulo de 16 cm de perímetro e

15 cm2 de área.
84 2 Equações e Inequações

8
20. A diferença de um número e o seu inverso é . Qual é esse
3
número?

21. A soma de dois números é 12 e a soma de seus quadrados é 74.

Determine os dois números.

22. Um pai tinha 30 anos quando seu lho nasceu. Se multiplicarmos

as idades que possuem hoje, obtém-se um produto que é igual

a três vezes o quadrado da idade do lho. Quais são as suas

idades?

23. Os elefantes de um zoológico estão de dieta juntos. Num período

de 10 dias devem comer uma quantidade de cenouras igual ao

quadrado da quantidade que um coelho come em 30 dias. Em

um dia os elefantes e o coelho comem juntos 1.444 kg de cenoura.

Quantos kilos de cenoura os elefantes comem em 1 dia?

24. Sejam α1 e α2 as raízes do polinômio ax2 + bx + c, com a 6= 0.


Calcule as seguintes expressões, em função de a, b e c:

α1 + α2
(a) ;
2
√ √
(b) α1 + α2 ;
√ √
4 α + 4 α .
(c) 1 2

25. O número −3 é a raiz da equação x2 − 7x − 2c = 0. Nessas

condições, determine o valor do coeciente c.

26. Encontre o polinômio p(x) = 2x4 +bx3 +cx2 +dx+e que satisfaz
a equação p(x) = p(1 − x).
2.8 Exercícios 85

27. (OBM) Dois meninos jogam o seguinte jogo. O primeiro esco-

lhe dois números inteiros diferentes de zero e o segundo monta

uma equação do segundo grau usando como coecientes os dois

números escolhidos pelo primeiro jogador e 1.998, na ordem

que quiser (ou seja, se o primeiro jogador escolhe a e b o se-


gundo jogador pode montar a equação 1.998x2 + ax + b = 0 ou
ax2 + 1.998x + b = 0 etc.) O primeiro jogador é considerado ven-

cedor se a equação tiver duas raízes racionais diferentes. Mostre

que o primeiro jogador pode ganhar sempre.

28. (OBM) Mostre que a equação x2 + y 2 + z 2 = 3xyz tem innitas

soluções onde x, y, z são números inteiros.

29. (Gazeta Matemática, Romênia) Considere a equação

a2 x2 − (b2 − 2ac)x + c2 = 0,

onde a, b e c são números inteiros positivos. Se n ∈ N é tal que

p(n) = 0, mostre que n é um quadrado perfeito.

30. (Gazeta Matemática, Romênia) Sejam a, b ∈ Z. Sabendo que a

equação

(ax − b)2 + (bx − a)2 = x,

tem uma raiz inteira, encontre os valores de suas raízes.

31. (Gazeta Matemática, Romênia) Resolva a equação:

2x2
 
= x.
x2 + 1
Obs.: [x] é o menor inteiro maior ou igual a x.
86 2 Equações e Inequações

32. Demonstrar que:

(a) n4 + 4 não é primo se n > 1;


(b) generalize, mostrando que n4 + 4n não é primo, para todo

n > 1.

33. Para fazer 12 bolinhos, preciso exatamente de 100 g de açúcar,

50 g de manteiga, meio litro de leite e 400 g de farinha. Qual a

maior quantidade desses bolinhos que serei capaz de fazer com

500 g de açúcar, 300 g de manteiga, 4 litros de leite e 5 kg de

farinha ?

34. Dadas as frações

966666555557 966666555558
e ,
966666555558 966666555559
qual é maior?

35. Achar o maior valor inteiro positivo de n tal que

n200 < 5300 .

36. Achar o menor valor inteiro positivo de n tal que

1 2 3 n
10 11 · 10 11 · 10 11 · · · 10 11 > 100000.

37. Nove cópias de certas notas custam menos de R$ 10,00 e dez

cópias das mesmas notas (com o mesmo preço) custam mais de

R$ 11,00. Quanto custa uma cópia das notas?

38. Se enumeram de 1 até n as páginas de um livro. Ao somar estes

números, por engano um deles é somado duas vezes, obtendo-se

o resultado incorreto: 1.986. Qual é o número da página que foi

somado duas vezes?


2.8 Exercícios 87

39. Determine os valores de a para os quais a função quadrática


2
ax − ax + 12 é sempre positiva.

40. Ache os valores de x para os quais cada uma das seguintes ex-

pressões é positiva:

x x−3 x2 − 1
(a) (b) (c)
2
x +9 x+1 x2 − 3x
41. Resolver a equação:

[x]{x} + x = 2{x} + 10,

onde [x] denota a parte inteira de x. Por exemplo, [2, 46] = 2 e

[5, 83] = 5. O número {x} é chamado parte fracionária de x e é

denido por {x} = x − [x].

42. Mostre que entre os retângulos com um mesmo perímetro, o de

maior área é um quadrado.

43. Entre todos os triângulos isósceles com perímetro p xado, ache

as dimensões dos lados daquele que possui a maior área.

44. (OBM Júnior 1993)

É dada uma equação do segundo grau x2 + ax + b = 0, com

raízes inteiras a1 e b1 . Consideramos a equação do segundo grau


x + a1 x + b1 = 0. Se a equação x2 + a1 x + b1 = 0 tem raízes
2

2
inteiras a2 e b2 , consideramos a equação x + a2 x + b2 = 0. Se a
2
equação x +a2 x+b2 = 0 tem raízes inteiras a3 e b3 , consideramos
2
a equação x +a3 x+b3 = 0. E assim por diante. Se encontramos

uma equação com ∆ < 0 ou com raízes que não sejam inteiros,

encerramos o processo.
88 2 Equações e Inequações

Por exemplo, se começamos com a equação x2 = 0 podemos

continuar o processo indenidamente. Pede-se:

(a) Determine uma outra equação que, como x2 = 0, nos per-

mita continuar o processo indenidamente;

(b) Determine todas as equações do segundo grau completas a

partir das quais possamos continuar o processo indenida-

mente.
Referências Bibliográcas
[1] AIGNER, M. e ZIEGLER, G. (2002). As Provas estão

no Livro. Edgard Blücher.

[2] GARCIA, A. e LEQUAIN, I. (2003). Elementos de Ál-

gebra. Projeto Euclides, IMPA.

[3] LIMA, E. L.; CARVALHO, P. C. P.; WAGNER, E. e

MORGADO, A.C. (2004). A Matemática do Ensino Mé-

dio. Volume 1. Sociedade Brasileira de Matemática.

[4] LIMA, E.L.; CARVALHO, P. C. P.; WAGNER, E. e

MORGADO, A.C. (2004). A Matemática do Ensino Mé-

dio. Volume 2. Sociedade Brasileira de Matemática.

[5] LIMA,E.L.; CARVALHO,P. C. P.; WAGNER,E. e

MORGADO,A.C. (2004). A Matemática do Ensino Mé-

dio. Volume 3. Sociedade Brasileira de Matemática.

[6] LIMA, E.L.; CARVALHO, P. C. P.; WAGNER,E. e

MORGADO, A.C. (2001). Temas e Problemas. Socie-

dade Brasileira de Matemática.

[7] LIMA, E.L. (2001). Álgebra Linear. Sociedade Brasileira

de Matemática.

285
286 REFERÊNCIAS BIBLIOGRÁFICAS

[8] MORAIS FILHO, D. C. (2007). Um Convite à Matemá-

tica. EDUFCG.

[9] MORGADO, A.; CARVALHO, J.; CARVALHO, P.;

FERNANDEZ, P. (1991). Análise Combinatória e Pro-

babilidade . Sociedade Brasileira de Matemática.

[10] RIBENBOIM, P. (2001). Números Primos: Mistérios e

Recordes. Sociedade Brasileira de Matemática.

[11] SANTOS, J. P. O. (1993) Introdução à Teoria dos Nú-

meros. IMPA.

[12] SANTOS, J. P. O.; MELLO, M. P. e MURARI, I. T.

C. (2006). Introdução à Análise Combinatória. Editora

Unicamp.

[13] SOARES, M. G. (2005). Cálculo em uma Variável Com-

plexa. Sociedade Brasileira de Matemática.


Mestrado Profissional
em Matemática em Rede Nacional

Iniciação à Matemática

Autores:

Krerley Oliveira Adán J. Corcho

Unidade II:

Capítulos III e IV
3
Divisibilidade

Os números governam o mundo.

Platão

A teoria dos números é o ramo da Matemática que estuda os mis-

térios dos números e teve sua origem na antiga Grécia. Os belíssimos

problemas ligados a esta área constituem, até hoje, uma das princi-

pais fontes inspiradoras dos amantes da Matemática. Além disso, essa

área possui várias aplicações úteis a humanidade, como por exemplo,

o processo de criptograa usado em transações pela Internet.

Alguns problemas em teoria dos números demoram séculos para

serem resolvidos, como por exemplo o último teorema de Fermat, que


arma que não existe nenhum conjunto de inteiros positivos x, y, z e n

com n maior que 2 que satisfaça xn + y n = z n . Esse problema foi ob-


jeto de fervorosas pesquisas durante mais de 300 anos e foi nalmente

demonstrado em 1995 pelo matemático Andrew Wiles.

Ainda hoje persistem muitas questões naturais e simples sem res-

posta. Por exemplo, ninguém sabe mostrar (apesar de todo mundo

89
90 3 Divisibilidade

acreditar que é verdade!) todo natural par é soma de dois pri-


que

mos. Essa é a famosa conjectura de Goldbach. Essa simplicidade de se


anunciar problemas e a extrema diculdade em resolvê-los faz desta

área um grande atrativo para os matemáticos do mundo todo.

Este capítulo será dedicado ao estudo de algumas propriedades

básicas relativas aos números inteiros.

3.1 Conceitos Fundamentais e Divisão Eu-

clidiana

Denotamos por Z o conjunto dos números inteiros formado pelo con-

junto dos números naturais N = {1, 2, 3, . . .} munido do zero e dos

números negativos. Ou seja, Z = {. . . , −3, −2, −1, 0, 1, 2, 3, . . .}.


Começamos observando que a soma, diferença e produto de núme-

ros inteiros também serão números inteiros. Entretanto, o quociente

de dois inteiros pode ser um inteiro ou não.

Uma das propriedades fundamentais dos números naturais que uti-

lizaremos ao longo do texto é o conhecido princípio da boa ordenação,


que arma o seguinte:

Princípio da Boa Ordenação: todo subconjunto não vazio A ⊆ N


possui um elemento menor que todos os outros elementos deste, ou
seja, existe a ∈ A tal que a ≤ n para todo n ∈ A.
Por exemplo, se A é o conjunto dos números pares, o menor ele-

mento de A é o número 2. Por outro lado, observamos que o conjunto

dos números inteiros não goza da boa ordenação.

Apesar do princípio da boa ordenação parecer inocente e natural,

muitos resultados importantes a respeito dos números naturais decor-


3.1 Conceitos Fundamentais e Divisão Euclidiana 91

rem do mesmo, como veremos ao longo de todo este capítulo.

Denição 3.1. Sejam a e b inteiros. Dizemos que a divide b se existe


um inteiro q tal que b = aq . Também usaremos as frases a é divisor

de b ou b é múltiplo de a para signicar esta situação.

Usaremos a notação a | b para representar todas as frases equi-

valentes ditas anteriormente. Se a não for divisor de b, então escre-

veremos a - b.

Exemplo 3.2. 7 | 21 pois 21 = 7 · 3. Por outro lado 3 - 8 pois


considerando o conjunto M = {3m, m ∈ N} = {3, 6, 9, 12, . . .} dos
múltiplos positivos de 3 vemos que 8 não pertence ao mesmo.

A seguinte proposição é um bom exercício para entender os con-

ceitos enunciados acima.

Proposição 3.3. Sejam a, b e c números inteiros. Então,


(a) se a | b e b | c então a | c;

(b) se a | b e a | c então a | (b + c) e a | (b − c);

(c) se a e b são positivos e a | b então 0 < a ≤ b;

(d) se a | b e b | a então a = b ou a = −b.

Demonstração. Se a|b e b|c então existem inteiros q1 e q2 tais que

b = aq1 (3.1)

c = bq2 . (3.2)
92 3 Divisibilidade

Substituindo (3.1) em (3.2) temos que

c = aq1 q2 = aq, onde q = q1 q2 ∈ Z, (3.3)

provando isto a armação feita em (a).

Agora provaremos (b). Com efeito, se a | b e a | c valem as

igualdades

b = aq1 , q1 ∈ Z (3.4)

c = aq2 , q2 ∈ Z. (3.5)

Operando com os ambos lados das igualdades (3.4) e (3.5) temos que

b + c = a(q1 + q2 ) e b − c = a(q1 − q2 ),
| {z } | {z }
r∈Z s∈Z

obtendo assim o resultado desejado.

Continuamos agora com a prova de (c). De fato, se a | b, sendo

ambos positivos, então b = aq com

q ≥ 1. (3.6)

Logo, multiplicando por a ambos lados de (3.6) temos (como a é posi-


tivo) que

b = aq ≥ a > 0,

como esperávamos.

Finalmente, provaremos (d). Com este propósito observamos que

se a|b e b|a |a| divide |b| e |b| divide |a|. Portanto, pelo item
então

(c) temos que |a| ≤ |b| e |b| ≤ |a|, ou seja, |a| ≤ |b| ≤ |a|. Logo,

|a| = |b| e consequentemente a = b ou a = −b.


3.1 Conceitos Fundamentais e Divisão Euclidiana 93

Exemplo 3.4. Prove que o número N = 545362 − 7 não é divisível por


5.

Solução. Vamos mostrar isso utilizando o método do absurdo. Se


45362
este número fosse divisível por 5, então 5 − 7 = 5q . Logo, 7 =
45362
5 − 5q , ou seja, 7 seria divisível por 5, o que é um absurdo.

O próximo passo de nossa discussão é ver o que acontece quando

um número não é divisível por outro. Por exemplo, analisemos se 31 é

divisível por 7 e para isto listaremos a diferença entre 31 e os múltiplos

positivos de 7, isto é:

r1 = 31 − 7 · 1 = 24,
r2 = 31 − 7 · 2 = 17,
r3 = 31 − 7 · 3 = 10,
r4 = 31 − 7 · 4 = 3,
r5 = 31 − 7 · 5 = −4,
r6 = 31 − 7 · 6 = −11,
.
.
.

Claramente 31 não é divisível por 7, pois caso contrário teríamos

que alguma das diferenças acima seria igual a zero, o que é impossível

pois as diferenças rq = 31 − 7q com 1 ≤ q ≤ 4 são todas positivas

e com q ≥5 são todas negativas. Entretanto, notamos que entre as

diferenças positivas a única que é menor que 7 corresponde ao caso

q = 4. O resultado seguinte nos diz o que acontece no caso geral da

divisão de um inteiro b por um inteiro positivo a.


94 3 Divisibilidade

Teorema 3.5 (Divisão Euclidiana) . Dados dois inteiros a e b, sendo


a positivo, existem únicos inteiros q e r tais que

b = aq + r, 0 ≤ r < a.

Se a - b, então r satisfaz a desigualdade estrita 0 < r < a.


Demonstração. Por simplicidade, suporemos que b é positivo. Se b<
a, basta tomar q=0 e r = b.
b = a, então tomamos q = 1 e
Se r = 0.
Assim, assumiremos também que b > a > 0. Consideremos o conjunto

R = {b − aq ∈ Z; b − aq ≥ 0} ⊆ N ∪ {0} (3.7)

Notemos que o conjunto R é não vazio, pois b − a ∈ R, já que

b − a > 0. Deste modo, pelo princípio da boa ordenação temos que


R admite um menor elemento, que denotaremos por r. Claramente
r = b − aq ≥ 0, para algum q ≥ 0. Além disso, r < a pois caso
contrário

r = b − aq ≥ a ⇒ b − a(q + 1) ≥ 0. (3.8)

Por outro lado,

a > 0 ⇒ b − a(q + 1) < b − aq. (3.9)

Das desigualdades (3.8) e (3.9) segue que

0 ≤ b − a(q + 1) < b − aq,

contradizendo o fato de que r = b−aq é o menor elemento não negativo


de R.
Agora provaremos que de fato r e q, escolhidos desta forma, são

únicos. Com efeito, suponhamos que existem outros inteiros r1 e q1


tais que

b = aq1 + r1 , 0 ≤ r1 < a.
3.1 Conceitos Fundamentais e Divisão Euclidiana 95

Então resulta que aq + r = aq1 + r1 . Logo,

(r − r1 ) = (q1 − q)a; (3.10)

sendo assim, r−r1 é múltiplo de a. Mas, em virtude de −a < r−r1 < a,


o único valor que r − r1 pode tomar, sendo este múltiplo de a, é

r − r1 = 0. Portanto, r = r1 , de onde se deduz diretamente de (3.10)


que q = q1 .

Os números q e r no enunciado do teorema acima são chamados,

respectivamente, de quociente e resto da divisão de b por a.


Um resultado imediato da divisão euclidiana é o seguinte.

Corolário 3.6. Dados dois números naturais a e b com 1 < a ≤ b,


existe um número natural n tal que

na ≤ b < (n + 1)a.

Demonstração. Pela divisão euclidiana, existem únicos q, r ∈ N com

0≤r<a tais que b = aq + r. Assim

aq ≤ b = aq + r < aq + a = a(q + 1).

Basta agora tomar q=n para obtermos o resultado.

Os exemplos a seguir apresentam a utilidade do Teorema 3.5.

Exemplo 3.7. Se a é um natural com a ≥ 3, então a2 deixa resto 1


na divisão por a − 1. Consequentemente, a − 1 divide a2 − 1.

Solução. Usando a identidade a2 − 1 = (a − 1)(a + 1) temos que


a2 = (a − 1)(a + 1) + 1 com 1 < a − 1, de onde segue o resultado.
96 3 Divisibilidade

O próximo exemplo, como veremos, motiva a procura de cami-

nhos ecientes para encontrar o resto que deixa um número quando é

dividido por outro.

Exemplo 3.8. Um turista brasileiro chega a Cuba e troca parte de


seu dinheiro na casa de câmbio, recebendo 175 notas de 50 pesos e
213 notas de 20 pesos. Ele decide trocar este dinheiro pela maior
quantidade possível das famosas moedas de 3 pesos cubanos, porque
elas têm gravada a imagem do guerrilheiro Che Guevara. Quanto
sobrou do dinheiro depois de fazer a troca pelas moedas?

Solução. Para resolver este problema basta achar o resto que deixa o

número n = 175 · 50 + 213 · 20 quando é dividido por 3. Entretanto,

queremos destacar que não é preciso fazer os produtos e a soma envol-

vidos no número n. Em lugar de fazer isto substituímos cada número

que aparece em n pelo resto que este deixa na divisão por 3, formando
assim um novo número n1 , ou seja,

n1 = 1 · 2 + 0 · 2 = 2.

Agora procuramos o resto que n1 deixa na divisão por 3, que obvi-

amente é 2. A surpresa é que este resto é o mesmo que deixa n na

divisão por 3. Logo, sobraram 2 pesos depois de fazer a troca.

A solução do exemplo anterior é uma aplicação particular do se-

guinte lema que é de muita utilidade na resolução de problemas.

Lema 3.9 (Lema dos Restos). A soma e o produto de quaisquer dois


números naturais deixa o mesmo resto que a soma e o produto dos
seus restos, na divisão por um inteiro positivo a.
3.1 Conceitos Fundamentais e Divisão Euclidiana 97

Demonstração. Sejam n1 , n2 ∈ Z. Fazendo a divisão com resto de

ambos os números por a temos que

n1 = aq1 + r1 e n2 = aq2 + r2 ,

com 0 ≤ r1 , r2 < a. Então,

n1 n2 = (aq1 + r1 )(aq2 + r2 )
= a2 q1 q2 + aq1 r2 + aq2 r1 + r1 r2
(3.11)
= a(aq1 q2 + q1 r2 + q2 r1 ) + r1 r2
= aq + r1 r2 ,

onde q = aq1 q2 +q1 r2 +q2 r1 . Agora dividimos r1 r2 por a para obtermos

r1 r2 = ap + r, p ∈ Z, 0 ≤ r < a. (3.12)

Das igualdades (3.11) e (3.12) segue que

n1 n2 = aq + ap + r = a(p + q) + r, 0 ≤ r < a. (3.13)

Portanto, de (3.12) e (3.13) concluímos que os restos que deixam n1 n2


e r1 r2 na divisão por a são iguais, cando provado o resultado para o

produto. A prova para a soma é análoga.

Observação 3.10. A vantagem do lema é que em certos problemas


que envolvem números muito grandes podemos substituir estes por nú-
meros muito menores e mais confortáveis para trabalhar.
Vejamos como aplicar o lema dos restos nos seguintes exemplos a

seguir.

Exemplo 3.11. Prove que o produto de dois números naturais con-


secutivos é sempre divisível por 2.
98 3 Divisibilidade

Solução. Se n ∈ N temos que provar que an = n(n + 1) é divisível por

2. Quando fazemos a divisão de n por 2 temos duas possibilidades

para o resto: r=0 ou r = 1. Analisemos os dois casos por separado.

• [r = 0] Neste caso o resto que deixa an na divisão por 2 é o

mesmo que o resto que deixa 0(0+1)=0, logo an é divisível por

2.

• [r = 1] Neste caso podemos substituir an por 1(1+1)=2 e o

resto que este último deixa quando é dividido por 2 é 0, logo an


também é divisível por 2.

Mostraremos agora como utilizar o exemplo anterior pra resolver

um dos problemas da 1a Olimpíada Brasileira de Matemática.

Exemplo 3.12. Prove que se n é ímpar, então n2 − 1 é múltiplo de


8.
Solução. Como n é ímpar, podemos escrever n = 2m + 1, para algum

k ∈ Z. Logo

n2 − 1 = (2m + 1)2 − 1 = 4m2 + 4m + 1 − 1 = 4m2 + 4m.

Assim,

n2 − 1 = 4m(m + 1).
Observe que de acordo com o exemplo 3.11, m(m + 1) é múltiplo de

2. Portanto, m(m + 1) = 2q para algum q ∈ Z, de aonde

n2 − 1 = 4m(m + 1) = 4 · 2q = 8q,

como queríamos demonstrar.


3.2 Bases Numéricas 99

Exemplo 3.13. Prove que em qualquer triângulo retângulo com lados


inteiros, pelo menos um deles é múltiplo de 3.

Solução. Comecemos analisando quais são os restos possíveis para a

divisão por 3 de um número que é quadrado. De acordo com o lema

dos restos temos a seguinte tabela para os restos de n e n2 , na divisão

por 3:

n n2
0 0

1 1

2 1

Resumindo, se um número não é múltiplo de 3 então o resto da divisão

de seu quadrado por 3 deve ser igual a 1.

Agora denotemos por a e b os catetos e por c a hipotenusa. Supo-

nhamos que nenhum deles é divisível por 3. Então a2 e b2 deixam resto


1 na divisão por 3. Logo, a2 + b 2 deixa resto 12 + 12 = 2 na divisão
por 3; mas isto é uma contradição pois, pelo Teorema de Pitágoras,

a2 + b 2 = c 2 e c2 deixa resto 1 quando é dividido por 3.

3.2 Bases Numéricas

Começamos esta seção com uma brincadeira interessante.

João, ao sair da aula de matemática do professor Peitágoras, en-

controu Pedro e lhe propôs a seguinte brincadeira:

 Pense numa peça de dominó, Pedro. Vou adivinhar que peça é

essa usando uma fórmula mágica.

 Ok, João. Pode começar, já pensei.


100 3 Divisibilidade

x y

Figura 3.1: Peça de Dominó

- Escolha um dos números na peça e multiplique por 5. Depois

disso some três a esse resultado. Multiplique agora o número que você

obteve por dois. Some isto com o outro número da peça. Qual foi o

resultado?

 Foi 40.

 Então a peça que você escolheu foi a 3 com 4!

 Como você acertou? Me ensina!

Claro que de mágico João não tinha nada e decidiu contar seu

segredo a Pedro.

O jogo funciona assim: cada parte da peça de dominó pode ser

considerada como um dos dígitos de um número de 2 algarismos, o qual

denotamos por n = xy = 10x + y (veja a Figura 3.1). Acompanhando

os passos de João, temos que:

(5x + 3)2 + y = 40 ⇔ 10x + y = 34, (3.14)

que claramente, tem por soluções: x=3 e y = 4, usando a represen-

tação de 34 na base decimal.

No sistema de numeração decimal, também conhecido como sis-

tema numérico na base 10, todo número pode ser representado como

uma sequência de 10 símbolos, constituídos pelo 0 (zero) e os alga-

rismos 1, 2, 3, . . . , 9. Por exemplo, 345 escreve-se na base decimal da


3.2 Bases Numéricas 101

seguinte forma

345 = 300 + 40 + 5 = 3 · 102 + 4 · 10 + 5,

assim como 2768 se escreve da forma

2768 = 2000 + 700 + 60 + 8 = 2 · 103 + 7 · 102 + 6 · 10 + 8.

De modo geral, se denotamos por a = an an−1 . . . a1 a0 o número inteiro


positivo formado pelos algarismos an , an−1 , . . . , a1 e a0 , nessa ordem,

então a se escreve na base decimal da forma

a = an 10n + an−1 10n−1 + . . . + a1 10 + a0 (3.15)

Antes de provar alguns dos critérios de divisibilidade mais po-

pulares do sistema de numeração decimal, provamos uma identidade

muito útil.

Lema 3.14. Sejam a, b, n ∈ N. Temos que

an − bn = (a − b)(an−1 + an−2 b + · · · + abn−2 + bn−1 ).

Consequentemente, se 0 < b < a, então a − b divide an − bn .

Demonstração. Primeiro provaremos que a propriedade vale para b=


1. Com efeito, considerando a soma geométrica

s = 1 + a + a2 + · · · + an−1

e multiplicando s por a temos que

as = (a + a2 + · · · + an−1 ) + an = s − 1 + an .
102 3 Divisibilidade

Assim, (a − 1)s = as − s = an − 1, de onde se segue que

an − 1 = (a − 1)(an−1 + an−2 + · · · + a + 1). (3.16)

Daí temos a validade para b = 1.


an − bn = bn ( ab )n − 1 .
 
Para b∈N qualquer, observe que Usando

esta expressão e (3.16) tem-se

an − bn = bn ( ab − 1) ( ab )n−1 + ( ab )n−2 + · · · + ( ab ) + 1
 

= (a − b)bn−1 ( ab )n−1 + ( ab )n−2 + · · · + ( ab ) + 1


 
(3.17)

= (a − b)(an−1 + an−2 b + · · · + abn−2 + bn−1 ),


obtendo-se a igualdade clamada.

Proposição 3.15 (Critérios de Divisibilidade) . Seja a = an . . . a1 a0


um inteiro positivo, então
(a) a é divisível por 10 se, e somente se, a0 for igual a 0;
(b) a é divisível por 3 ou por 9 se, e somente se, a soma dos seus
dígitos é divisível por 3 ou por 9, respectivamente;
(c) a é divisível por 5 se, e somente se, a0 for igual a 0 ou 5.
Demonstração. Utilizando a representação decimal (3.15) temos que

a = 10(an 10n−1 + an−1 10n−2 + · · · + a1 ) + a0 .

Então, pela Proposição 3.3-(b) tem-se que 10 | a se, e somente se,

10 | a0 , prondose-se assim o critério (a).

Para provar (b) observemos que

a = an 10n + an−1 10n−1 + · · · + 10a1 + a0


= an (10n − 1) + an−1 (10n−1 − 1) + · · · + (10 − 1)a1 (3.18)

+ an + an−1 + · · · + a1 + a0 .
3.2 Bases Numéricas 103

Pelo Lema 3.14 temos que 10j − 1 = 9qj para todo 1 ≤ j ≤ n, daí

segue-se

a = 9(an qn + an−1 qn−1 + · · · + a1 ) + an + an−1 + · · · + a1 + a0 .

Então, aplicando novamente o item (b) da Proposição 3.3 temos que

9|a se, e somente se, 9 | (an + an−1 + · · · + a1 + a0 ).

A prova para o caso da divisibilidade por 3 segue de maneira idêntica,

logo ca provado o item (b).

A prova do item (c) segue de maneira muito semelhante e deixamos

a mesma a cargo do leitor.

Exemplo 3.16. Prove sem fazer muitas contas que o número

N = 13424136 + 1234567890

é divisível por 3.

Solução. Note que não precisamos fazer a soma dos números ante-

riores. Para mostrar isso, basta aplicar o item (b) da Proposição 3.3 e

o item (b) da Proposição 3.15, observando que cada um dos números

acima é divisível por 3, pois a soma de seus dígitos é um múltiplo de

3.

Finalizamos esta seção com uma aplicação da divisão euclidiana

que nos mostra que, analogamente à representação decimal, qualquer

número admite uma representação única em qualquer outra base nu-

mérica.
104 3 Divisibilidade

Teorema 3.17 (Bases Numéricas) . Dados a, b ∈ N, com b > 1, exis-


tem únicos números naturais r0 , r1 , . . . , rn tais que 0 ≤ ri ≤ b − 1,
0 ≤ i ≤ n, e satisfazendo

a = rn bn + rn−1 bn−1 + · · · + r1 b + r0 .

A representação acima é dita representação de a na base b e usaremos


a notação

a = (rn cn−1 . . . r1 r0 )b ,

para fazer referência a esta.

Demonstração. Apliquemos sucessivamente a divisão euclidiana como

segue:

a = bq0 + r0 , r0 < b,
q0 = bq1 + r1 , r1 < b,
q1 = bq2 + r2 , r2 < b,
. . . .
. . . .
. . . .

qj−1 = bqj + rj , rj < b,

e assim por diante. Como a > q0 > q1 > q2 > · · · > qj−1 , para algum
j = n deveremos ter que qn−1 < b. Logo, qj = 0 para todo j ≥ n,
assim como rj = 0 para todo j ≥ n + 1. Das igualdades acima, para
3.2 Bases Numéricas 105

1 ≤ j ≤ n, tem-se

a = bq0 + r0 ,
bq0 = b2 q1 + br1 ,
b2 q1 = b3 q2 + b2 r2 ,
(3.19)
. . .
. . .
. . .

bn−1 qn−2 = bn qn + bn−1 rn−1


bn qn−1 = bn+1 0 + bn rn .

Efetuando a soma de todas as igualdades em (3.19) obtemos

a = rn bn + rn−1 bn−1 + · · · + r1 b + r0 .

A unicidade dos números ri vem da unicidade dos restos na divisão

euclidiana.

Observação 3.18. O sistema de numeração na base 2 é também co-


nhecido como sistema binário e é o sistema habitualmente utilizado no
funcionamento dos computadores.

Exemplo 3.19. Se deseja pesar qualquer número inteiro de gramas de


ouro, entre 1g e 100g , numa balança de dois pratos, onde os pesos só
podem ser usados no prato esquerdo da balança. Mostre que a escolha
adequada de 7 pesos diferentes é suciente para realizar esta tarefa.

Demonstração. Usando o sistema de numeração em base 2 temos que

qualquer número a tal que 1 ≤ a ≤ 100 pode ser expressado de forma

única como

a = r6 26 + r5 25 + r4 24 + r3 23 + r2 22 + r1 2 + r0 1,
106 3 Divisibilidade

com ri ∈ {0, 1}, 0 ≤ i ≤ 1. Observe que 2n ≥ 128, com n ≥ 7, logo

estas potências não são consideradas. notemos também que o fato de

cada ri ser 0 ou 1 nos diz que não precisamos repetir nenhum dos

pesos na realização de qualquer pesada. Logo, os pesos

1, 22 , 23 , 24 , 25 , 26

são sucientes para realizar as pesadas de gramas de ouro entre 1g e

100g .

3.3 Máximo Divisor Comum e Mínimo Múl-

tiplo Comum

Nesta seção estudaremos dois conceitos fundamentais, que aparecem

naturalmente em vários problemas de divisibilidade, assim como a

relação existente entre eles.

3.3.1 Máximo Divisor Comum

O primeiro destes conceitos está relacionado com os inteiros positivos

que dividem simultaneamente a dois inteiros prexados e é denomi-

nado máximo divisor comum.


Daqui por diante só consideraremos os divisores positivos dos nú-

meros.

Denição 3.20 (Máximo Divisor Comum). Sejam a e b inteiros dife-


rentes de zero. O máximo divisor comum, resumidamente mdc, entre
a e b é o número d que satisfaz as seguintes condições:

(a) d é um divisor comum de a e b, isto é, d | a e d | b;


3.3 Máximo Divisor Comum e Mínimo Múltiplo Comum 107

(b) d é o maior inteiro positivo com a propriedade (a).


Neste caso, denotamos o mdc entre a e b por d = mdc(a, b) ou por
d = (a, b). Se (a, b) = 1, então dizemos que a e b são primos entre si.

Exemplo 3.21. Observando que 12 = 6 · 2, 18 = 6 · 3 temos que


mdc.(12, 18) = 6. Por outro lado, mdc.(4, 15) = 1, logo os números 4
e 15 são primos entre si.
Vejamos agora algumas das propriedades mais importantes dos

divisores comuns de dois inteiros.

Proposição 3.22. Sejam a e b dois inteiros. Então valem as seguintes


armações.
(a) Se a é múltiplo de b, então (a, b) = b.

(b) Se a = bq + c, c 6= 0, então o conjunto dos divisores comuns dos


números a e b coincide com o conjunto dos divisores comuns dos
números b e c. Particularmente, (a, b) = (b, c).
Demonstração. Começamos com a prova de (a). Com efeito, todo

divisor comum dos números a e b é um divisor de b. Reciprocamente,


usando que a é múltiplo de b, todo divisor de b é também um divisor

de a, ou seja, um divisor comum dos números a e b. Portanto, o

conjunto dos divisores comuns dos números a e b é igual ao conjunto

dos divisores de b. Como o maior divisor de b é ele mesmo, resulta que

(a, b) = b.
Vejamos (b). Usando o item (b) da Proposição 3.3 temos que

todo divisor comum de a e b também divide c e, consequentemente, é

um divisor deb e c. Pela mesma razão todo divisor comum de b e c


também divide a e, consequentemente, é um divisor de a e b. Portanto
108 3 Divisibilidade

os divisores comuns de a e b são os mesmos que os divisores comuns

de b e c. Particularmente, também coincidem os maiores divisores

comuns, ou seja, (a, b) = (b, c).

O teorema a seguir é uma das ferramentas básicas na resolução

de problemas que envolvem o mdc entre dois números. O resultado

foi provado pela primeira vez por Claude-Gaspard Bachet de Méziriac

(1581-1638) e mais tarde generalizado para polinômios por Étienne

Bézout (1730-1783). Frequentemente, na literatura se enuncia este

resultado como teorema (ou identidade) de Bézout, esquecendo-se o

nome de Bachet.

Teorema 3.23 (Teorema de Bachet-Bézout). Se d é o mdc de a e b,


então existem números inteiros x0 e y0 tais que d = (a, b) = ax0 + by0 .

Demonstração. Considere a combinação linear ax + by , onde x e y


percorrem todos os inteiros. Este conjunto de inteiros, denotado por

Ca,b = {ax + by; x, y ∈ Z},

inclui valores positivos e negativos. Além disso, escolhendo x = y = 0,


vemos que Ca,b também contém o zero.

Pelo princípio da boa ordenação, podemos escolher x0 e y0 tais que

λ = ax0 +by0 seja o menor número inteiro positivo contido no conjunto


Ca,b .
Agora mostraremos que λ|a e λ | b. Provaremos que λ|a e o

outro segue analogamente. Usaremos para este propósito o método de

redução ao absurdo, ou seja, vamos supor que λ-a e obteremos uma

contradição.
3.3 Máximo Divisor Comum e Mínimo Múltiplo Comum 109

Usando a divisão euclidiana, de λ - a segue que existem inteiros q


e r tais que a = λq + r com 0 < r < λ. Portanto,

r = a − λq = a − q(ax0 + by0 ) = a(1 − qx0 ) + b(−qy0 )

e assim r está no conjunto Ca,b , o que contradiz a hipótese de λ ser o

menor elemento positivo contido em Ca,b .


Uma vez que λ divide a e b só resta provar que λ = d. Com efeito,

desde que d = (a, b), podemos escrever a = da1 , b = db1 e

λ = ax0 + by0 = d(a1 x0 + b1 y0 ).

Assim d | λ. Logo pela parte (c) da Proposição 3.3, concluímos que

d ≤ λ. Agora d < λ é impossível pois d = mdc(a, b), e portanto

d = λ = ax0 + by0 .

A seguinte proposição resume algumas consequências importantes

da demonstração dada ao teorema de Bézout.

Proposição 3.24. Sejam d, λ ∈ N e a, b, c ∈ Z. Então valem as


seguintes armações:

(a) Se d | a e d | b, então d | (a, b).

(b) O mdc.(a, b) é o menor valor positivo de ax + by , onde x e y


percorrem todos os números inteiros.

(c) (λa, λb) = λ(a, b).

(d) Se d | a e d | b, então ( ad , db ) = d1 (a, b). Consequentemente,


 
a b
, = 1.
(a, b) (a, b)
110 3 Divisibilidade

(e) Se (a, c) = (b, c) = 1, então (ab, c) = 1.

(f) Se c | ab e (b, c) = 1, então c | a.

Demonstração. A prova de (a) é consequência imediata da igualdade

(a, b) = ax0 + by0 anunciada no teorema de Bézout; assim como (b)

segue diretamente da demonstração dada a este teorema.

Para provar (c), primeiro observamos que

(λa)x + (λb)y = λ(ax + by) onde x, y ∈ Z.

Usando o item (a) e o fato de λ ser positivo, da igualdade acima segue


que


(λa, λb) = min (λa)x + (λb)y > 0; x, y ∈ Z

= λ min ax + by ; x, y ∈ Z
= λ(a, b).

A armação feita em (d) segue diretamente de (c), observando que

   
a b a b
(a, b) = d , d =d , .
d d d d

Continuamos com a prova de (e). De (a, c) = (b, c) = 1, temos que

existem inteiros xj e yj , j = 1, 2, tais que

ax1 + cy1 = 1,
bx2 + cy2 = 1.

Multiplicando lado a lado as igualdades obtemos

(x1 x2 )ab + (ax1 y2 + y1 bx2 + cy1 y2 )c = 1.


|{z} | {z }
x y
3.3 Máximo Divisor Comum e Mínimo Múltiplo Comum 111

Então, usando o item (b) e a igualdade acima resulta que (ab, c) = 1.


Finalmente, provaremos (f ). Das hipóteses temos que existem in-

teiros x0 e y0 tais que

bx0 + cy0 = 1.

Multiplicamos a igualdade acima por a em ambos lados para obtermos

abx0 + acy0 = a.

Por outro lado, ab = cq para algum inteiro q. Usando esta condição

na última igualdade temos que

cqx0 + acy0 = c(qx0 + ay0 ) = a,

logo c | a.

3.3.2 Algoritmo de Euclides

Apesar de conhecermos propriedades teóricas do mdc entre dois intei-

ros, encontrá-lo de fato pode ser uma tarefa complicada, sem auxílio

das ferramentas corretas. Lembrando o seu signicado, o leitor talvez

pudesse pensar que devemos calcular todos os divisores de a, todos

os divisores de b e descobrir qual é o maior elemento comum aos dois

conjuntos.

Para achar o mdc se faz uso de um importante método denominado

algoritmo de Euclides .

Teorema 3.25 (Algoritmo de Euclides). Dados dois inteiros positivos,


a e b, aplicamos sucessivamente a divisão euclidiana para obter a se-
112 3 Divisibilidade

guinte sequência de igualdades





 b = aq1 + r1 , 0 ≤ r1 < a,





 a = r1 q2 + r2 , 0 ≤ r2 < r1 ,

r = r q + r , 0 ≤ r3 < r2 ,

1 2 3 3
(3.20)


 ··· ··· ··· ··· ···


rn−2 = rn−1 qn + rn , 0 ≤ rn < rn−1 ,






r
n−1 = rn qn+1 ,

até algum rn dividir rn−1 . Assim, o mdc.(a, b) = rn , ou seja, é o


último resto não-nulo no processo de divisão anterior.

Observação 3.26. Quando lidamos com números pequenos achar o


mdc é uma tarefa fácil pois podemos calcular o mdc valendo-nos das
fatorações dos números envolvidos. No entanto, quando estamos tra-
balhando com números grandes o algoritmo de Euclides, em geral, é
mais fácil que a fatoração, podendo ser esta última bem difícil.

Demonstração do algoritmo de Euclides. Começamos observando que

o processo de divisão (3.20) é nito. Com efeito, a sequência de núme-

rk é estritamente decrescente e está contida no conjunto


ros inteiros

{r ∈ Z, 0 ≤ r < a}, portanto não pode conter mais do que a intei-


ros positivos. Examinando as igualdades (3.20) de cima para baixo e

usando a Proposição 3.22 temos que

(a, b) = (a, r1 ) = (r1 , r2 ) = · · · = (rn−1 , rn ) = rn .


3.3 Máximo Divisor Comum e Mínimo Múltiplo Comum 113

Observação 3.27. Notemos que o teorema de Bézout também pode


ser obtido como consequência do processo de divisão (3.20). Com
efeito, podemos escrever

rn = rn−2 − rn−1 qn o
⇒ rn = rn−2 − (rn−3 − rn−2 qn−1 )qn .
rn−1 = rn−3 − rn−2 qn−1

Logo, conseguimos escrever rn em termos de rn−2 e rn−3 . Utilizando a


expressão rn−2 = rn−4 − rn−3 qn−2 podemos escrever rn como combina-
ção de rn−3 e rn−4 . Repetindo este processo várias vezes, concluímos
que existem x, y ∈ Z tais que

d = rn = xr1 + yr2 .

Ora, como r1 = b − aq1 e r2 = a − r1 q2 = a(1 + q1 q2 ) − bq2 , então,


substituindo estes valores na última igualdade obtemos o Teorema de
Bézout.

Exemplo 3.28. Achar o máximo divisor comum dos números 471 e


1.176.

Solução. Aplicando o algoritmo de Euclides obtemos a seguinte sequên-

cia de divisões com resto:

1176 = 471 · 2 + 234,


471 = 234 · 2 + 3,
234 = 78 · 3,

então o mdc(471, 1176) = 3.


2n + 8
Exemplo 3.29. Provar que a fração é irredutível para todo
4n + 15
número natural n.
114 3 Divisibilidade

Solução. Usando o algoritmo de Euclides temos que

4n + 15 = (2n + 8) · 1 + 2n + 7,
2n + 8 = (2n + 7) · 1 + 1,
2n + 7 = (2n + 7) · 1.

Então o mdc.(4n + 15, 2n + 8) = 1 e portanto 4n + 15 e 2n + 8 são

primos entre si para qualquer valor de n.

Exemplo 3.30. Achar o mdc.(111


| .{z
. . 111}, 11
| .{z
. . 11})
100 vezes 60 vezes

Solução. Primeiro escrevemos os números na base decimal, isto é,

111 . . 111} = 1099 + 1098 + · · · + 1,


| .{z
100 vezes

11 . . 11} = 1059 + 1058 + · · · + 1.


| .{z
60 vezes

Aplicamos agora o algoritmo de Euclides para obter as seguintes igual-

dades

111 . . 111} = (1059 + 1058 + · · · + 1)1040 + 1039 + 1038 + · · · + 1,


| .{z
100 vezes

1059 + 1058 + · · · + 1 = (1039 + 1038 + · · · + 1)1020 +


+ 1019 + 1018 + · · · + 1,
1039 + 1038 + · · · + 1 = (1019 + 1018 + · · · + 1)1020 +
+ 1019 + 1018 + · · · + 1.

Disso resulta que

19 18
| {z } | {z } = 10 + 10 + · · · + 1 = |11 .{z
mdc.(111 . . . 111, 11 . . . 11) . . 11} .
100 vezes 60 vezes 20 vezes
3.3 Máximo Divisor Comum e Mínimo Múltiplo Comum 115

3.3.3 Mínimo Múltiplo Comum

Agora passamos ao segundo conceito importante desta seção. O mesmo

está relacionado com os inteiros positivos que são simultaneamente

múltiplos de dois inteiros prexados e é denominado mínimo múltiplo


comum.

Denição 3.31 (Mínimo Múltiplo Comum) . Sejam a e b inteiros


diferentes de zero. O mínimo múltiplo comum, resumidamente mmc,
entre a e b é o inteiro positivo m que satisfaz as seguintes condições:

(a) m é um múltiplo comum de a e b, isto é, a | m e b | m;

(b) m é o menor inteiro positivo com a propriedade (a).

Neste caso, denotamos o mmc entre a e b por m = mmc(a, b) ou por


m = [a, b].

Resumimos a seguir algumas das propriedades fundamentais do

mmc de dois inteiros.

Proposição 3.32. Sejam a, b, c ∈ Z e λ ∈ Z. Então valem as se-


guintes armações:

(a) se c é múltiplo comum de a e b, então [a, b] | c;

(b) [λa, λb] = λ[a, b];

(c) |ab| = [a, b] · (a, b).

Demonstração. Começamos com a prova de (a). A divisão com resto

de c por [a, b] nos dá

c = [a, b]q + r, 0 ≤ r < [a, b]. (3.21)


116 3 Divisibilidade

Da igualdade anterior, basta provar que r = 0 para obter o resultado

desejado. Suponhamos, pelo contrário, que 0 < r < [a, b]. Notemos

que tanto a como b dividem c e [a, b]. Logo, pelo item (b) da Pro-

posição 3.3 e a igualdade (3.21), temos que a e b também dividem r,


ou seja, r é múltiplo comum de a e b e não pode ser menor que [a, b],
contradizendo nossa suposição.

Prosseguimos com a prova de (b). Observemos que λ[a, b] é múlti-

plo comum de λa e λb, logo pelo item (i) vale que

[λa, λb] ≤ λ[a, b]. (3.22)

Por outro lado, [λa, λb] = q1 λa = q2 λb, para alguns inteiros q1 e q2 ;


[λa,λb]
logo, é um múltiplo comum de a e b. Portanto,
λ

[λa, λb]
[a, b] ≤ ⇔ λ[a, b] ≤ [λa, λb]. (3.23)
λ
Das igualdades (3.22) e (3.23) segue que

λ[a, b] ≤ [λa, λb] ≤ λ[a, b],

de onde vem diretamente o resultado.

Para provar (c) podemos supor sem perda de generalidade que a e

b são positivos devido às igualdades

[a, b] = [a, −b] = [−a, b] = [−a, −b].

Dividiremos a prova em dois casos:

Caso 1: (a, b) = 1.
Sabemos que b | [a, b] e [a, b] = qa, para algum q ∈ N. Então b | qa
e além disso (a, b) = 1. Logo, pelo item (v) da Proposição 3.24 temos

que b | q . Portanto, b ≤ q e consequentemente

ab ≤ aq = [a, b]. (3.24)


3.3 Máximo Divisor Comum e Mínimo Múltiplo Comum 117

Entretanto, da denição de [a, b] vale que

[a, b] ≤ ab. (3.25)

Das desigualdades (3.24) e (3.25) segue que ab ≤ [a, b] ≤ ab. Assim,

ab = [a, b] = [a, b] · 1 = [a, b] · (a, b).

Caso 2: (a, b) > 1.  


a
Da parte (c) da Proposição 3.24 sabemos que , b
(a,b) (a,b)
= 1.
Aplicando o caso anterior vale que
   
a b a b a b
· = , · , .
(a, b) (a, b) (a, b) (a, b) (a, b) (a, b)
Multiplicamos esta última igualdade por (a, b)2 e usamos o item (b)

provado anteriormente, assim como a parte (d) da Proposição 3.24

para obter
   
a b a b
ab = (a, b) , (a, b) , = [a, b] · (a, b).
(a, b) (a, b) (a, b) (a, b)

Exemplo 3.33. Dois amigos passeiam de bicicleta, na mesma dire-


ção, em torno a uma pista circular. Para dar uma volta completa um
deles demora 15 minutos e o outro demora 18 minutos. Eles partem
juntos e combinam interromper o passeio quando os dois se encontra-
rem pela primeira vez no ponto de partida. Quantas voltas deu cada
um?
Solução. Denotemos por n1 e n2 , respectivamente, o número de voltas
que dá cada um dos amigos. Notemos que o tempo total da corrida é

o menor valor positivo de T que satisfaz as igualdades

T = 15n1 = 18n2 ,
118 3 Divisibilidade

ou seja
15 · 18
T = [15, 18] = = 90.
3
Portanto, n1 = 6 e n2 = 5.

Finalizamos esta seção com um exemplo que nos fornece uma bela

interpretação geométrica do mínimo múltiplo comum. O mesmo foi

proposto na Olimpíada Brasileira de Matemática.

Exemplo 3.34. Um retângulo de lados inteiros AB = m e CD = n,


é dividido em quadrados de lado 1. Em cada um dos vértices ele possui
um pequeno orifício. Um raio de luz entra no retângulo por um dos
vértices, na direção da bissetriz do ângulo reto, e é reetido sucessi-
vamente nos lados do retângulo. Quantos quadrados são atravessados
pelo raio de luz?

D C

A B

Figura 3.2: Interpretação geométrica do mmc

Solução. Se zermos alguns testes preliminares dando valores a me


n, veremos que em cada caso a resposta coincidirá com o mmc(m,n).

Provemos que isto de fato vale para m e n quaisquer. Para realizar a

prova nos auxiliaremos da Figura 3.2.


3.3 Máximo Divisor Comum e Mínimo Múltiplo Comum 119

Primeiramente, notemos que cada vez que o raio de luz atravessa

um quadrado ele avança uma unidade tanto na direção horizontal como

na direção vertical. Usando este fato fazemos as observações a seguir.

• Se o raio entra pelo vértice A, terá que atravessar m quadrados

até chegar ao ladoBC , imediatamente mais m para chegar ao


lado AD , depois mais m para chegar novamente ao lado BC , e

assim sucessivamente. Além disso, depois do raio percorrer pm

quadrados, com p ∈ N, estará batendo no lado BC ou no lado

AD.

• Analogamente o raio baterá no lado AB ou no lado DC se, e

somente se, atravessar qn quadrados, com q ∈ N.

• Somente nos vértices B, C e D do retângulo pode acontecer que

o raio incidente saia do retângulo, terminando assim o processo

de reexão.

Usando as observações acima é fácil ver que o raio chegará a um

vértice quando chegar simultaneamente a dois lados perpendiculares

do retângulo. Portanto, deve ter atravessado um número x de quadra-


dos tal que x = pm = qn, ou seja, x deverá ser um múltiplo comum

de m
n. É claro que a primeira vez que o raio chega a um vértice
e o

número x é o menor múltiplo comum de m e n, isto é, x = [m, n].

Finalmente, observamos que nenhum dos quadrados é atravessado

duas vezes no percurso do raio de A até bater no primeiro vértice, pois


como vemos na gura numa das direções os quadrados atravessados

serão todos cinzas e na outra direção, serão todos brancos.


120 3 Divisibilidade

3.3.4 Equações Diofantinas Lineares

Consideremos a equação

ax + by = c, (3.26)

onde a, b, c ∈ Z, com a 6= 0 e b 6= 0.
A equação (3.26) é chamada de equação diofantina linear e uma

solução desta é qualquer par de inteiros (x, y) que satisfaçam (3.26).

É conhecido que todos os pontos do plano, com coordenadas (x, y),


que satisfazem a igualdade (3.26) representam, geometricamente, uma

reta. Logo, as soluções de uma equação diofantina linear são os pontos

de coordenadas inteiras do plano cartesiano, que estão dispostos sobre

a reta que esta representa. Por exemplo, os pontos (−1, −2) e (1, 1)
são soluções da equação diofantina 3x − 2y = 1, veja a Figura 3.3.
3
y
2

1

0

ℓ x
-1

-2

-3
-3 -2 -1 0 1 2

Figura 3.3: A equação da reta ` é 3x − 2y = 1.

Naturalmente nos perguntamos: É sempre possível achar soluções

para uma equação diofantina linear? A resposta é não; o próximo

resultado nos diz quando isto é possível. Além disso, se uma equação

diofantina linear tem uma solução na verdade ela tem uma innidade

de soluções.
3.3 Máximo Divisor Comum e Mínimo Múltiplo Comum 121

Proposição 3.35. A equação diofantina linear


ax + by = c, a, b, c ∈ Z, com a 6= 0 e b 6= 0, (3.27)

tem solução se, e somente se, d | c, onde d = (a, b). Além disso,
se (x0 , y0 ) é uma solução, então o conjunto de soluções de (3.27) é
constituído por todos os pares de inteiros (x, y) da forma:
x = x0 + t db e y = y0 − t ad , t ∈ Z. (3.28)

Demonstração. (x0 , y0 ) é uma solução


Primeiramente suponhamos que

de (3.27), logo ax0 + by0 = c. Usando que d = (a, b) sabemos que

existem inteiros q1 e q2 , tais que dq1 = a e dq2 = b. Portanto, se

verica a igualdade

dq1 x0 + dq2 y0 = d(q1 x0 + q2 y0 ) = c,

de onde segue obviamente que d | c.


Reciprocamente, suponhamos que d|c e portanto c = qd com q
inteiro. O teorema de Bézout nos garante a existência de dois inteiros,

x0 e y0 , ax0 + by0 = d. Multiplicando


tais que ambos os lados desta

última igualdade por q temos que

ax0 q + by0 q = c,

logo o par (x1 , y1 ), com x1 = x 0 q e y1 = y0 q , é solução da equação

diofantina.

Resta provar agora que temos innitas soluções da forma (3.28).

Com efeito, sendo (x, y) uma outra solução qualquer além de (x0 , y0 ),
vale que ax0 + by0 = c = ax + by , de onde ax0 + by0 = ax + by . Desta

igualdade obtemos a(x − x0 ) = b(y0 − y) e dividimos esta última por

d para obtermos
a b
(x − x0 ) = (y0 − y).
d d
122 3 Divisibilidade

Como ( ad , db ) = 1, então temos que


a
d
| (y0 − y) e
b
d
| (x − x0 ). Logo,

existe inteiro t tal que

x = x0 + t db e y = y0 − t ad .

Por outro lado, é fácil vericar que para qualquer inteiro t as expressões
achadas acima para x e y resolvem a equação diofantina.

A seguir damos um exemplo de como proceder para resolver equa-

ções diofantinas.

Exemplo 3.36. Achar todas as soluções inteiras da equação


12x + 33y = 27.

Solução. Observemos que (12, 33) = 3 e que 3 | 27, logo a equa-

ção tem innitas soluções. Como sabemos, basta achar uma delas

e teremos as restantes. Para achar esta solução particular podemos

trabalhar de duas maneiras, que descrevemos a seguir:

Alternativa 1: reduzimos a equação à forma equivalente

4x + 11y = 9,

e por tentativa e erro vemos que x0 = 5 e y0 = −1 solucionam a

mesma. Então pela Proposição 3.35 temos que

x = 5 + 11t e y = 4t − 1, t ∈ Z,

esgotam todas as soluções que procuramos.

Alternativa 2: aplicamos o algoritmo de Euclides para achar o

mdc (12, 33), obtendo os seguintes resultados:

33 = 12 · 2 + 9,
12 = 9 · 1 + 3,
9 = 3 · 3 + 0.
3.4 Números Primos e Compostos 123

Da segunda e primeira igualdades temos, respectivamente, que

3 = 12 − 9 · 1 e 9 = 33 − 12 · 2.

Usando estas duas obtemos

3 = 12 − (33 − 12 · 2) · 1
= 12 − 33 + 12 · 2
= 3 · 12 − 1 · 33,

ou seja, achamos x0 = 3 e y0 = −1, garantidos pelo teorema de


Bézout, que validam 3 = 12x0 +33y0 . Multiplicamos por 9 esta última

igualdade para obter

27 = 12(9x0 ) + 33(9y0 ).

Portanto, x
e0 = 9x0 = 27 e ye0 = 9y0 = −9 resolvem, particularmente,
a equação diofantina. Analogamente, como na alternativa anterior,

podemos escrever a solução geral da forma:

x = 27 + 11s e y = 4s − 9, s ∈ Z.

3.4 Números Primos e Compostos

Ao longo da história da Matemática, os números primos foram pro-

tagonistas de célebres problemas que motivaram o desenvolvimento

de teorias e técnicas pelas mentes mais férteis, como Fermat, Euler e

Gauss. Até hoje muitos desses problemas, simples de enunciar, que

envolvem números primos são desaos intelectuais para toda a huma-

nidade.
124 3 Divisibilidade

Esta seção será dedicada ao estudo de propriedades básicas dos

números primos. Todo número natural n maior do que 1 tem pelo

menos 2 divisores, claramente 1 e n. Isto motiva a seguinte denição.

Denição 3.37 (Números Primos e Compostos). Um inteiro positivo


n ≥ 2 é dito primo se os únicos divisores que ele tem são 1 e ele
próprio; caso contrário, é dito composto.

Observação 3.38. De modo geral o número 1 não é considerado nem


primo nem composto.

Exemplo 3.39. Os números 2, 3, 5, 7, e 11 são primos e os números


10, 15, 35 e 348 são compostos.

Exemplo 3.40. O número n = 220 − 254 é composto.


Solução. Escrevemos n de outra forma, com o objetivo de facilitar

nosso trabalho. Com efeito, observemos que

n = (210 )2 − (252 )2 = 10242 − 6252 ,

logo é composto por ser diferença de quadrados. Além disso,

n = 10242 − 6252 ,
= (1024 − 625)(1024 + 625),
(3.29)
= 399 · 1649,
= 3 · 133 · 1649.

Portanto, podemos concluir que 3 | n.

Proposição 3.41. Seja n > 1 um número inteiro. Então


(a) o menor divisor de n diferente de 1 é um número primo;
3.4 Números Primos e Compostos 125

(b) se n é composto, o seu menor divisor diferente de 1 não é maior



que n. Em outras palavras, se n não possui divisores diferentes

de 1, menores ou igual que n, então n é primo.

Demonstração. Começamos provando (a). Seja p o menor divisor de

n, diferente de 1. Se p fosse composto teria algum divisor q tal que

1 < q < p; mas


q|p e p | n,
o que nos diz que q | n, e isto contradiz a hipótese levantada sobre p.
Para provar (b) denotamos por p o menor divisor de n, diferente

de 1. Portanto, n = pq com q ≥ p. Multiplicando ambos lados da

desigualdade por p obtemos

n = pq ≥ p2 ,

e consequentemente vale n ≥ p.

Agora vamos enunciar um dos resultados mais clássicos da Mate-

mática, que garante a existência de innitos números primos. Até

onde se conhece, a demonstração a seguir foi a primeira demonstração

escrita utilizando o método de redução ao absurdo e é devida a Eu-

clides cerca de 300 a.C. Para outras seis provas, incluindo a moderna

prova de Fustenberg, recomendamos os livros [1] e [10].

Teorema 3.42 (Teorema de Euclides). A quantidade de números pri-


mos é innita.

Demonstração. Faremos a prova por redução ao absurdo. Suponha

que existe uma quantidade nita de números primos e denotemos estes

por

p1 , p2 , p3 , . . . , pk .
126 3 Divisibilidade

Consideremos o número

n = p1 p2 p 3 · · · pk + 1

e chamemos de q o seu menor divisor primo. Obviamente q não coin-

cide com nenhum dos números pi , 1 ≤ i ≤ k , pois caso contrário,

como ele divide n, teria que dividir 1, o que é impossível. Logo, te-

mos uma contradição à hipótese de termos uma quantidade nita de

primos.

Os números primos também podem ser caracterizados da seguinte

maneira:

Proposição 3.43. Um inteiro positivo p é primo se, e somente se,


satisfaz a seguinte propriedade:

p | ab =⇒ p | a ou p | b (3.30)

onde a, b ∈ Z.
Demonstração. Primeiramente, suponhamos que p é primo e que p - b,
logo (p, b) = 1. Então, pelo item (f ) da Proposição 3.24 temos que

p | a.
Reciprocamente, suponhamos que, a propriedade 3.30 é válida e

além disso vamos supor, pelo absurdo, que p não é primo. Então,

p = d1 d2 , com 1 < d1 < p, 1 < d2 < p. (3.31)

De (3.30) segue que p | d1 ou p | d2 ; consequentemente

p ≤ d1 , ou p ≤ d2 , (3.32)

contradizendo isto o armado em (3.31).


3.5 Procurando Primos 127

3.5 Procurando Primos

Os números primos além de belos e desaadores do ponto de vista

matemático, são extremamente importantes para as atividades usuais

de nosso dia a dia. Por exemplo, nenhuma transação bancária ou pela

internet estaria segura sem o uso de números primos muito grandes.

Assim, surge naturalmente a pergunta de como podemos produzi-los

em grandes quantidades. Essa pergunta sempre intrigou os matemá-

ticos e continua sem solução até os dias atuais. Apesar deles serem

abundantes, em quantidade innita de acordo com o Teorema 3.42,

não existe nenhum método razoável de produção de números primos,

mesmo tendo em mãos a alta tecnologia de hoje em dia. Porém, ao

longo do tempo algumas fórmulas e algoritmos se mostraram úteis

para a descoberta de números primos.

3.5.1 O Crivo de Eratóstenes

O crivo de Eratóstenes é um algoritmo que nos permite achar todos

os números primos que são menores ou iguais que um natural N dado.

Segundo a tradição, este método foi criado pelo matemático grego

Eratóstenes (285-194 a.C.).

O método consiste nos seguintes passos: escrevemos os números de

forma ordenada a partir de 2, isto é,

2, 3, 4, 5, 6, 7, 8, 9, 10, 11, 12, 13, 14, 15, 16, 17, . . . , n (3.33)

• Observamos que o primeiro primo que aparece em (3.33) é 2 e

imediatamente apagamos da lista (3.33) todos os múltiplos de

2 maiores que ele, por serem compostos; resta assim a seguinte


128 3 Divisibilidade

lista

2, 3, 5, 7, 9, 11, 13, 15, 17 . . .

• O primeiro número não apagado que aparece na lista restante é

3, que também é primo. Imediatamente apagamos da lista todos

os múltiplos de 3 maiores que ele, por serem compostos; resta

agora a lista

2, 3, 5, 7, 11, 13, 17, . . .

• O primeiro número não apagado que aparece na lista que restou

do passo anterior é 5, que também é primo. Imediatamente

apagamos da lista todos os múltiplos de 5 maiores que ele, por

serem compostos.

• Repetimos este processo até que o primeiro número não apagado



da lista em questão seja maior que n, pois graças à Proposição
3.41-(b) a partir desse momento todos os números restantes são

os primos menores ou iguais que n..



Por exemplo, se n = 40, temos que 40 = 6, 324555. Então,

aplicando o método:

2 3 4 5 6 7 8 9 10

11 12 13 14 15 16 17 18 19 20

21 22 23 24 25 26 27 28 29 30

31 32 33 34 35 36 37 38 39 40

Passo 1: ordenamos os números

2 3 5 7 9

11 13 15 17 19

21 23 25 27 29

31 33 35 37 39
3.5 Procurando Primos 129

Passo 2: tiramos os múltiplos de 2

2 3 5 7

11 13 17 19

23 25 29

31 35 37

Passo 3: tiramos os múltiplos de 3

2 3 5 7

11 13 17 19

23 29

31 37

Passo 4: tiramos os múltiplos de 5

Como 72 = 49 > 40, paramos agora.

Observação 3.44. Note que ao começar a apagar os múltiplos de um


número primo p podemos começar a apagar a partir de p2 , pois se
supomos que existe um número composto m não apagado menor que
p2 , temos que m = p1 q1 , sendo p1 seu menor divisor primo. Então,

pelo item (b) da Proposição 3.41, p1 < m < p, logo m deveria ter
sido apagado pois é múltiplo de um primo menor que p.

3.5.2 Primos de Mersenne

Marin Mersenne (1588-1648) foi um monge francês que nasceu na ci-

dade de Maine e foi um dos grandes inuenciadores da Matemática


130 3 Divisibilidade

2 3 5 7 11 13 17 19 23 29

31 37 41 43 47 53 59 61 67 71

73 79 83 89 97 101 103 107 109 113

127 131 137 139 149 151 157 163 167 173

179 181 191 193 197 199 211 223 227 229

233 239 241 251 257 263 269 271 277 281

283 293 307 311 313 317 331 337 347 349

353 359 367 373 379 383 389 397 401 409

419 421 431 433 439 443 449 457 461 463

467 479 487 491 499 503 509 521 523 541

Tabela 3.1: Os primeiros 100 números primos

francesa nos séculos XVI e XVII. Apaixonado pelos números, teve en-

tre seus correspondentes Descartes, Fermat, Pascal e Galileu. Entre

suas várias descobertas, ele estudou os números da forma:

Mn = 2n − 1.

Observe que vale o seguinte fato a respeito desses números:

Proposição 3.45. Se Mn é primo, então n é primo.


Demonstração. Provar essa proposição equivale a mostrar que a sua

forma contrarrecíproca vale. Ou seja, que se n é composto, digamos

n = a.b, com a ≥ b > 1, então Mn também é composto. De fato,

usando o Lema 3.14, podemos decompô-lo do seguinte modo:

Ma.b = 2ab − 1 = 2a(b−1) − 2a(b−2) + · · · + 2a + 1 2b − 1 .


 
3.5 Procurando Primos 131

Porém, não é verdade a recíproca da armação acima. Por exem-

plo, Hudalricus Regius mostrou em 1536 que M11 = 211 − 1 = 2.047


não é primo, já que 2.047 = 23 · 89.
Em 1643, Mersenne armou que para

n = 2, 3, 5, 7, 13, 17, 19, 31, 67, 127 e 257,

os valores de Mn são todos primos e para todos os outros valores de n


menores que 257, Mn é composto.
Hoje sabemos que Mersenne errou na sua armação, esquecendo

três valores de n onde Mn é primo: 61, 89 e 107 e incluindo M67 e M257


como números primos. Para mais informações, sugerimos a página web
http://primes.utm.edu/mersenne/index.html.

Finalizamos esta seção, com um critério interessante, devido à ma-

temática francesa Sophie Germain (1776-1831), que nos permite saber

quando um número não é primo.

Proposição 3.46 (Identidade de Sophie Germain) . Dados a, b ∈ R,


vale a igualdade

a4 + 4b4 = (a2 + 2b2 + 2ab)(a2 + 2b2 − 2ab).

Demonstração. A prova segue das seguintes igualdades:

a4 + 4b4 = a4 + 4a2 b2 + 4b4 − 4a2 b2


= (a2 + 2b2 )2 − 4a2 b2
= (a2 + 2b2 + 2ab)(a2 + 2b2 − 2ab).

Como aplicação desta identidade vejamos os seguintes exemplos.


132 3 Divisibilidade

Exemplo 3.47. qn = n4 + 4n é composto, para todo n ∈ N.


Solução. O conjunto dos números naturais é particionado em duas

classes disjuntas:o conjunto dos números pares e o conjunto dos nú-

meros ímpares. Estudaremos cada classe por separado. Assim,

• sen é um número par, então n = 2m para algum inteiro positivo


m ≥ 1. Deste modo,

n4 + 4n = (2m)4 + 42m = 16m4 + 24m ,


= 2 8m4 + 24m−1 .


Portanto, neste caso, n4 + 4n ≥ 2. Logo, se n>1 é qualquer


4 n
número inteiro positivo par temos que n +4 não é um número

primo;

• se n é um número ímpar, então n = 2m + 1 para algum inteiro

positivo m ≥ 1. Assim,

n4 + 4n = (2m + 1)4 + 42m+1 = (2m + 1)4 + 4 · 42m


= (2m + 1)4 + 4 · 24m = (2m + 1)4 + 4 · (2m )4 .

Logo, tomando a = 2m + 1 e b = 2m , o resultado é uma con-

sequência direta da identidade de Sophie Germain.

Exemplo 3.48. 520 + 230 é um número composto.


Solução. Escrevemos

4 4
520 + 230 = 55·4 + 22 · 228 = 55 + 4 · 27 ,

de onde podemos usar a Identidade de Sophie Germain com a = 55 e


7 20 30
b=2 para comprovar que o número 5 +2 é composto.
3.5 Procurando Primos 133

3.5.3 O Teorema Fundamental da Aritmética

Os números primos são as células dos números naturais, no sentido

de que qualquer número natural é produto de números primos. Por

exemplo,

560 = 56 · 10 = 7 · 8 · 5 · 2 = 7 · 2 · 2 · 2 · 5 · 2,

onde cada um dos fatores que aparecem no produto são números pri-

mos. Perguntamo-nos, o que acontece se começamos com uma outra

fatoração inicial de 560, por exemplo, 560 = 28 · 20. Vejamos:

560 = 28 · 20 = 14 · 2 · 10 · 2 = 7 · 2 · 2 · 5 · 2 · 2.

Surpreendentemente chegamos à mesma representação anterior, salvo

a ordem dos fatores.

2 7
5
2

Figura 3.4: O número 560 é composto de 4 células do tipo 2, uma célula

do tipo 7 e uma célula do tipo 5.

O fato observado acima vale para qualquer número natural maior

que 1. Especicamente, temos o seguinte resultado conhecido como

teorema fundamental da aritmética .


134 3 Divisibilidade

Teorema 3.49 (Teorema Fundamental da Aritmética). Todo número


natural n maior que 1 pode ser escrito como um produto

n = pα1 1 pα2 2 pα3 3 · · · pαmm , (3.34)

onde m ≥ 1 é um número natural, αi ∈ N e pi é primo para todo


1 ≤ i ≤ m . Além disso, a fatoração em (3.34) é única se exigirmos
que p1 < p2 < · · · < pm .
Demonstração. Seja n um inteiro maior que 1. Denotando por p1 seu

menor divisor primo tem-se que

n = p1 β1 , 1 ≤ β1 < n.

Se β1 = 1, entãoN1 = p1 e a fatoração desejada é obtida. Caso


contrário, denotando por p2 o menor divisor primo de β1 tem-se que

n = p1 p2 β2 , 1 ≤ β2 < β1 .

Seβ2 = 1, então n = p1 p2 e novamente chegamos à fatoração desejada.


Caso contrário, denotando por p3 o menor divisor primo de β2 tem-se

que

n = p1 p2 p3 β3 , 1 ≤ β3 < β2 .
Continuando este processo sucessivamente obtemos então uma sequên-

cia estritamente decrescente de números naturais αn , ou seja,

n > β1 > β2 > β2 > · · · > βn > βn+1 > · · · ≥ 1,

Então, pelo princípio da boa ordem, só pode existir uma quantidade

nita de índices n tais que βn > 1 e consequentemente βn+1 = 1, de

onde segue que

n = p1 p2 · · · pn .
3.5 Procurando Primos 135

Notemos que na representação acima os pi podem-se repetir, resul-

tando nalmente a representação desejada em (3.34).

Provaremos agora a unicidade de tal fatoração. Com efeito, supo-

nha que existem duas fatorações:

pα1 1 pα2 2 pα3 3 · · · pαmm = n = q1β1 q2β2 q3β3 · · · qsβs

Pela Proposição 3.43 temos que cada pi divide algum qj , logo pi =


qj , por serem primos. Portanto, cada pi aparece no lado direito da

igualdade acima, e, um argumento análogo nos dá que cada qj também


aparece no lado esquerdo da igualdade. Então, como os pi s e os qj s

são diferentes dois a dois e organizados crescentemente, temos m = s

e a igualdade se reduz a

pα1 1 pα2 2 pα3 3 · · · pαmm = pβ1 1 pβ2 2 pβ3 3 · · · pβmm .

Suponhamos agora que α1 seja diferente de β1 ; sem perda de ge-

neralidade vamos supor que α1 < β1 . Portanto,

pα2 2 · pα3 3 · · · pαmm = pβ1 1 −α1 pβ2 2 pβ3 3 · · · pβmm ,

e como β1 − α1 > 0 então, pela Proposição 3.43 temos que p1 di-


vide algum pj , com j > 1, o que é impossível. Portanto, α1 = β1 .

Similarmente provamos que αi = βi , com i = 1, . . . , n.

Observação 3.50. O teorema fundamental da aritmética foi enun-


ciado precisamente por Gauss (1777-1855). Seus antecessores, Fer-
mat, Euler, Lagrange e Legendre, utilizavam este teorema sem a preo-
cupação de tê-lo enunciado ou demonstrado com precisão. Uma prova
alternativa deste teorema será apresentada no Capítulo 6, usando o
método de indução.
136 3 Divisibilidade

Exemplo 3.51. Prove que um número n é par se, e somente se, o


número 2 aparece na fatoração de n em fatores primos.
Solução. Obviamente, se 2 aparece na fatoração em primos de N,
então N é par. Ora, se n é par temos que n = 2q . Por outro lado qe
n se fatoram, respectivamente, como

q = q1α1 q2α2 · · · qm
αm
e n = pβ1 1 pβ2 2 · · · pβs s .
Logo,

2 · q1α1 q2α2 · · · qm
αm
= pβ1 1 pβ2 2 · · · pβs s .
Pela unicidade da fatoração, para algum i, com 1 ≤ i ≤ s, o cor-

respondente pi deve ser igual a 2. Portanto, 2 aparece na fatoração de

n.
Exemplo 3.52. Seja A = {1, 2, 3, 4, 5, 6, 7}. É possível decompor
o conjunto A em dois subconjuntos disjuntos tais que o produto dos
elementos de um seja igual ao produto dos elementos do outro?
Solução. Mostraremos que é impossível fazer esta decomposição. Com

efeito, suponha que existem tais conjuntos, A1 = {p1 , p2 , . . . , pr } e

A2 = {q1 , q2 , . . . , qs }. Então

p1 p2 · · · pr = q1 q 2 · · · qs
| {z } | {z }
α β
e além disso, como os conjuntos A1 e A2 são disjuntos, temos que o

número 5 aparece no produto α ou no produto β, mas não em ambos

simultaneamente. Por outro lado, o Teorema 3.49 nos diz que a fatora-

ção em primos de α é igual à fatoração em primos de β , logo o número


5 deveria aparecer tanto no produto α como no produto β , contra-

dizendo isto o fato anterior. Portanto não existe uma decomposição

com as condições exigidas.


3.5 Procurando Primos 137

Exemplo 3.53. Encontre todos os números inteiros e positivos n com


a propriedade de que o conjunto

A = {n, n + 1, n + 2, n + 3, n + 4, n + 5}

pode ser particionado em dois subconjuntos tais que o produto dos


elementos de um dos subconjuntos seja igual ao produto dos elementos
do outro.

Demonstração. Digamos que seja possível essa decomposição para al-

gum n e vamos denotar os conjuntos que obtemos com a decomposição


por A1 e A2 . Observando a decomposição dos elementos dos subcon-

juntos em fatores primos, temos que todo fator primo de A1 também

deverá pertencer a A2 . No conjunto dos seis números só podemos ter

um múltiplo de 7, por isso não podemos tomar n como múltiplo deste

primo. Analogamente para primos maiores que 7. Analisando o primo

5, concluímos que n e n + 5 são múltiplos de 5, pois se não, cairíamos


na análise anterior. Assim, os números n + 1, n + 2, n + 3 e n + 4 são
α β
da forma 2 3 . Como entre eles existem dois ímpares, logo teremos

duas potências de 3 cuja diferença é 2, um absurdo. Assim, não existe

n que satisfaz as condições do enunciado.

Finalizamos esta seção com um exemplo que mostra como podemos

combinar os fatos estudados para resolver problemas mais difíceis

Exemplo 3.54. Encontre todos os números que são formados por 4


algarismos da forma aabb e que sejam quadrados perfeitos.
138 3 Divisibilidade

Solução. Como o número aabb é um quadrado perfeito, signica que:

n2 =aabb
n2 =103 a + 102 a + 10b + b = 103 + 102 · a + (10 + 1) · b


n2 =1100 · a + 11 · b
n2 =11 100a + b = 11 99a + a + b .
 

Como 11 é primo é fácil ver, usando a Proposição 3.43, que 112 | N 2 .


Segue-se então que 11 | (99a+a+b). Portanto, 11 | (a+b). Como aabb

tem 4 algarismos, segue-se que a 6= 0; portanto a ∈ {1, 2, 3, . . . , 9} e

b ∈ {0, 1, 2, . . . , 9}. De onde a + b ≤ 18. Logo, necessariamente


devemos ter a + b = 11. Podemos observar que a 6= 1, pois se a = 1

então b = 10. Analogamente, b 6= 0, 1. Portanto,

a ∈ {2, 3, 4, . . . , 9} e b ∈ {2, 3, 4, . . . , 9}.

Como em todo número quadrado perfeito o algarismo das unidades

somente pode acabar em 0, 1, 4, 5, 6 e 9. Segue-se que

b ∈ {4, 5, 6, 9}.

Certamente b 6= 5, pois todo número que acaba em 5 quando é elevado


ao quadrado sempre acaba em 25. Assim,

b ∈ {4, 6, 9}.

• Se b = 4, então a = 7. Neste caso o número seria 7.744 que é

um quadrado perfeito;

• Se b = 6, então a = 5. Neste caso o número seria 5.566 que não

é um quadrado perfeito;
3.6 Exercícios 139

• Se b = 9, então a = 2. Neste caso o número seria 2.299 que não

é um quadrado perfeito.

Finalmente, a única solução possível é aabb = 7.744 = 882 .

3.6 Exercícios

1. Encontre o resto que deixa

(a) 2001 · 2002 · 2003 · 2004 + 20052 quando é dividido por 7;

(b) 2100 quando é dividido por 3;

28
(c) (1237156 + 34) quando é dividido por 111.

2. Provar que o número n5 + 4n é divisível por 5 para qualquer

número natural n.

3. Prove que se n é ímpar

(a) n3 − n é divisível por 24;

(b) n2 − 1 é divisível por 8;

(c) n2 + (n + 2)2 + (n + 4)2 + 1 é divisível por 12.

4. O número 21093 − 2 é divisível por 10932 ?

5. Prove que (999994)1234567890 − 1 é divisível por 333331.

6. O número N = 42005 + 20054 é primo?

7. Demonstre que o número 1 |000 {z


. . . 00} 1 é composto.
2006 zeros
140 3 Divisibilidade

8. Utilizando o fato de que o resto de um quadrado quando dividido

por 4 só pode ser 0 ou 1, dê uma outra solução para o problema

do Exemplo 3.54.

9. Dados três inteiros, x, y, z , tais que x2 + y 2 = z 2 , mostre que x


e y não são ambos ímpares e que xy é múltiplo de 6.

10. Demonstre que o quadrado de um inteiro é da forma 8n ou 8n+1


ou 8n + 4.

11. Três números primos p, q e r, maiores que 3, formam uma pro-


gressão aritmética, ou seja, q = p + d e r = p + 2d. Prove que d
é divisível por 6.

12. Demonstrar que existem innitos números primos da forma 4m+


3 e da forma 6m + 5, onde m ∈ Z.

13. Encontrar o último dígito dos números

(a) 19892005 ;
(b) 777777 + 250 ;
(c) 1 + 22 + 32 + · · · + 20052 .

14. Prove que a soma dos quadrados de cinco números consecutivos

não é um quadrado perfeito.

15. Prove que 1 |00 ·{z


· · 00} 5 |00 ·{z
· · 00} 1 não é um cubo perfeito.
100−zeros 100−zeros

16. Seja b um inteiro positivo. Enuncie e prove o critério de divisi-

bilidade por b no sistema de numeração de base b.

17. Prove que os números


3.6 Exercícios 141

1 1 1
(a) αn = 1 ++ + · · · + , com n > 1,
2 3 n
1 1 1
(b) βn = + + · · · + , com n > 0,
3 5 2n + 1
não são inteiros.

18. Considere o polinômio p(n) = am nm + am−1 nm−1 + · · · + a0 de


grau m ≥ 1 com coecientes inteiros e n ∈ N. Prove que p(n) é

um número composto para innitos valores de n.

Sugestão: Use o fato de que existe a ∈ N tal que α = |p(a)| > 1


e mostre que α divide a p(αk + a), para todo k ∈ Z.

19. Dizemos que um conjunto An formado por n inteiros positivos

escritos no sistema binário (base 2) é regular se, para qualquer

s inteiro não negativo a quantidade de números de An que con-


s
templam 2 na representação binária é par. Dizemos que An é

irregular se, pelo menos para algum s, este número é ímpar. De-

monstre que um sistema irregular pode se converter em regular

excluindo-se apenas um único elemento do mesmo, e, um sistema

regular pode se converter em irregular excluindo-se qualquer um

dos seus elementos.

20. Seja n um inteiro positivo. Demonstrar que todos os coecientes

do desenvolvimento do binômio de Newton (a + b)n são ímpares


s
se, e somente se, n é da forma 2 − 1.

21. Prove que se (x0 , y0 ) é uma solução da equação diofantina linear


ax − by = 1, então a área do triângulo cujos vértices são (0, 0),
(b, a) e (x0 , y0 ) é 1/2.
142 3 Divisibilidade

22. Qual é a menor distância possível entre dois pontos (x1 , y1 ) e

(x2 , y2 ), com coordenadas inteiras, situados sobre a reta denida


pela equação diofantina ax + by = c?
4
O Princípio da Casa dos
Pombos

Uma vez um matemáti o me falou que o verdadeiro prazer não está

em a har a verdade, mas em pro urar por ela.

Leo Tolstoy

Um interessante instrumento elementar para tratar problemas mate-

máticos relacionados à existência de elementos de conjuntos validando

certas exigências é o chamado princípio de Dirichlet , também conhe-


cido como princípio da casa dos pombos (PCP) . Este princípio foi
usado por Dirichlet (1805-1859) para resolver problemas na Teoria

dos Números, entretanto ele possui um grande número de aplicações

em diversos ramos da Matemática como Combinatória e Geometria.

A seguir enunciamos a versão mais simples do PCP.

Proposição 4.1 (PCP  Versão Simples) . Se distribuímos N + 1


pombos em N casas, então alguma das casas contém dois ou mais
pombos.

143
144 4 O Princípio da Casa dos Pombos

P1 P2 ········· PN
C1 C2 CN

PN +1

Figura 4.1: Em cada casa Cj , 1 ≤ j ≤ N , coloca-se um único pombo,

denotado por Pj . O pombo restante, denotado por PN +1 , deve ir para

alguma das casas, juntando-se ao que já se encontrava contido nela

Demonstração. A prova deste princípio é muito fácil e decorre de fa-

zer uma simples contagem dos pombos contidos em todas as casas de-

pois de distribuídos. Com efeito, suponhamos pelo contrário que em

cada casa não existe mais do que um pombo, então contando todos

os pombos contidos nas N casas não teremos mais do que N pombos,

contradizendo isto a hipóteses de termos N +1 pombos distribuídos

nas N casas (ver Figura 4.1).

Não é difícil detectar quando o princípio pode ser usado, mas a

principal diculdade para aplicá-lo reside em identicar, em cada pro-

blema, quem faz papel de pombos e quem faz papel de casas.

Nas seguintes seções discutiremos vários exemplos de diferentes

naturezas onde o princípio da casa dos pombos é aplicado com sucesso.


4.1 Primeiros Exemplos 145

4.1 Primeiros Exemplos

Exemplo 4.2. Numa oresta crescem 1.000 jaqueiras. É conhecido


que uma jaqueira não contém mais do que 600 frutos. Prove que
existem 2 jaqueiras na oresta que têm a mesma quantidade de frutos.

Solução. Temos 1.000 jaqueiras, representando os pombos, e 601 casas

identicadas pelos números 0, 1, 2, 3, . . . , 600. O número k associado

a cada casa signica que nela serão colocadas jaqueiras que têm exa-

tamente k frutos. Como 1000 > 602 = 601 + 1, o PCP nos garante

que existem duas jaqueiras com a mesma quantidade de frutos.

Exemplo 4.3. Em uma reunião há n pessoas. Mostre que existem


duas pessoas que conhecem exatamente o mesmo número de pessoas.

Solução. Os pombos neste caso são as n pessoas. As casas são enume-

radas com os números 0, 1, 2, . . . , n − 1, indicando estes que na mesma


serão colocadas pessoas que têm essa quantidade de conhecidos. No-

temos que uma das casas enumeradas com 0 ou n − 1 permanece


desocupada, pois a possibilidade de conhecer 0 e n − 1 pessoas não

acontece simultaneamente. Logo, nas n − 1 casas restantes haverá

uma ocupada por dois ou mais pombos, depois de serem distribuídos.

Portanto, existem no mínimo duas pessoas com o mesmo número de

conhecidos.

Exemplo 4.4. Dados 8 números inteiros mostre que existem dois


deles cuja diferença é divisível por 7.

Solução. Consideramos os 8 números como sendo os pombos e as casas

como sendo os 7 possíveis restos na divisão por 7. Como temos 8=


7+1 números o PCP nos diz que existem dois números dentro dos
146 4 O Princípio da Casa dos Pombos

8 dados que têm o mesmo resto quando divididos por 7. Finalmente,

observamos que se dois números deixam o mesmo resto na divisão por

7 então a diferença entre eles é divisível por 7.

Uma forma alternativa e muito útil na qual pode-se apresentar o

princípio da casa dos pombos é a seguinte:

Proposição 4.5 (PCP  Versão Alternativa) . Se a soma de n nú-


meros naturais é igual S , então existe pelo menos um deles que não
é maior que S/n, assim como existe pelo menos um deles que não é
menor que S/n.
Exemplo 4.6. Numa família formada por 5 pessoas a soma das idades
é de 245 anos. Prove que podem ser selecionados 3 membros da família
cuja soma das idades não é menor que 147.
Solução. 5 5!

Temos um total de
3
= 3!2!
= 10 trios diferentes formados

por membros da família. Além disso, cada pessoa aparece exatamente


4 4!

em
2
= 2!2!
=6 trios. Então, denotando por Ej a soma das idades

dos membros de cada trio Tj , j = 1, 2 . . . 10, temos que

E1 + E2 + · · · + E10 = 6 · 245 = 1470;


1470
consequentemente existe algum trio Tj ∗ tal que Ej ∗ ≥ 10
= 147.

4.2 Uma Versão mais Geral

A seguinte versão mais geral do PCP é bastante útil na resolução de

alguns problemas.

Proposição 4.7 (PCP  Versão Geral). Se distribuímos N k + 1 pom-


bos em N casas, então alguma das casas contém pelo menos k + 1
pombos.
4.2 Uma Versão mais Geral 147

A prova deste enunciado mais geral é similar à anterior. Com efeito,

suponhamos pelo contrário que em cada casa não existe mais do que

k pombos, então contando todos os pombos contidos nas N casas não

teremos mais do que Nk pombos, contradizendo isto a hipóteses de

termos Nk + 1 pombos distribuídos nas N casas.

Notemos que se k = 1, esta versão mais geral coincide com a versão


mais simples.

Exemplo 4.8. Num colégio com 16 salas são distribuídas canetas nas
cores preta, azul e vermelha para realizar uma prova de concurso. Se
cada sala recebe canetas da mesma cor então prove que existem pelo
menos 6 salas que receberam canetas da mesma cor.
Solução. Fazendo a divisão com resto de 16 por 3 temos que 16 =
3 · 5 + 1. Consideramos as 16 salas como sendo os pombos e as três

cores, preto, azul e vermelho como sendo as casas. Logo, podemos

colocar cada sala em uma das três cores. Assim, o PCP com N =3
e k = 5 nos dá que existe uma casa com pelo menos 6 pombos, ou seja,
existem no mínimo 6 salas que receberam canetas da mesma cor.

Exemplo 4.9. Uma equipe formada por seis alunos de Matemática é


selecionada para representar o Brasil numa olimpíada internacional.
Mostre que necessariamente existem três deles que se conhecem mu-
tuamente, ou três deles que não se conhecem mutuamente.
Solução. Resolveremos o problema com o auxílio da Figura 4.2. Cada

aluno Aj , com j = 1, 2, . . . , 6, é representado por um dos vértices de

um hexágono regular. Quando dois alunos se conhecem traçamos o

segmento de reta que liga os vértices correspondentes com uma linha

contínua; caso contrário traçamos este segmento com uma linha pon-

tilhada. Logo, usando este esquema, o problema equivale a provar


148 4 O Princípio da Casa dos Pombos

que sempre existe um triângulo de lados contínuos ou um triângulo de

lados pontilhados com vértices no conjunto A = {A1 , A2 , . . . , A6 }.


Temos 5 segmentos (pombos) incidindo no vértice A1 , cada um

deles contínuo ou pontilhado (estes dois tipos de linhas são conside-

radas como as casas). Como 5 = 2 · 2 + 1, pelo PCP temos que 3

dos 5 segmentos são contínuos ou pontilhados. Suponhamos que 3 são

contínuos (caso contrário o argumento é similar) e denotemos estes

por A1 A3 , A1 A4 e A1 A6 (ver Figura 4.2). Se algum dos segmentos


A3 A4 , A3 A6 ou A4 A6 for contínuo então este segmento junto aos que
se ligam com A1 formam um triângulo de lados contínuos. Por outro

lado, se nenhum deles for contínuo, então eles formam um triângulo

de lados pontilhados, completando isto a demonstração.

A3 A2

A4 A1

A5 A6

Figura 4.2: O triângulo A1 A2 A5 indica que os alunos A1 , A2 e A5 não se

conhecem mutuamente e o triângulo A1 A4 A6 indica que os alunos A1 , A4


e A6 se conhecem mutuamente
4.3 Aplicações na Teoria dos Números 149

4.3 Aplicações na Teoria dos Números

Nesta seção apresentamos alguns exemplos de aplicações do PCP na

Teoria dos Números. A primeira delas é:

Exemplo 4.10. Se n e m são números naturais, então o conjunto


A = {m + 1, m + 2, . . . , m + n} possui algum divisor de n.

Solução. Temos n números diferentes no conjunto acima. Vamos utili-

zar o método de redução ao absurdo. Se não existisse nenhum múltiplo

de n, quando dividíssemos os números do conjunto A por n, os res-

tos pertenceriam ao conjunto B = {1, 2, . . . , n − 1}, que possui n − 1


elementos. Logo, devem existir dois números m + i e m + j , com

1 ≤ i < j ≤ n tais que o resto da divisão de m + i por n é o mesmo


que o resto da divisão de m + j por n. Logo, m + j − (m + i) é um

múltiplo de n, o que implica que n > j − i ≥ 1 é múltiplo de n menor

que n (absurdo!). Logo, deve existir algum múltiplo de n no conjunto

A.

Como consequência desse exemplo, podemos resolver o próximo

problema.

Exemplo 4.11. Demonstrar que todo inteiro tem um múltiplo cuja


representação decimal começa com o bloco de dígitos 1234567890.

Solução. m e n são inteiros positivos, pelo exemplo anterior um


Se

dos número m + 1, m + 2, . . . , m + n é múltiplo de n. Assim, dado n


n+1
um inteiro qualquer, escolhe-se m = 1234567890×10 . Deste modo,

todos os inteiros m + 1, m + 2, . . . , m + n começam com 1234567890 e

algum deles é múltiplo de n.


150 4 O Princípio da Casa dos Pombos

Exemplo 4.12. Dado um número inteiro positivo n, mostre que existe


um múltiplo de n que se escreve com os algarismos 0 e 1 apenas. (Por
exemplo, se n = 3, temos 111 ou 1.101 etc.)

Solução. Consideramos os n+1 números

1, 11, 111, 1111, . . . , 111


| {z· · · 1} (4.1)
n+1−vezes

como sendo os pombos e n casas enumeradas com os números

0, 1, 2, 3, . . . , n − 1,

ou seja, com os possíveis restos na divisão por n. Similarmente ao

exemplo anterior existem dois números na lista (4.1) que deixam o

mesmo resto na divisão por n e, portanto, a diferença entre o maior e

o menor é múltiplo de n. Obviamente a diferença entre dois números

quaisquer da lista (4.1) resulta em um número formado apenas pelos

algarismos 0 e 1.

Exemplo 4.13. Prove que entre n + 1 elementos escolhidos no con-


junto {1,2,3, . . . , 2n} existem dois que são primos relativos.

Solução. A escolha das casas e dos pombos neste exemplo não é tão ób-

via. Os pombos representam os n + 1 números escolhidos do conjunto


{1, 2, . . . , 2n} e as casas são escolhidas como sendo os n conjuntos:

Cj = {2j − 1, 2j}, 1 ≤ j ≤ n.

Logo, pelo PCP, quando distribuímos os n + 1 números nos n conjun-


tos Cj , 1 ≤ j ≤ n, dois deles carão juntos em algum conjunto Cj , ou

seja, estes números serão consecutivos e portanto primos entre si.


4.4 Aplicações Geométricas 151

Finalizaremos esta seção com uma outra prova do teorema de

Bachet-Bézout, (veja o Teorema 3.23).

Exemplo 4.14. Seja d = (a, b) o mdc entre os números naturais a e


b. Então, existem x e y números inteiros tais que

ax + by = d.

Solução. Denotando por m = a/d e n = b/d, podemos supor que a e

b são primos entre si. Realmente, se podemos escrever

mx + ny = 1

então, substituindo os valores de m e n na equação acima, temos que

ax + by = d.
Se (a, b) = 1, considere a sequência A = {a, 2a, . . . , ba}. Armamos

que existe algum número no conjunto A que deixa resto 1 quando

dividido por b. De fato, se isso não ocorresse, teríamos b números em

A deixando no máximo b − 1 restos diferentes quando divididos por


b. Logo, pelo PCP, dois deles, digamos ia e ja com b > j > i ≥ 1,
devem deixar o mesmo resto quando divididos por b. assim, (j − i)a

é divisível por b. Como estamos supondo que (a, b) = 1, temos que b

deve dividir j − i > 0. Como b > j − i, temos um absurdo.

Assim, algum dos números em a deixa resto 1 quando divididos

por b. Digamos que esse número seja ax. Logo, ax − 1 é múltiplo de

b, onde ax − 1 = by , o que encerra nossa prova.

4.4 Aplicações Geométricas

Na geometria também encontramos belas aplicações do PCP. Vejamos


os problemas a seguir para constatar isto.
152 4 O Princípio da Casa dos Pombos

Exemplo 4.15. Mostre que se tomamos cinco pontos quaisquer sobre


um quadrado de lado 1, então pelo menos dois deles não distam mais

que 2/2.

Solução. Vamos dividir o quadrado em quatro quadradinhos de lado

1/2, como mostra a gura. Logo, pelo PCP pelo menos dois deles de-




1
• •

vem estar no mesmo quadradinho, uma vez que temos 4 quadradinhos

e 5 pontos. Logo, como a maior distância num quadrado é a diagonal,

o Teorema de Pitágoras nos garante que a distância desses dois pontos



é no máximo 2/2, como queríamos mostrar.

Exemplo 4.16. Na região delimitada por um triângulo equilátero de


lado 4 são marcados 10 pontos no interior deste. Prove que existe ao
menos um par destes pontos cuja distância entre eles não é maior que

3.

Solução. Dividimos o triângulo equilátero de lado 4 em 16 triângulos

equiláteros menores de lado 1, conforme a Figura 4.3.

Agora pintamos os triângulos nas cores branco e cinza de maneira

que dois triângulos vizinhos, isto é, com um lado comum, são pintados

de cores diferentes. Se tivéssemos dois pontos no mesmo triângulo a

distância máxima possível entre eles seria 1 e o problema estaria resol-

vido. Se tivéssemos pontos em triângulos vizinhos, a maior distância



possível entre eles seria 3 e também isto resolveria o problema. Se

não tivéssemos nenhum dos casos anteriores, não seria difícil ver que
4.5 Miscelânea 153

C

E
• •

• •
• •
• •
A D B

Figura 4.3: O triângulo DBE é equilátero de lado 3

os 10 pontos deveriam estar situados sobre os 10 triângulos brancos,

contendo cada triângulo exatamente um ponto. Dividindo o triângulo

DBE em 4 triângulos congruentes de lado 3/2 pelo PCP temos que


pelo menos dois dos 6 pontos contidos em DBE estão num destes 4

triângulos, logo a distância entre eles não é maior que 3/2 < 3. Com
isto terminamos nossa prova.

4.5 Miscelânea

Os problemas que apresentamos a seguir usam o PCP combinado com

outras idéias que são muito empregadas nas suas soluções.

Exemplo 4.17. Em cada quadradinho de um tabuleiro 3 × 3 é colocado


um dos números: -1, 0 ou 1. Prove que entre todas as somas das
linhas, colunas e diagonais do tabuleiro há duas que são iguais. Por
exemplo, no tabuleiro abaixo a soma da segunda linha é 2, que coincide
com a soma da terceira coluna.
154 4 O Princípio da Casa dos Pombos

-1 -1 1

1 0 1

0 -1 0

Solução. SejaS = a1 + a2 + a3 , onde cada a1 , a2 e a3 podem tomar


valores: −1, 0 e 1. Então, temos 7 valores possíveis para S (casas),

que são: −3, −2, −1, 0, 1, 2, 3.

O tabuleiro 3×3 tem 3 linhas, 3 colunas e 2 diagonais, portanto, ao

somarmos os elementos de cada uma das linhas, colunas e diagonais,

obteremos 8 números (pombos). Como existem somente 7 valores

possíveis para estes números, pelo PCP pelo menos dois deles devem

ser iguais.

Exemplo 4.18. Dado qualquer conjunto A formado por 10 números


naturais escolhidos entre 1 e 99, inclusos, demonstre que existem dois
subconjuntos disjuntos e não vazios de A tal que a soma dos seus res-
pectivos elementos é igual.

Solução: É conhecido que A tem 210 − 1 = 1.023 subconjuntos não-

vazios diferentes. A soma dos elementos de cada um deles dá uma

quantidade menor do que 1.000, pois o subconjunto com no máximo

10 elementos de maior soma possível é o formado por 90, 91, . . . , 99,


e nesse caso 90 + 91 + · · · + 99 = 945. Agora consideramos os pombos

como sendo os 1.023 subconjuntos distintos de A e as casas como

sendo as somas possíveis dos elementos de cada um dos conjuntos.

Logo, como o número de conjuntos é maior que o número de somas

possíveis, devem existir dois conjuntos B e C de A, de tal modo que

a soma dos elementos de B é igual à soma dos elementos de C. Se B


4.5 Miscelânea 155

e C são disjuntos, acabou a prova. Se não, considere D = B − B ∩ C


e E = C − B ∩ C . Logo, os conjuntos D e E são disjuntos e a soma
dos seus elementos é a mesma, pois retiramos de ambos a mesma

quantidade.

Exemplo 4.19. Qual é o maior número de quadradinhos de um ta-


buleiro de 8 × 8 que podem ser pintados de preto, de forma tal que
qualquer arranjo de três quadradinhos, como mostra a Figura 4.4, te-
nha pelo menos um dos quadradinhos não pintado de preto?

Figura 4.4: Tridominós

Solução. Primeiramente, pintamos o tabuleiro de 8×8 como um tabu-


leiro de jogar xadrez, ou seja, 32 quadradinhos pintados de branco e

32 quadradinhos pintados de preto (ver Figura 4.5).

Figura 4.5: Tabuleiro de xadrez


156 4 O Princípio da Casa dos Pombos

Notemos que uma vez pintado o tabuleiro desta forma é satisfeita

a exigência do problema, pois nunca temos 2 quadradinhos vizinhos

(quadradinhos com um lado comum) pintados de preto.

Mostraremos agora que se pintamos 33 quadradinhos de preto en-

tão a condição exigida no problema falha. De fato, se dividimos o

tabuleiro em 16 quadrados de 2×2 (casas) e pintamos 33 quadra-

dinhos de preto (pombos); então, como 33 = 16 · 2 + 1, pela versão


geral do PCP um dos 16 quadrados de 2 × 2 contém 3 quadradinhos
pintados de preto. Portanto, este último contém um arranjo como na

Figura 4.4 completamente pintado de preto.

Resumindo, o número máximo de quadradinhos que podemos pin-

tar de preto é 32.

Exemplo 4.20. Dados sete números reais arbitrários, demonstre que


existem dois deles, digamos x e y , tais que
x−y 1
0≤ ≤√
1 + xy 3
Solução. Primeiramente observamos que a expressão
x−y
1+xy
nos faz pen-

sar na fórmula
tan α − tan β
tan(α − β) = . (4.2)
1 + tan α tan β
Sejam x1 , x2 , · · · , x7 os sete números selecionados arbitrariamente.

Lembramos que a função tangente é uma bijeção entre o intervalo

(− π2 , π2 ) e os números reais R, logo para cada xi , 1 ≤ i ≤ 7, existe um


αi ∈ (− π2 , π2 ) tal que tan(αi ) = xi . Dividimos o intervalo (− π2 , π2 ) em
π
seis subintervalos de comprimento , como mostra o desenho a seguir.
6

Pelo PCP dois dos números αi pertencem ao mesmo subintervalo.

Denotemos os mesmos por αi1 e αi2 e suponhamos, sem perda de


4.6 Exercícios 157

αi1 αi2
− π2 π
π
2
6

generalidade, que αi1 ≤ αi2 . Então vale

π
0 ≤ αi2 − αi1 ≤ .
6
Usando o fato de que a tangente é uma função crescente e a fórmula

(4.2) temos que

π
tan(0) ≤ tan(αi2 − αi1 ) ≤ tan( ).
6
Equivalentemente,
xi 2 − xi 1 1
0≤ ≤√ .
1 + xi 2 xi 1 3

4.6 Exercícios

1. Seja C um conjunto formado por cinco pontos de coordenadas

inteiras no plano. Prove que o ponto médio de algum dos seg-

mentos com extremos em C tem também coordenadas inteiras.

2. O conjunto dos dígitos 1, 2, ..., 9 é dividido em três grupos.

Prove que o produto dos números de algum dos grupos deve ser

maior que 71.

3. Prove que se N é ímpar então para qualquer bijeção

p : IN → IN
158 4 O Princípio da Casa dos Pombos

do conjunto IN = {1, 2, . . . , N } o produto P (p) = (1 − p(1))(2 −


p(2)) · · · (N − p(N )) é necessariamente par.
(Dica: O produto de vários fatores é par se, e somente se, um dos

fatores é par.)

4. Dado um conjunto de 25 pontos no plano tais que entre quaisquer

3 deles existe um par com distância menor que 1. Prove que

existe um círculo de raio 1 que contém pelo menos 13 dos 25

pontos dados.

5. Prove que entre quaisquer 5 pontos escolhidos dentro de um

triângulo equilátero de lado 1 sempre existe um par deles cuja

distância não é maior que 0,5.

6. Marquemos todos os centros dos 64 quadradinhos de um ta-

buleiro de xadrez de 8 × 8. É possível cortar o tabuleiro com 13

linhas retas que não passem pelos pontos marcados e de forma

tal que cada pedaço de recorte do tabuleiro tenha no máximo

um ponto marcado?

7. Prove que existem duas potências de 3 cuja diferença é divisível

por 1.997.

8. São escolhidos 6 números quaisquer pertencentes ao conjunto

A = {1, 2, 3, . . . , 10}.

Prove que existem dois desses seis números cuja soma é ímpar.

9. Seja x um número real arbitrário. Prove que entre os números

x, 2x, 3x, . . . , 101x


4.6 Exercícios 159

existe um tal que sua diferença com certo número inteiro é menor

0,011.

10. Mostre que entre nove números que não possuem divisores pri-

mos maiores que cinco, existem dois cujo produto é um qua-

drado.

11. Um disco fechado de raio um contém sete pontos, cujas distân-

cias entre quaisquer dois deles é maior ou igual a um. Prove que

o centro do disco é um destes pontos.

12. Na região delimitada por um retângulo de largura quatro e altura

três são marcados seis pontos. Prove que existe ao menos um



par destes pontos cuja distância entre eles não é maior que 5.

13. Seja a um número irracional. Prove que existem innitos núme-

ros racionais r = p/q tais que |a − r| < 1/q 2 .

14. Suponha que cada ponto do reticulado plano é pintado de vermelho


ou azul. Mostre que existe algum retângulo com vértices no reticulado
e todos da mesma cor.

15. Um certo livreiro vende pelo menos um livro por dia. Sabendo que o
livreiro vendeu 463 livros durante 305 dias consecutivos, mostre que
em algum período de dias consecutivos o livreiro vendeu exatamente
144 livros.
Referências Bibliográcas
[1] AIGNER, M. e ZIEGLER, G. (2002). As Provas estão
no Livro. Edgard Blücher.

[2] GARCIA, A. e LEQUAIN, I. (2003). Elementos de Ál-


gebra. Projeto Euclides, IMPA.

[3] LIMA, E. L.; CARVALHO, P. C. P.; WAGNER, E. e

MORGADO, A.C. (2004). A Matemática do Ensino Mé-


dio. Volume 1. Sociedade Brasileira de Matemática.

[4] LIMA, E.L.; CARVALHO, P. C. P.; WAGNER, E. e

MORGADO, A.C. (2004). A Matemática do Ensino Mé-


dio. Volume 2. Sociedade Brasileira de Matemática.

[5] LIMA,E.L.; CARVALHO,P. C. P.; WAGNER,E. e

MORGADO,A.C. (2004). A Matemática do Ensino Mé-


dio. Volume 3. Sociedade Brasileira de Matemática.

[6] LIMA, E.L.; CARVALHO, P. C. P.; WAGNER,E. e

MORGADO, A.C. (2001). Temas e Problemas. Socie-

dade Brasileira de Matemática.

[7] LIMA, E.L. (2001). Álgebra Linear. Sociedade Brasileira


de Matemática.

285
286 REFERÊNCIAS BIBLIOGRÁFICAS

[8] MORAIS FILHO, D. C. (2007). Um Convite à Matemá-


tica. EDUFCG.

[9] MORGADO, A.; CARVALHO, J.; CARVALHO, P.;

FERNANDEZ, P. (1991). Análise Combinatória e Pro-


babilidade . Sociedade Brasileira de Matemática.

[10] RIBENBOIM, P. (2001). Números Primos: Mistérios e


Recordes. Sociedade Brasileira de Matemática.

[11] SANTOS, J. P. O. (1993) Introdução à Teoria dos Nú-


meros. IMPA.

[12] SANTOS, J. P. O.; MELLO, M. P. e MURARI, I. T.

C. (2006). Introdução à Análise Combinatória. Editora

Unicamp.

[13] SOARES, M. G. (2005). Cálculo em uma Variável Com-


plexa. Sociedade Brasileira de Matemática.
Mestrado Profissional
em Matemática em Rede Nacional

Iniciação à Matemática

Autores:

Krerley Oliveira Adán J. Corcho

Unidade III:

Capítulos V e VI
160
5
Contagem

Toda vez que puder, onte.

Fran is Galton

Neste capítulo discutiremos problemas envolvendo a contagem de


elementos de um conjunto nito dado. Por exemplo, responderemos
perguntas do tipo: de quantos modos podemos distribuir 32 seleções
nacionais de futebol em seis grupos de quatro times cada?
Para solucionar questões como esta, utilizaremos como ferramentas
básicas os princípios aditivo e multiplicativo da contagem. Veremos
também que o uso simultâneo destes princípios será muito útil para
resolver problemas com certos níveis de complexidade. Além disso,
serão abordados os conceitos de permutações, arranjos e combinações,
sendo estes de muita importância por serem os alicerces de um ramo
da matemática denominado combinatória.
Antes de prosseguirmos daremos algumas denições e notações que
serão úteis ao longo de todo o capítulo. Dado um conjunto A deno-
tamos por |A| a quantidade de elementos que este possui. O produto
cartesiano de n conjuntos A1 , A2 , . . . , An−1 e An é o conjunto denido

161
162 5 Contagem

por

A1 × A2 × · · · × An := (a1 , a2 , . . . , an ); ai ∈ Ai , i = 1, 2, . . . , n ,

onde cada elemento (a1 , a2 , . . . , an ) é chamado de n-upla ordenada.


Denotaremos o conjunto vazio com o símbolo ∅. O leitor que deseja
rever os conceitos básicos da teoria de conjuntos, pode achá-los muito
bem expostos em [3].

5.1 Princípio Aditivo da Contagem

O princípio aditivo da contagem garante que dados dois conjuntos


nitos que não têm elemento em comum, o número de elementos da
união é exatamente a soma do número de elementos de cada um, ou
seja, se A1 e A2 são disjuntos (isto é, A1 ∩ A2 = ∅), então
|A1 ∪ A2 | = |A1 | + |A2 |.

Apesar de sua simplicidade, muitos problemas podem ser resolvi-


dos utilizando esse simples princípio. A seguir enunciamos uma ex-
tensão deste princípio para um número nito qualquer de conjuntos.

Princípio Aditivo da contagem: Dados os conjuntos nitos A1 ,


A2 , . . . , An dois a dois disjuntos (isto é, Ai ∩ Aj = ∅ , ∀ i 6= j ),
temos que
|A1 ∪ A2 ∪ · · · ∪ An | = |A1 | + |A2 | + · · · + |An |.

Exemplo 5.1. Em Maceió entraram em cartaz 4 lmes distintos e


2 peças de teatro. Se Pedro Vítor só tem dinheiro para assistir a um
lme ou a uma peça de teatro, diga quantos são os possíveis programas
de Pedro Vítor.
5.1 Princípio Aditivo da Contagem 163

Solução. Denotemos por f1 , f2 , f3 e f4 os quatro lmes que estão em


cartaz e por t1 e t2 as duas peças de teatro. Agora, representemos pelo
par (i, j), com 0 ≤ i ≤ 4 e 0 ≤ j ≤ 2, o programa que consiste em as-
sistir ao lme fi e à peça tj (caso i = 0 ou j = 0 isso signica que não
será assistido a nenhum lme ou a nenhuma peça, respectivamente).
Pelas limitações econômicas do Pedro Vítor temos que ele só pode
escolher um programa dentro dos seguintes conjuntos disjuntos:
 
A1 = (1, 0), (2, 0), (3, 0), (4, 0) e A2 = (0, 1), (0, 2) .
Logo, no total são |A1 ∪ A2 | = |A1 | + |A2 | = 6 programas distintos,
entre os quais Pedro Vítor terá que escolher um.
Exemplo 5.2. Numa reunião havia um certo número de pessoas e
todos os presentes apertaram as mãos entre si. Sabendo-se que ao todo
foram feitos 66 cumprimentos, calcule o número de pessoas presentes
à reunião.
Solução. Vamos enumerar as pessoas com os números do conjunto
P = {1, 2, . . . , n}. A cada aperto de mão associaremos um par (i, j),
signicando que a pessoa i apertou a mão da pessoa j . Assim, os
apertos de mão envolvendo a pessoa 1 foram:

A1 = {(1, 2), (1, 3), . . . , (1, n)}.


Do mesmo modo, denimos os apertos de mão envolvendo a pessoa 2
que não envolvem a pessoa 1, como:
A2 = {(2, 3), (2, 4), . . . , (2, n)}.
Note que o aperto (2, 1) é o mesmo que o aperto (1, 2), já que se 1
aperta a mão de 2, então 2 aperta a mão de 1. Analogamente,
Ai = {(i, i + 1), (i, i + 2), . . . , (i, n)}, para 1 ≤ i ≤ n.
164 5 Contagem

Note que Ai ∩ Aj = ∅ para i 6= j . Observe também que todos os


apertos aparecem em um dos conjuntos Ai . Assim, A1 ∪ · · · ∪ An
contém todos os apertos de mão. Logo, pelo princípio aditivo:
|A1 ∪ A2 ∪ · · · ∪ An | = |A1 | + |A2 | + . . . |An |
= (n − 1) + (n − 2) + · · · + 2 + 1
(n − 1)n
= = 66.
2
Resolvendo em n, temos que n = 12.

Vimos que o princípio aditivo nos fornece o número de elementos


de qualquer união de conjuntos dois a dois disjuntos. Discutiremos
agora uma extensão do princípio para qualquer união de conjuntos,
não necessariamente dois a dois disjuntos.
Proposição 5.3. Sejam A1 e A2 dois conjuntos nitos quaisquer.
Então,
|A1 ∪ A2 | = |A1 | + |A2 | − |A1 ∩ A2 |.
Demonstração. Observe que
A1 ∪ A2 = (A1 − A2 ) ∪ A2

onde a união é dois a dois disjunta. Pelo princípio aditivo, temos que
|A1 ∪ A2 | = |A1 − A2 | + |A2 |. (5.1)

Analogamente, aplicando novamente este princípio, temos que

|A1 | = |A1 − A2 | + |A1 ∩ A2 |; (5.2)


A proposição segue imediatamente combinando as igualdades (5.1) e
(5.2).
5.1 Princípio Aditivo da Contagem 165

Para chegar a uma expressão análoga à do princípio aditivo, vamos


fazer mais um caso, considerando agora três conjuntos.

Corolário 5.4. Sejam A1 , A2 e A3 três conjuntos nitos quaisquer.


Então,

|A1 ∪ A2 ∪ A3 | =|A1 | + |A2 | + |A3 |



− |A1 ∩ A2 | + |A1 ∩ A3 | + |A2 ∩ A3 |
+ |A1 ∩ A2 ∩ A3 |.

Demonstração. Pela Proposição 5.3 temos que,

|A1 ∪ (A2 ∪ A3 )| = |A1 | + |A2 ∪ A3 | − |A1 ∩ (A2 ∪ A3 )|,

de onde,

|A1 ∪ A2 ∪ A3 | = |A1 | + |A2 ∪ A3 | − |(A1 ∩ A2 ) ∪ (A1 ∩ A3 )|.

Novamente, pela Proposição 5.3 temos que,

|A1 ∪ A2 ∪ A3 | = |A1 | + |A2 | + |A3 | − |A2 ∩ A3 | − |(A1 ∩ A2 ) ∪ (A1 ∩ A3 )|.

Aplicando mais uma vez a Proposição 5.3 temos que,

|(A1 ∩ A2 ) ∪ (A1 ∩ A3 )| = |A1 ∩ A2 | + |A1 ∩ A3 | − |(A1 ∩ A2 ) ∩ (A1 ∩ A3 ).

Combinando as duas últimas igualdades obtemos

|A1 ∪ A2 ∪ A3 | =|A1 | + |A2 | + |A3 |



− |A1 ∩ A2 | + |A1 ∩ A3 | + |A2 ∩ A3 |
+ |A1 ∩ A2 ∩ A3 | ,

como desejávamos.
166 5 Contagem

Para facilitar nossa escrita, vamos denotar por A1 A2 . . . Ak o con-


junto A1 ∩ A2 ∩ · · · ∩ Ak . Assim, outra forma de enunciar o Corolário
5.4 é a seguinte:

[3 X 3 X X

A i = |Ai | − |Ai1 Ai2 | + |Ai1 Ai2 Ai3 |.

i=1 i=1 1≤i1 <i2 ≤3 1≤i1 <i2 <i3 ≤3

De forma geral, dados os conjuntos nitos A1 , A2 , . . . , An , as ex-


pressões anteriores nos levam a denir os números:
n
X
S1 = |Ai |
i=1
X
S2 = |Ai1 Ai2 |,
1≤i1 <i2 ≤n
..
.
X
Sk = |Ai1 Ai2 . . . Aik |,
1≤i1 <i2 <···<ik ≤n
..
.
Sn = |A1 A2 . . . An |.

Assim, a versão mais geral do princípio aditivo, também conhecida


como princípio de inclusão e exclusão, é:

Princípio Aditivo - Versão Geral: Sejam A1 , A2 . . . , An


conjuntos nitos quaisquer. Então,

[n

Ai = S1 − S2 + S3 − S4 + · · · + (−1)n−1 Sn .

i=1

Não iremos provar essa versão, mas o leitor pode (e deve!) mostrá-la
como exercício, repetindo os argumentos anteriores.
5.1 Princípio Aditivo da Contagem 167

Exemplo 5.5. No Colégio Fantástico foram entrevistados 78 estudan-


tes. Destes, 32 estavam fazendo um curso de francês; 40 um curso de
física; 30 um curso de matemática; 23 um curso de história; 19 francês
e física; 13 francês e matemática; 15 física e matemática; 2 francês e
história; 15 física e história; 14 matemática e história; 8 francês, fí-
sica e matemática; 8 francês, física e história; 2 francês, matemática
e história; 6 física, matemática e história e 2 estavam fazendo todos
os quatro cursos. Quantos estudantes estavam fazendo pelo menos 1
curso nas 4 áreas mencionadas?

Solução. Denotemos por A1 , A2 , A3 , e A4 os conjuntos dos estudan-


tes que fazem francês, física, matemática e história, respectivamente.
Observemos que as igualdades

|A1 | = 32,
|A2 | = 40,
|A3 | = 30,
|A4 | = 23,

4
X
nos dão que S1 = |Ai | = 125; as igualdades
i=1

|A1 A2 | = 19,
|A1 A3 | = 13,
|A1 A4 | = 2,
|A2 A3 | = 15,
|A2 A4 | = 15,
|A3 A4 | = 14,
168 5 Contagem

X
nos dão que S2 = |Ai1 Ai2 | = 78; as igualdades
1≤i1 <i2 ≤4

|A1 A2 A3 | = 8,
|A1 A2 A4 | = 8,
|A1 A3 A4 | = 2,
|A2 A3 A4 | = 6,

X
nos dão que S3 = |Ai1 Ai2 Ai3 | = 24; assim como que S4 =
1≤i1 <i2 <i3 ≤4
|A1 A2 A3 A4 | = 2.
[ 4

Segue-se então, do princípio aditivo, que Ai = 125 − 78 + 24 −

i=1
2 = 69.

Denição 5.6. Denimos o complementar do conjunto A em relação


ao conjunto U como sendo um subconjunto de U dado por

Ac = x ∈ U; x ∈
/A .

Figura 5.1: A área branca corresponde a Ac e o conjunto U é representado


por todo o retângulo
5.1 Princípio Aditivo da Contagem 169

Neste caso é fácil vericar que os conjuntos A e Ac são disjuntos e


que U = A ∪ Ac . Segue-se do princípio aditivo que |U| = |A| + |Ac |;
portanto,
|Ac | = |U| − |A|.
Analogamente, dados dois conjuntos A1 ⊂ U e A2 ⊂ U , temos que
A1 ∪ A2 e (A1 ∪ A2 )c são disjuntos e, aliás, U = (A1 ∪ A2 ) ∪ (A1 ∪ A2 )c .
Novamente, pelo princípio aditivo, vale que
|U| = |A1 ∪ A2 | + |(A1 ∪ A2 )c |;

e consequentemente temos que


|(A1 ∪ A2 )c | = |U| − (|A1 | + |A2 |) + |A1 A2 |.

Similarmente, dados três conjuntos A1 ⊂ U, A2 ⊂ U e A3 ⊂ U


podemos demonstrar que
|(A1 ∪ A2 ∪ A3 )c | = |U| − (|A1 | + |A2 | + |A3 |)
+ (|A1 A2 | + |A1 A3 | + |A2 A3 |)
− |A1 A2 A3 |.
Então, usando a notação S0 = |U|, temos a seguinte proposição:
Proposição 5.7. Para toda família de subconjuntos Ai ⊂ U , i =
1, 2, . . . , n, vale a relação:
!c
[ n 

Ai = S0 − S1 − S2 + S3 − S4 − · · · + (−1)n−1 Sn

i=1

= S0 − S1 + S2 − S3 + S4 − · · · + (−1)n Sn ,
ou resumidamente,
n !c
[ n
X
c c c
Ai = |A1 A2 · · · An | = (−1)j Sj .

i=1 j=0
170 5 Contagem

Observação 5.8. Observemos que na última relação da proposição


usamos a conhecida Lei de DeMorgan: o complementar da união de
uma família nita de conjuntos, em relação a um conjunto U , é a
intersecção dos complementares de cada um deles.

5.2 Princípio Multiplicativo de Contagem

Começamos esta seção discutindo um problema relacionado com o


apaixonante jogo de xadrez. Ele consiste no seguinte: queremos saber
de quantas maneiras diferentes podemos colocar duas torres num tabu-
leiro de xadrez de forma tal que nenhuma ataque a outra. Uma situa-
ção como a que procuramos é mostrada na Figura 5.2, pois lembramos
que torres só se movimentam na direção horizontal ou na direção verti-
cal do tabuleiro. Antes de prosseguir deixamos claro o seguinte: se na
Figura 5.2 trocamos a posição da torre a com a torre b consideraremos
isto como uma situação diferente.

Figura 5.2: Torres que não se atacam

Notemos o seguinte: uma vez que coloquemos uma das torres numa
5.2 Princípio Multiplicativo de Contagem 171

casa do tabuleiro não podemos colocar a segunda torre na mesma


linha ou coluna em que esta se encontra, pois ela seria ameaçada.
Como cada linha e cada coluna contém 8 casas do tabuleiro, sendo
uma delas comum a ambas, então temos 15 posições proibidas para
colocar a segunda torre, ou seja, ela só pode ser colocada em 64−15 =
49 posições diferentes. Resumindo, por cada uma das 64 possíveis
posições para a torre a temos 49 possibilidades diferentes para colocar
a torre b, totalizando 64·49 = 3.136 formas diferentes de colocar ambas
as torres no tabuleiro sem que elas se ataquem.
O exemplo acima traz a essência do que é chamado princípio mul-
tiplicativo da contagem : se um evento A1 pode ocorrer de m maneiras
distintas e, se para cada uma dessas m maneiras possíveis de A1 ocor-
rer, um outro evento A2 pode ocorrer de n maneiras distintas, então o
número de maneiras de ocorrerem sucessivamente os eventos A1 e A2
é m · n.
Na linguagem matemática: relembramos que dados dois conjuntos
A1 e A2 , podemos construir um par ordenado (a1 , a2 ) tomando um
elemento a1 ∈ A1 , denominado o primeiro elemento do par, e um
elemento a2 ∈ A2 , denominado o segundo elemento do par. O conjunto
A1 × A2 é constituido por todos os pares ordenados construídos dessa
forma. Assim sendo, a versão mais simples do princípio multiplicativo
nos garante que

|A1 × A2 | = |A1 | |A2 |.

Uma extensão deste princípio para um número nito qualquer de


conjuntos é a seguinte:

princípio multiplicativo da contagem: Dados os conjuntos


172 5 Contagem

nitos A1 , A2 , . . . , An temos que

|A1 × A2 × · · · × An | = |A1 | · |A2 | · · · |An |.

Note que neste princípio, não é necessária nenhuma hipótese adi-


cional sobre os conjuntos Ai . Vamos agora dar alguns exemplos de
como aplicar esse princípio.

Exemplo 5.9. Em Maceió entraram em cartaz 4 lmes distintos e


2 peças de teatro. Se agora o Pedro Vítor tem dinheiro para assistir
exatamente a um lme e a uma peça de teatro, diga quantos são os
possíveis programas que Pedro Vítor pode fazer.

Solução. Denotemos por f1 , f2 , f3 e f4 os quatro lmes que estão em


cartaz e por t1 e t2 as duas peças de teatro. Denamos os conjuntos

A1 = {f1 , f2 , f3 , f4 } e A2 = {t1 , t2 }.

Neste caso, as condições econômicas do Pedro Vítor permitem que


ele escolha um elemento do conjunto A1 e outro elemento do conjunto
A2 . Este tipo de escolha representa-se pelo conjunto


A1 × A2 = (fi , tj ); 1 ≤ i ≤ 4 e 1 ≤ j ≤ 2 ,

onde cada par (fi , tj ) representa o programa que consiste em assistir


ao lme fi e à peça tj . Logo, no total são |A1 × A2 | = |A1 | · |A2 | = 8
programas distintos.

Exemplo 5.10. Se numa loja de doces existem 9 tipos distintos de


balas e 5 tipos distintos de chiclete, diga quantas escolhas podemos
fazer para comprar somente uma bala e um chiclete.
5.2 Princípio Multiplicativo de Contagem 173

Solução. Denotemos por b1 , b2 , b3 , b4 , b5 , b6 , b7 , b8 e b9 os nove tipos


distintos de balas e por c1 , c2 , c3 , c4 e c5 os cinco tipos distintos de
chicletes. Denamos os conjuntos

B = {b1 , b2 , b3 , b4 , b5 , b6 , b7 , b8 , b9 } e C = {c1 , c2 , c3 , c4 , c5 }.

Como precisamos comprar simultaneamente um elemento do conjunto


B e um elemento do conjunto C , então o conjunto B × C me dá o
conjunto de todas as escolhas possíveis. Logo, o número de escolhas
possíveis para comprar simultaneamente um tipo de bala e um tipo
de chiclete é |B × C| = 9 · 5 = 45.

Exemplo 5.11. De quantas maneiras 2 pessoas podem estacionar seus


carros numa garagem com 10 vagas?

Solução. Observando que a primeira pessoa pode estacionar seu carro


de 10 formas distintas e que a segunda pessoa pode estacionar seu
carro de 9 formas distintas, temos pelo princípio multiplicativo que
existem 9 · 10 = 90 formas possíveis nas quais duas pessoas podem
estacionar seus carros numa garagem com 10 vagas.

Exemplo 5.12. Dado o número 720, diga

(a) quantos divisores inteiros e positivos ele possui;

(b) entre seus divisores inteiros e positivos, quantos são pares;

(c) entre seus divisores inteiros e positivos, quantos são ímpares;

(d) dos divisores acima, quantos são quadrados perfeitos.


174 5 Contagem

Solução. Pelo teorema fundamental da aritmética, todo número in-


teiro positivo é primo ou produto de primos. Observe que a decom-
posição de 720 em fatores primos vem dada por:

720 = 24 · 32 · 51 . (5.3)

Agora denamos os seguintes conjuntos:

A ={todos os divisores de 720 que são da forma 2k , onde k ∈ Z+ },


B ={todos os divisores de 720 que são da forma 3m , onde m ∈ Z+ },
C ={todos os divisores de 720 que são da forma 5n , onde n ∈ Z+ }.

Observemos que 0 ≤ k ≤ 4, pois se k > 4 então pelo menos a potência


25 deveria estar presente em (5.3); como isto não acontece segue-se
que 0 ≤ k ≤ 4, de modo que

A = 20 , 21 , 22 , 23 , 24 ,
seguindo o mesmo raciocínio, podemos demonstrar que 0 ≤ m ≤ 2 e
que 0 ≤ n ≤ 1. Assim,
 
B = 30 , 31 , 32 e C = 50 , 51 .

(a) O conjunto de todos os possíveis divisores de 720 pode ser identi-


cado com o conjunto A×B ×C . De onde o número de divisores
inteiros e positivos de 720 é |A×B ×C|. Porém, o princípio mul-
tiplicativo nos garante que |A×B ×C| = |A|·|B|·|C|. Portanto,
o número de divisores inteiros e positivos de 720 é 5 × 3 × 2 = 30,
pois |A| = 5, |B| = 3 e |C| = 2.

(b) Para obter o conjunto de todos os divisores pares de 720 deve-


mos remover o elemento 20 do conjunto A. Assim, o conjunto de
5.2 Princípio Multiplicativo de Contagem 175

todos os divisores pares e positivos de 720 vem dado pelo con-


junto A − {20 } × B × C . O princípio multiplicativo nos garante

que A − {20 } × B × C = A − {20 } · |B| · |C|. Portanto, o
número de divisores pares e positivos de 720 é 4 × 3 × 2 = 24,

pois A − {20 } = 4, |B| = 3 e |C| = 2.

(c) Para obter o conjunto de todos os divisores ímpares de 720 deve-


mos remover os elementos 21 , 22 , 23 e 24 do conjunto A. Assim,
o conjunto de todos os divisores ímpares e positivos de 720 vem
dado pelo conjunto

A − {21 , 22 , 23 , 24 } × B × C.

O princípio multiplicativo nos garante que



A − {21 , 22 , 23 , 24 } × B × C = A − {21 , 22 , 23 , 24 } · |B| · |C|.

Portanto, o número de divisores ímpares e positivos de 720 é



1 × 3 × 2 = 6; pois A − {21 , 22 , 23 , 24 } = 1, |B| = 3 e |C| = 2.

(d) Para obter o conjunto de todos os divisores de 720 que são qua-
drados perfeitos devemos car com as potências pares nos con-
juntos A, B e C , respectivamente. Portanto, devemos remover
os elementos 21 , 23 do conjunto A. Também devemos remover o
elemento 31 do conjunto B . Finalmente do conjunto C devemos
remover o elemento 51 . Logo, o conjunto de todos os divisores
quadrados perfeitos e positivos de 720 vem dado pelo conjunto
  
D := A − {21 , 23 } × B − {31 } × C − {51 } .

O princípio multiplicativo nos garante que



D = A − {21 , 23 } · B − {31 } · C − {51 } .
176 5 Contagem

Portanto, o número de divisores quadrados perfeitos e positivos



de 720 é 3 · 2 · 1 = 6; pois A − {21 , 23 } = 3, B − {31 } = 2

e C − {31 } = 1. Observe que {1, 4, 9, 16, 36, 144} é o conjunto
dos divisores de 720 que são quadrados perfeitos.

Exemplo 5.13. Se um número natural n se fatora como

n = pk11 · pk22 · · · pkr r , (5.4)

onde os pi são números primos distintos e cada ki ∈ Z+ , então o


número de divisores positivos de n, denotado por d(n) é

d(n) = (k1 + 1)(k2 + 1) . . . (kr + 1).

Solução. Dena o conjunto

A1 ={todos os divisores de n que são da forma pm


1 , onde m ∈ Z },
1 +

e em geral, dena

Ai ={ todos os divisores de n que são da forma pm


i , onde t ∈ Z }.
i +

Observemos que mi ≤ pi , pois se mi > pi , então pelo menos a potência


pki i +1 deveria estar presente em (5.4);como isto não acontece segue-se
que mi ≤ pi , de modo que

Ai = p0i , p1i , p2i , . . . , pki i , para i = 1, 2, 3, . . . , ki .

É imediato ver que Ai = ki + 1.
5.2 Princípio Multiplicativo de Contagem 177

O conjunto de todos os possíveis divisores de n vem dado pelo


conjunto A1 × A2 × · · · × Ar , de onde se conclui que o número de
divisores inteiros e positivos de n é
d(n) = |A1 × A2 × · · · × Ar | = |A1 | · |A2 | · · · |Ar |,
onde na última igualdade usamos o princípio multiplicativo. Portanto,
o número de divisores inteiros e positivos de n é
d(n) = (k1 + 1)(k2 + 1) · · · (kr + 1).

Exemplo 5.14. De quantas maneiras podemos escolher dois inteiros


de 1 a 20 de forma que a soma seja ímpar?
Solução. Observemos que
• a soma de dois números inteiros pares é um número par. Com
efeito, para quaisquer a, b ∈ Z temos que 2a + 2b = 2(a + b);
• a soma de dois números inteiros ímpares é um número par. Com
efeito, para quaisquer a, b ∈ Z temos que (2a + 1) + (2b + 1) =
2(a + b + 1);

• a soma de um número inteiro par com qualquer outro inteiro


ímpar sempre é um inteiro ímpar. Com efeito, para quaisquer
a, b ∈ Z temos que 2a + (2b + 1) = 2(a + b) + 1.
Isto nos sugere denir os conjuntos
P = {2, 4, 6, 8, 10, 12, 14, 16, 18, 20},
I = {1, 3, 5, 7, 9, 11, 13, 15, 17, 19},
onde P × I são todas as formas possíveis de somar um número inteiro
par com outro ímpar. O princípio multiplicativo nos garante que nossa
resposta é |P × I| = |P | · |I| = 100, pois |P | = |I| = 10.
178 5 Contagem

5.3 Uso Simultâneo dos Princípios Aditivo

e Multiplicativo

Aproveitamos esta seção para apresentar problemas um pouco mais


difíceis que os tratados nas seções anteriores. Nestes problemas, pre-
cisaremos empregar simultaneamente o Princípio Aditivo e o princípio
multiplicativo. Vamos ao primeiro deles:

Exemplo 5.15. Sabemos que no início da premiação da 1a fase da


Olimpíada Alagoana de Matemática existem 10 livros diferentes de
Álgebra, 7 livros diferentes de combinatória e 5 livros diferentes de
geometria para homenagear os vencedores. Danielle é a primeira a
pegar o prêmio que consiste em 2 livros, com a condição de que estes
não podem ser da mesma matéria. Diga quantas escolhas Danielle
pode fazer para pegar seu prêmio.

Solução. Denotemos por

A = {a1 , . . . , a10 }, C = {c1 , . . . , c7 } e G = {g1 , . . . , g5 },

os conjuntos de livros de álgebra, combinatória e geometria, respecti-


vamente. Observemos que |A| = 10, |C| = 7 e |G| = 5 e Danielle tem
as seguintes possibilidades de escolha:

• escolher um livro de A e um livro de C . Neste caso, Danielle tem


|A × C| = |A| · |C| = 70 escolhas possíveis (devido ao princípio
multiplicativo).

• escolher um livro de A e um livro de G . Neste caso, Danielle tem


|A × G| = |A| · |G| = 50 escolhas possíveis (devido ao princípio
multiplicativo) ou
5.3 Uso Simultâneo dos Princípios Aditivo e Multiplicativo 179

• escolher um livro de C e um livro de G . Neste caso, Danielle tem


|C × G| = |C| · |G| = 35 escolhas possíveis (devido ao princípio
multiplicativo).
Agora o Princípio Aditivo nos garante que o número total de escolhas
que Danielle pode fazer é 70 + 50 + 35 = 155.

Exemplo 5.16. Há 18 moças e 12 rapazes, onde 5 deles são irmãos


(3 moças e 2 rapazes) e os restantes não possuem parentesco. Diga
quantos casamentos são possíveis naquela turma (sabendo que irmãos
não se casam).
Solução. Observemos que 15, entre as 18 moças, não têm parentesco
nenhum com os 12 rapazes, logo, pelo princípio multiplicativo temos
que é possível efetuar 15 · 12 = 180 casamentos diferentes entre eles.
Por outro lado, as 3 moças restantes podem efetuar casamento com 10
dos 12 rapazes, pois 2 deles são seus irmãos. Novamente, pelo princípio
multiplicativo é possível realizar 3·10 = 30 casamentos diferentes neste
caso. Finalmente, o Princípio Aditivo nos dá que podem ser realizados
um total de 180 + 30 = 210 casamentos.
Exemplo 5.17. Quantas palavras de 5 caracteres podem ser formadas
com as letras α, β e γ de modo que em cada palavra não falte nenhuma
dessas letras?
Solução. Denamos os seguintes conjuntos,
U ={palavras de 5 caracteres só com as letras α, β e γ};
Aα ={palavras que estão em U e onde não aparece a letra α};
Aβ ={palavras que estão em U e onde não aparece a letra β};
Aγ ={palavras que estão em U e onde não aparece a letra γ}.
180 5 Contagem

Por exemplo,
• a palavra γγγγγ ∈ Aα ∩ Aβ ;

• a palavra γααγα ∈ Aβ ;

• a palavra βαβββ ∈ Aγ .

Primeiramente, notemos que cada caracter de U pode ser escolhido


de 3 formas distintas. Segue-se então do Princípio Multiplicativo que
existem 35 formas de escrever uma palavra de 5 caracteres usando um
alfabeto de 3 letras, isto é,

S0 = |U| = 35 = 243.

Calculemos agora |Aα |, isto é, o número de palavras onde não apa-


rece a letra α. Para isto, observemos que cada caractere em Aα pode
ser escolhido de 2 formas. Logo, o princípio multiplicativo nos garante
que existem 25 palavras em Aα , ou seja, |Aα | = 25 . Analogamente,
podemos mostrar que |Aβ | = |Aγ | = 25 . Portanto,

S1 = |Aα | + |Aβ | + |Aγ | = 25 + 25 + 25 = 96.

Prosseguimos com o cálculo de |Aα Aβ |, isto é, do número de pala-


vras onde não aparecem as letras α e β ; portanto, cada caractere em
Aα Aβ pode ser escolhido de 1 forma. Logo, o princípio multiplicativo
nos garante que existe 15 = 1 palavra em Aα Aβ , ou seja, |Aα Aβ | = 1.
Similarmente, podemos mostrar que |Aα Aγ | = |Aβ Aγ | = 1. Portanto,

S2 = |Aα | + |Aβ | + |Aγ | = 3.

Por m, achamos |Aα Aβ Aγ |, que nos dá o número de palavras onde


não aparecem as letras α, β e γ ; mas cada palavra em Aα Aβ Aγ tem
5.4 Permutações Simples 181

que usar pelo menos um dos caracteres proibidos. Logo,

S3 = |Aα Aβ Aγ | = 0.

Finalmente, observamos que o conjunto das palavras de 5 caracte-


res que podem ser formadas com as letras α, β e γ de modo que em
cada palavra não falte nenhuma dessas letras é exatamente o conjunto
Acα Acβ Acγ . Usando a Proposição 5.7, temos:

|Acα Acβ Acγ | =S0 − S1 + S2 − S3


=243 − 96 + 3 − 0
=150.

5.4 Permutações Simples

Denimos o fatorial n! de um inteiro positivo n

n! = n · (n − 1) · (n − 2) · · · 2 · 1

se n > 0 e 0! = 1, por convenção. Observe que o fatorial cresce muito


rapidamente quando n cresce. Por exemplo, para os 10 primeiros
valores de n

1!=1 2!=2 3!=6 4!=24 5!=120


6!=720 7!=5.040 8!=40.320 9!=362.880 10!=3.628.800

Denição 5.18. Uma permutação simples de n objetos distintos é


qualquer agrupamento ordenado desses n objetos. Denotaremos por
Pn o número de todas as permutações simples de n objetos dados.
182 5 Contagem

Por exemplo, todas as permutações dos 3 elementos do conjunto


A = {a1 , a2 , a3 } são:

σ1 = (a1 , a2 , a3 ),
σ2 = (a1 , a3 , a2 ),
σ3 = (a2 , a1 , a3 ),
σ4 = (a2 , a3 , a1 ),
σ5 = (a3 , a1 , a2 ),
σ6 = (a3 , a2 , a1 ).

Proposição 5.19. Seja n ≥ 1. O número total de permutações sim-


ples de n objetos O = {o1 , o2 , . . . , on } é dado por Pn = n!
Demonstração. É claro que a fórmula vale para n = 1. Vejamos agora
que existe a seguinte relação entre Pn e Pn−1 para n ≥ 2:

Pn = nPn−1 . (5.5)

Para comprovar isto, para cada i denamos Ai como sendo as permu-


tações dos n − 1 objetos {o1 , . . . , oi−1 , oi+1 , . . . , on }. Note que |Ai | =
Pn−1 , para cada i = 1, 2, . . . , n. Assim, para obtermos uma permu-
tação dos n objetos, basta que xemos o objeto inicial oi e tomemos
um elemento do conjunto Ai , que é uma permutação dos n − 1 objetos
restantes. Pelo princípio aditivo, temos que:

Pn = |A1 | + |A2 | + · · · + |An | = nPn−1 .

Como a equação (5.5) é válida para todo n ≥ 2, podemos aplicá-la


para n − 1, obtendo:

Pn−1 = (n − 1)Pn−2 ,
5.4 Permutações Simples 183

de onde vem que


Pn = n(n − 1)Pn−2 .
Repetindo este argumento, obtemos que
Pn = n(n − 1)(n − 2) · · · 3 · 2 · 1 = n!,

como queríamos demonstrar.


Exemplo 5.20. De quantas maneiras podemos formar uma la com
4 pessoas?
Demonstração. Observe que se enumeramos os lugares da la e enu-
meramos as pessoas, pa , pb , pc , pd , cada distribuição vai corresponder a
uma permutação do conjunto {1, 2, 3, 4}. Por exemplo, a distribuição
(pc , pa , pb , pd ) corresponde à permutação (3, 1, 2, 4). Assim, o número
de distribuições na la é 4! = 24.
Exemplo 5.21. De quantas maneiras k moças e k rapazes podem
formar pares para uma dança?
Solução. Estando as moças em uma la e os rapazes em outra, pode-
mos enumerá-los com números de 1, 2, . . . , k . A uma permutação des-
ses números, digamos (a1 , a2 , . . . , ak ) com ai ∈ {1, 2, . . . , k} faremos
uma associação da mulher i com o rapaz ai . Por exemplo, a permu-
tação (2, 1, 3, . . . , k) signica que a moça 1 dançará com o rapaz 2, a
moça 2 com o rapaz 1, e a moça i com o rapaz i, para i ≥ 3.
Observe que toda associação de k moças e k rapazes produz uma
permutação, de modo que o número de associações possíveis das mo-
ças com os rapazes é igual ao número de permutações dos elementos
do conjunto {1, 2, 3, . . . , k}. Pela Proposição 5.19 existem k! modos
diferentes de combinar as moças com os rapazes.
184 5 Contagem

5.5 Arranjos Simples

Denição 5.22. Consideremos n objetos e p um inteiro positivo tal


que 0 < p ≤ n. Um arranjo simples de classe p dos n objetos dados
é uma seleção de p objetos distintos dentre estes que diferem entre si
pela ordem de colocação ou pela natureza de cada um, isto é, o que
importa é quem participa ou o lugar que ocupa. Denotaremos por Apn
o número de arranjos simples de classe p de n objetos.

Por exemplo, dados os objetos o1 , o2 e o3 todos os arranjos possíveis


de classe 2 são: A1 = (o1 , o2 ), A2 = (o2 , o1 ), A3 = (o1 , o3 ), A4 =
(o3 , o1 ), A5 = (o2 , o3 ) e A6 = (o3 , o2 ).

Observação 5.23. Notemos que um arranjo simples de classe n de n


objetos dados não é mais que uma permutação desses n objetos. Logo,
Pn = Ann = n!.

Proposição 5.24. Seja n ≥ 1. O número total de arranjos simples


de classe p de n objetos O = {o1 , o2 , . . . , on } é dado por Apn = n!
(n−p)!
.

Demonstração. Para n = 1 a fórmula é obviamente válida. Similar-


mente ao caso das permutações, primeiramente provaremos que para
n ≥ 2 vale a seguinte igualdade:

Apn = nAp−1
n−1 . (5.6)

Agora denimos os conjuntos Ai como sendo os arranjos simples de


classe p − 1 dos n − 1 objetos {o1 , . . . , oi−1 , oi+1 , . . . , on }. Note que
n−1 , para cada i = 1, 2, . . . , n. Assim, para obtermos um
|Ai | = Ap−1
arranjo simples de classe p dos n objetos, basta que xemos o objeto
inicial oi e tomemos um elemento do conjunto Ai , que é uma arranjo
5.5 Arranjos Simples 185

de classe p − 1 dos n − 1 objetos restantes. Pelo princípio aditivo,


temos que:

Apn = |A1 | + |A2 | + · · · + |An | = nAp−1


n−1 .

Como nossa equação (5.6) é válida para todo n ≥ 2, podemos aplicá-la


para n − 1, obtendo:

Ap−1 p−2
n−1 = (n − 1)An−2 ,

de onde vem que


Apn = n(n − 1)Ap−2
n−2 .

Repetindo este argumento sucessivamente, obtemos que

p−(p−1)
Apn = n(n − 1)(n − 2) · · · (n − (p − 2))An−(p−1)
= n(n − 1)(n − 2) · · · (n − p + 2)A1n−p+1 .

Notemos agora que A1n−p+1 = n − p + 1; logo, da igualdade anterior


segue-se que

Apn = n(n − 1)(n − 2) · · · (n − p + 2)(n − p + 1)


n(n − 1)(n − 2) · · · (n − p + 2)(n − p + 1) × (n − p) · · · 1
=
(n − p) · · · 1
n!
= ,
(n − p)!

como desejávamos.

Agora vamos dar alguns exemplos de como aparecem problemas


práticos que requerem fazer este tipo de cálculo. O primeiro dele tem
186 5 Contagem

a ver com a formação de palavras diferentes com um conjunto dado


de letras.
Um anagrama de uma palavra é uma permutação de letras dessa
palavra para formar outra, a qual pode carecer de signicado. Por
exemplo:
• um anagrama de amor é roma;

• um anagrama de celia é alice;

• um anagrama de caterina é natercia;

• um anagrama de elvis é lives.


Exemplo 5.25. Quantos anagramas de p letras distintas podemos
formar com um alfabeto de 23 letras, sendo p < 23?
Solução. Como as letras são diferentes, nosso problema consiste em
achar todos os arranjos de classe p de 23 objetos dados, que neste caso
são as 23 letras do alfabeto. Logo, este número é
23!
Ak23 = .
(23 − k)!

Exemplo 5.26. De quantos modos 2 pessoas podem se sentar em 5


cadeiras que estão em la?
Solução. Este problema é equivalente a achar o número total de ar-
ranjos de classe 2 de 5 objetos, correspondendo as 5 cadeiras aos 5
objetos e as duas pessoas indicando a ordem do arranjo. Logo, este
número é dado por
5!
A25 = = 20.
3!
5.5 Arranjos Simples 187

Exemplo 5.27. Considere os dígitos 2, 3, 4, 5, 7 e 9. Supondo que a


repetição de dígitos não seja permitida, responda às seguintes pergun-
tas:

(a) Quantos números de três dígitos podem ser formados?

(b) Entre os achados em (a) quantos são pares?

(c) Entre os achados em (a) quantos são ímpares?

(d) Entre os achados em (a) quantos são múltiplos de 5?

(e) Entre os achados em (a) quantos são menores do que 400?

Solução. Seja O = {2, 3, 4, 5, 7, 9} nosso conjunto de objetos.

(a) A quantidade de números de três dígitos que podemos formar


sem repetição de algum deles é claramente o número de arranjos
de classe 3 dos 6 dígitos de O, isto é,
6!
A36 = = 120.
3!

(b) Sabemos que em todo número par o último dígito é um múltiplo


de 2, isto é, ele acaba em 0, 2, 4, 6 ou 8. Então, em nosso caso
as únicas possibilidades são que o número termine em 2 ou 4.
Supondo que o último dígito seja 2, temos que preencher as duas
casas restantes com os dígitos pertencentes ao conjunto O − {2}.
Assim, existem A2|O−{2}| = A25 = 5! 3!
= 20 números dos achados
em (a) que nalizam em 2. De forma análoga, existem A2|O−{4}| =
5!
A25 = 3! = 20 números dos achados em (a) que nalizam em 4.
Logo, entre os números achados em (a) existem 20 + 20 = 40
números pares.
188 5 Contagem

(c) Todo conjunto de números pode ser dividido em duas classes


disjuntas: a classe dos números pares e a classe dos números ím-
pares que pertencem ao mesmo. Segue-se que dentre os números
achados em (a) existem 120 − 40 = 80 números ímpares.

(d) Todo número múltiplo de 5 acaba em 0 ou 5; no nosso caso te-


mos que a única possibilidade para o último dígito é 5. Assim
o problema consiste em preencher as duas casas restantes com
dígitos do conjunto O − {5}. De onde se segue que a quanti-
dade de números múltiplos de 5 existentes em (a) vem dada por
5!
A2|O−{5}| = A25 = 3! = 20.

(e) Para obter os números menores do que 400 a casa das centenas
só poderá ser ocupada pelos dígitos 1, 2 ou 3. Como 1 ∈ / O,
temos que as únicas possibilidades em nosso caso são 2 ou 3.
Então, supondo que o primeiro dígito do número seja 2, devemos
preencher duas casas restantes com os dígitos pertencentes a
O − {2}. De forma análoga, existem A2|O−{3}| = A25 = 5! 3!
= 20
números dos achados em (a) e que começam com 3. Logo, dentre
os números achados em (a) existem 20 + 20 = 40 menores do que
400.

5.6 Combinações Simples

O conceito de combinação simples surge naturalmente quando tenta-


mos responder à seguinte pergunta:de quantas formas diferentes pode-
mos selecionar p objetos dentro de n objetos dados?
5.6 Combinações Simples 189

Por exemplo, suponha que queremos enfeitar uma festa de aniver-


sário com bolas de dois tipos de cores e na loja onde as compraremos
existem bolas nas cores azul, verde e vermelha. De quantas formas
distintas podemos enfeitar nossa festa? É claro que podemos enfeitar
a festa de 3 formas diferentes: com bolas em azul e verde; com bolas
em azul e vermelho ou com bolas em verde e vermelho.
Notemos que, ao contrário do caso em que trabalhamos com arran-
jos, quando fazemos uma seleção de duas cores não estamos inte-
ressados na ordem em que elas foram escolhidas.

Denição 5.28. Consideremos n objetos e p um inteiro positivo tal


que 0 < p ≤ n. Uma combinação simples de classe p dos n objetos
dados é uma seleção de p objetos distintos entre estes que diferem
entre si apenas pela natureza de cada um, isto é, o que importa é
simplesmente quem participa no grupo selecionado. Denotaremos por

n
p
o número de combinações simples de classe p de n objetos.

Proposição 5.29. Seja n ≥ 1. O número total de combinações


simples de classe p de n objetos O = {o1 , o2 , . . . , on } é dado por
n
 n!
p
= p!(n−p)!
.

Demonstração. Veremos a seguir que arranjos simples e combinações


simples de classe p estão estreitamente relacionados. Com efeito, para
cada combinação simples formada por p objetos distintos de O pode-
mos gerar todos os arranjos simples de classe p formados por estes p
objetos. Basta para isto fazer todas as suas permutações possíveis.
Obtém-se assim p ! arranjos simples diferentes com esses p objetos.
Resumindo, para cada combinação simples de classe p formada com
p objetos diferentes de O podemos fazer p ! arranjos simples diferen-
tes de classe p com estes mesmos objetos; logo, no total, teremos a
190 5 Contagem

seguinte relação:  
n n!
p! = Apn = ,
p (n − p)!
de onde segue-se que
 
n n!
= .
p p!(n − p)!

Exemplo 5.30. De quantas formas diferentes podemos construir uma


palavra de tamanho n com i letras a e n − i letras b?

Solução. A solução do problema equivale em escolher a posição das


i letras a em questão, uma vez que a posição das (n − i) letras b
restantes estará determinada. Se enumeramos as posições das letras
de 1 a n, uma palavra será formada ao xarmos a posição das i letras

a. Isso é exatamente ni , já que corresponde ao número de grupos
com i elementos (posições com letra a) tomados em um conjunto de n
elementos (todas as posições), que diferem somente por sua natureza.

Exemplo 5.31. De quantas formas podemos dividir um grupo 5 pes-


soas em um grupo de duas e outro de três?

Solução. Temos 52 = 25!3! ! = 10 formas diferentes de escolher duas
pessoas do grupo. Por cada uma dessas escolhas o outro grupo de três
pessoas é automaticamente determinado; logo, temos 10 possibilidades
diferentes de fazer a divisão.

Exemplo 5.32. De quantos modos podemos dividir 6 pessoas em:


(a) Dois grupos de 3 pessoas cada?
5.6 Combinações Simples 191

(b) Três grupos de 2 pessoas cada?


Solução. Começamos por (a). À primeira vista, parece que a resposta

deve ser n3 = 3!6!3! = 20, similarmente ao exemplo anterior. Porém,
aqui há um problema devido ao fato de estarmos dividindo em grupos
que têm a mesma quantidade de pessoas e, portanto, as permutações
de cada dois grupos formados são consideradas divisões iguais; logo,
devemos dividir o resultado por 2 !, obtendo assim 10 formas diferentes
de obter dois grupos com 3 pessoas cada.
Para resolver o item (b) seguimos os seguintes passos:
• Primeiramente calcularemos o número de formas possíveis para
dividir 6 pessoas em um grupo de 2 e outro grupo de 4; esta

quantidade vem dada por 62 = 4!6!2! .

• Agora dividiremos as 4 pessoas restantes em um grupo de 2 e



outro grupo de 2; esta quantidade vem dada por 42 = 2!4!2! .
 
Pelo princípio multiplicativo temos que existem 62 42 = (2!)
6!
3 possi-

bilidades de dividir 6 pessoas em 3 grupos com duas pessoas cada.


Igualmente ao caso anterior, aqui as permutações possíveis de cada 3
grupos formados são consideradas iguais; logo, devemos dividir este
último resultado por 3 !. Portanto, existem 15 formas diferentes de
dividir 6 pessoas em três grupos de 2 pessoas cada.
Exemplo 5.33. Se você possui 10 amigos, de quantas maneiras você
pode escolher dois ou mais deles para jantar?
Solução. Esquematizamos a solução da seguinte maneira:
• Primeiramente, vamos encontrar a quantidade de maneiras pelas

quais você pode jantar com 2 amigos; isto é feito de
10
2
formas
diferentes.
192 5 Contagem

• Depois, vamos encontrar a quantidade de maneiras pelas quais



você pode jantar com 3 amigos; isto é feito de
10
3
formas dife-
rentes.

• Em seguida, encontramos a quantidade de maneiras pelas quais



você pode jantar com 4 amigos; isto é feito de
10
4
formas dife-
rentes.

• Em geral, o número de maneiras diferentes que você tem de



jantar com p amigos é dado por 10p .

Pelo princípio aditivo, temos que a quantidade de formas diferentes


que você tem de jantar com 2 ou mais de seus amigos, é dada por
       
10 10 10 10
+ + ··· + + = 1013,
2 3 9 10

sendo este o número procurado.

Exemplo 5.34. De um grupo de 10 pessoas das quais 4 são mulheres,


quantas comissões de 5 pessoas podem ser formadas de modo que pelo
menos uma mulher faça parte?

Solução. Sendo que o grupo tem 10 pessoas e 4 destas são mulheres,


segue-se que no grupo temos 6 homens. Para formar um grupo de 5
pessoas com pelo menos uma mulher, temos as seguintes alternativas:

• Nosso grupo é composto por uma mulher e 4 homens; neste caso


 
poderemos formar 41 64 = 60 comissões de 5 pessoas.

• Nosso grupo é composto por 2 mulheres e 3 homens; neste caso


 
poderemos formar 42 63 = 120 comissões de 5 pessoas.
5.7 O Binômio de Newton 193

• Nosso grupo é composto por 3 mulheres e 2 homens; neste caso


 
poderemos formar 43 62 = 60 comissões de 5 pessoas.

• Nosso grupo é composto por 4 mulheres e um homem; neste caso


 
poderemos formar 44 61 = 6 comissões de 5 pessoas.

Pelo princípio aditivo temos que é possível formar 246 comissões de 5


pessoas de modo que pelo menos uma mulher faça parte.

5.7 O Binômio de Newton

Nesta seção, estudaremos uma fórmula que generaliza a conhecida


expressão
(a + b)2 = a2 + 2ab + b2 .

Essa fórmula é conhecida como o binômio de Newton ou fórmula bino-


mial de Newton, devido ao Matemático Isaac Newton (1642-1727). A
fórmula binomial de Newton pode ser motivada pelas seguintes igual-
dades que são fáceis de vericar:

   
1 1 1
(a + b) = a + b = a+ b,
0 1
     
2 2 2 2 2 2 2 2
(a + b) = a + 2ab + c = a + ab + b,
0 1 2
       
3 3 2 2 3 3 3 3 2 3 2 3 3
(a + b) = a + 3a b + 3ab + c = a + a b+ ab + b.
0 1 2 3

Os casos particulares acima podem ser estendidos para qualquer po-


tência inteira positiva de a + b, ou seja, vale o seguinte resultado:
194 5 Contagem

Teorema 5.35 (Fórmula Binomial de Newton). Sejam a e b números


reais e n ∈ N, então
         
n n n n n−1 1 n n−i i n 1 n−1 n n
(a+b) = a + a b +· · ·+ a b +· · ·+ a b + b .
0 1 i n−1 n

Os números ni , 0 ≤ i ≤ n, são chamados também de coecientes bino-
miais.
Demonstração. Expandimos o binômio no produto de seus n fatores,
isto é,

(a + b)n = (a + b)(a + b) · · · (a + b) . (5.7)


| {z }
n−fatores

Se desenvolveremos o produto destes n fatores iguais acima obtemos


uma soma nita de termos da forma a1 a2 · · · an , onde cada aj , 1 ≤
j ≤ n, toma valor a ou b. Notemos que em cada termo se o número
b aparece i vezes, então o número a aparecerá (n − i) vezes, ou seja,
quando cada termo for multiplicado deverá tomar valor igual a an−i bi ,
para algum 1 ≤ i ≤ n. Por exemplo, os n termos
abb · · · b = abn , bab · · · b = abn , ..., bbb · · · ba = abn

têm o mesmo valor . Assim, para calcular o coeciente do termo ai bn−i


que aparece na equação (5.7), basta responder à seguinte pergunta: de
quantos modos podemos formar uma palavra com i letras a e (n − i)
letras b? A resposta dessa pergunta foi estudada no Exemplo 5.30 e é

simplesmente ni . Logo, a expressão na equação (5.7) é
       
n n
n n n−1 1 n 1 n−1 n n
(a + b) = a + a b + ··· + ab + b ,
0 1 n−1 n
o que prova o teorema.
5.8 Contagem e Probabilidades 195

A fórmula binomial de Newton nos dá algumas propriedades interes-


santes dos coecientes binomiais que resumimos na próxima proposi-
ção.
Proposição 5.36. Seja n ∈ N. As seguintes igualdades são válidas:
    
(a) n
0
+ n
1
+ ··· + n n
+ · · · + n−1
i
+ nn = 2n ;
   
(b) n0 − n1 + · · · + (−1)i ni + · · · + (−1)n nn = 0.
Demonstração. Para a letra (a), basta tomar a = b = 1 e expanda
2n = (1 + 1)n no Binômio de Newton. Para a letra (b), tome a = 1
e b = −1 e expanda 0 = (1 − 1)n no binômio de Newton, observando
que (−1)n é igual a 1 se n é par, e igual a 1 se n é ímpar.

5.8 Contagem e Probabilidades

Uma das aplicações interessantes da contagem de elementos de um


conjunto é quando desejamos estudar a probabilidade de eventos alea-
tórios. Por exemplo, se lançarmos um dado de seis faces, temos os
seguintes resultados possíveis:

Ω = {1, 2, . . . , 6}.

Se desejamos saber qual é a chance de que ocorra um número


primo no lançamento, devemos contar quantos primos aparecem em
{1, 2, 3, 4, 5, 6} e dividir por 6. Ou seja, a chance de ocorrer um número
primo num lançamento de um dado de seis faces é 3/6 = 0, 5.
Denimos a probabilidade de um subconjunto A ⊂ Ω como o nú-
mero
|A|
p(A) = .
|Ω|
196 5 Contagem

Também chamamos o subconjunto Ω de todos os resultados possíveis


de espaço amostral e um subconjunto A de Ω de evento. Por exemplo,
podemos calcular a probabilidade de escolhermos um número par no
conjunto 1, 2, 3, . . . , 15. Neste caso, o conjunto Ω está claro e é igual a
Ω = {1, 2, 3, . . . , 15}. O conjunto A é A = {2, 4, 6, . . . , 14}. Logo,
|A| 7
p(A) = = .
|Ω| 15
Assim, ca claro que a maior diculdade para calcular a proba-
bilidade de um evento é contar quantos elementos pertencem a este
evento e quantos elementos pertencem ao espaço amostral. A seguir,
veremos um exemplo mais elaborado onde aplicamos a noção de ar-
ranjo simples.

Exemplo 5.37. Calcular a probabilidade de que escolhendo um grupo


de 44 pessoas, existam pelo menos duas que fazem aniversário no
mesmo dia do ano.
Solução. Podemos reescrever isso do seguinte modo: num saco existem
bolas enumeradas com os números 1, 2, . . . , 365 (correspondentes aos
dias do ano). Retiramos a bola b1 e anotamos o número que apareceu.
Devolvemos a bola ao saco e efetuamos uma nova retirada, anotando
novamente o número que aparece. Repetindo este processo 44 vezes,
obtemos uma lista com 44 números. Assim, a pergunta se transforma
em: de quantos modos diferentes podemos escolher 44 bolas enume-
radas com os números 1, 2, 3, . . . , 365 com reposição, tal que existam
pelo menos duas bolas com o mesmo número?
A primeira coisa que devemos fazer é calcular o espaço amostral,
de todas as possibilidades possíveis de resultado. Como escolhemos
44 bolas enumeradas num saco, cada resultado possível é uma lista
5.9 Exercícios Propostos 197

(n1 , n2 , . . . , n44 ) com 44 números. Observe que, pelo princípio multi-


plicativo, |Ω| = 36544 , pois temos 365 opções para escolher n1 , 365
opções para escolher n2 , etc.
A segunda pergunta trata-se de saber quantos resultados são favo-
ráveis, ou seja, quantas são as escolhas tais que existam pelo menos
duas bolas com o mesmo número. Para isso é mais fácil contar quantas
escolhas existem tais que os 44 números são diferentes. Neste caso,
devemos escolher uma ordenação de 44 números distintos entre 365.
Isso corresponde à quantidade de arranjos de classe 44 num grupo de
365 elementos. Assim, concluímos que a probabilidade de que este
evento ocorra é
365!
36544 − A44
365 (365!−44!)
p= =1− .
36544 36544
Obter um valor aproximado para o número acima com o computador é
uma tarefa fácil nos dias atuais. Porém, aproximar expressões envolvendo
fatoriais (sem o uso do computador) é um fato conhecido há muito tempo
pela humanidade, através da famosa fórmula de Stirling.1 Com a ajuda
desta fórmula, obtemos que p é aproximadamente p ∼= 0.93, como havíamos
prometido no Capítulo 1.

Além dos exercícios abaixo, recomendamos a leitura de [9]. Lá, o


leitor encontrará material adicional sobre análise combinatória, bem
como uma ampla variedade de problemas.

5.9 Exercícios Propostos

1. De quantas maneiras podemos escolher três números distintos do


conjunto I50 = {1, 2, 3, . . . , 49, 50} de modo que sua soma seja
1 Grosseiramente, a fórmula de Stirling diz que o quociente entre n! e

2πnn en −n
está próximo de 1, para valores de n grandes.
198 5 Contagem

a) um múltiplo de 3?
b) um número par?
2. Considere o conjunto In = {1, 2, 3, . . . , n−1, n}. Diga de quantos
modos é possível formar subconjuntos de k elementos nos quais
não haja números consecutivos?
3. Considere as letras da palavra PERMUTA. Quantos anagramas
de 4 letras podem ser formados, onde:
a) não há restrições quanto ao número de consoantes ou vogais?
b) o anagrama começa e termina por vogal?
c) a letra R aparece?
d) a letra T aparece e o anagrama termina por vogal?
4. Calcular a soma de todos os números de 5 algarismos distintos
formados com os algarismos 1, 3, 5, 7 e 9.
5. Quantos números podem ser formados pela multiplicação de al-
guns ou de todos os números 2, 2, 3, 3, 3, 5, 5, 6, 8, 9, 9?
6. Entre todos os números de sete dígitos, diga quantos possuem
exatamente três dígitos 9 e os quatro dígitos restantes todos
diferentes?
7. De quantas maneiras podemos distribuir 22 livros diferentes en-
tre 5 alunos se 2 deles recebem 5 livros cada e os outros 3 recebem
4 livros cada?

8. Quantos são os números naturais de sete dígitos nos quais o


dígito 4 gura exatamente 3 vezes e o dígito 8 gura exatamente
2 vezes?
5.9 Exercícios Propostos 199

9. De quantas maneiras uma comissão de 4 pessoas pode ser for-


mada, de um grupo de 6 homens e 6 mulheres, se a mesma é
composta de um número maior de homens do que de mulheres?

10. O comprimento de uma palavra é a quantidade de caracteres que


ela possui. Encontre a quantidade de palavras de comprimento
5 que podemos formar fazendo uso de 10 caracteres distintos, de
forma que não existam três caracteres consecutivos idênticos em
cada palavra.

11. Quantos números inteiros existem entre 1 e 10.000 que não são
divisíveis por 3, 5 e 7?

12. Quantas são as permutações da palavra PROPOR nas quais não


existem letras consecutivas iguais?

13. De quantos modos 6 casais podem sentar-se ao redor de uma


mesa circular de tal forma que marido e mulher não quem jun-
tos?

14. Quantas são as permutações das letras da palavra BRASIL em


que o B ocupa o primeiro lugar, ou o R ocupa o segundo lugar,
ou o L o sexto lugar?

15. De quantas formas podemos representar o número 15 como soma


de vários números naturais?

16. Quantos quadrados perfeitos existem entre 40.000 e 640.000 que


são múltiplos simultaneamente de 3, 4 e 5?

17. Oito amigos vão ao cinema assistir a um lme que custa um real.
Quatro deles possuem uma nota de um real e quatro possuem
200 5 Contagem

uma nota de dois reais. Sabendo-se que o caixa do cinema não


possui nenhum dinheiro, como eles podem organizar uma la
para pagar o lme permitindo o troco pelo caixa?

18. Se considerarmos todas as congurações do tabuleiro com duas


torres que não se atacam, como no Exemplo 5.2, sem distinguir
as torres, quantas congurações obteremos?

19. Continuando o problema anterior, generalize-o para 3, 4, 5, . . .


torres que não se atacam, encontrando também o número máxi-
mo de torres que podem ser colocadas no tabuleiro de modo que
duas delas não se ataquem.

20. Tente fazer o problema anterior para cavalos de xadrez.

21. Mostre que em toda sequência de n2 + 1 inteiros distintos possui


uma subsequência crescente de n + 1 elementos ou uma sub-
sequência decrescente de n + 1 elementos.

22. Encontre o número de zeros que termina o número 2010!.

23. O jogo do 7 consiste em lançar dois dados e somar o número


obtido nas suas faces. Caso a soma seja 7, o jogador A ganha o
dois reais do jogador B . Caso a soma não seja 7, o jogador B
ganha um real de A. Pergunta-se: quem leva vantagem?

24. A função φ de Euler associa a cada número natural n o valor


φ(n) igual ao número de inteiros positivos menores ou iguais a
n relativamente primos com n. Ou seja,

φ(n) = {1 ≤ m ≤ n; (m, n) = 1} .
5.9 Exercícios Propostos 201

Usando os princípios estudados, mostre que se n se decompõe


em fatores primos como n = p1 α1 p2 α2 . . . pαk k , então
    
1 1 1
φ(n) = n 1 − 1− ... 1 − .
p1 p2 pk

O leitor pode achar mais informações sobre a função φ de Euler


nos livros [11] ou ainda [10].
202 5 Contagem
6
Indução Matemática

Se as pessoas não a ham a Matemáti a simples é só por que ainda

não per eberam o quanto a vida é ompli ada.

John von Neumann

Imagine uma la com innitos dominós, um atrás do outro. Supo-


nha que eles estejam de tal modo distribuídos que, uma vez que um
dominó caia, o seu sucessor na la também cai. O que acontece quando
derrubamos o primeiro dominó?
Apesar da simplicidade da pergunta acima ela traz em sua essência
toda a ideia usada no método da indução nita . Muitas descobertas
em Matemática são feitas baseadas na realização de testes que nos
fornecem evidências empíricas. Tais evidências são estudadas para
efetivamente vericarmos se os resultados que elas insinuam são ver-
dadeiros. O método da indução nita constitui uma ferramenta muito
útil na hora de desvendar a veracidade de resultados provenientes deste
tipo de estudo. Esse método é uma das grandes armas do matemático
moderno e tem utilidade na solução de vários problemas, como iremos
ver ao longo deste capítulo.

203
204 6 Indução Matemática

6.1 Formulação Matemática

No início do século XX, o matemático Giuseppe Peano (1858-1932)


estabeleceu os axiomas necessários que nos permitem hoje descrever
com precisão o conjunto dos números naturais. O último dos seus
axiomas diz o seguinte: seja A um subconjunto de N (A ⊂ N). Se 1 ∈
A e se, além disso, A contém todos os sucessores dos seus elementos,
então A = N.
Este axioma é conhecido como axioma de indução e serve como
base do método de demonstração por indução, o qual é de grande
utilidade para estabelecer provas rigorosas em Matemática.
O princípio da boa ordenação dos naturais, enunciado no Capí-
tulo 3, e o axioma de indução não são independentes e sem nenhuma
conexão. De fato, eles são equivalentes, ou seja, se consideramos o
princípio da boa ordenação como sendo um postulado podemos dedu-
zir o axioma de indução e, reciprocamente, se consideramos o axioma
de indução como sendo um postulado podemos deduzir o princípio da
boa ordenação.
No resto do capítulo, p(n) representa uma armação em relação ao
natural n, podendo esta ser verdadeira ou falsa.

Teorema 6.1 (Princípio da Indução Finita). Considere n0 um in-


teiro não negativo. Suponhamos que, para cada inteiro n ≥ n0 , seja
dada uma proposição p(n). Suponha que se pode vericar as seguintes
propriedades:

(a) p(n0 ) é verdadeira;

(b) se p(n) é verdadeira então p(n + 1) também é verdadeira, para


todo n ≥ n0 .
6.1 Formulação Matemática 205

Então, p(n) é verdadeira para qualquer n ≥ n0 .

A armação (a) é chamada de base da indução e a (b) de passo


indutivo. O fato de que p(n) é verdadeira no item (b) é chamado de
hipótese da indução.
Demonstração. Denamos o conjunto

V = {m inteiros não negativos; m ≥ n0 e p(m) é verdadeira} .

Notemos que V é não vazio, pois a condição (a) nos assegura que
n0 ∈ V . A prova do teorema é equivalente a mostrarmos que

V = {n0 , n0 + 1, n0 + 2, n0 + 3, · · · },

ou equivalentemente, a provarmos que o conjunto

F = {m inteiros não negativos; m ≥ n0 e p(m) é falsa}

é vazio. Suponhamos que F é não vazio. Pelo principio da boa orde-


nação existe um menor elemento m0 ∈ F , onde p(m0 ) é falso. Obser-
vemos que,

• m0 ≥ n0 + 1. De fato, m0 ≥ n0 , porém a possibilidade m0 = n0


contradiz a condição (a);

• m0 − 1 ∈ V . Com efeito, p(m0 − 1) é verdadeira pois, caso


contrário, m0 − 1 ∈ F e, além disso, m0 − 1 < m0 , contradizendo
isto a minimalidade de m0 .

Finalmente, como p(m0 − 1) é verdadeira, segue da condição (b) que


p(m0 ) também é verdadeira, o que é impossível pela denição de m0 .
Portanto, o conjunto F é vazio, concluindo-se assim a prova.
206 6 Indução Matemática

Para um pouco mais sobre a relação entre os princípios de indução


e da boa ordenação, recomendamos o Apêndice A da referência [11].
Observação 6.2. Uma grande vantagem do princípio da indução -
nita é poder provar que uma quantidade innita de armações são
verdadeiras, simplesmente vericando que uma quantidade nita des-
tas armações são verdadeiras. Deixaremos clara a utilidade deste
método resolvendo alguns problemas na próxima seção.

6.2 Aplicações

Dentro da grande gama de problemas que podem ser abordados apli-


cando o método de indução podemos distinguir três importantes gru-
pos:
• demonstração de identidades;

• demonstração de desigualdades;

• demonstração de problemas de divisibilidade.

A seguir damos vários exemplos de como aplicar o método em


problemas referentes a cada um destes grupos.

6.2.1 Demonstrando Identidades

Começamos com os seguintes problemas clássicos:

(P1) Determinar uma fórmula para a soma dos n primeiros números


pares, isto é,

sp (n) := 2 + 4 + 6 + · · · + 2n.
6.2 Aplicações 207

(P2) Determinar uma fórmula para a soma dos n primeiros números


ímpares, isto é,

si (n) := 1 + 3 + 5 + · · · + 2n − 1.

Para induzir ambas as fórmulas, primeiro fazemos os cálculos para


vários valores de n, os quais apresentamos na seguinte tabela.

n 1 2 3 4 5 ···
sp (n) 2 = 1 · 2 6=2·3 12 = 3 · 4 20 = 4 · 5 30 = 5 · 6 · · ·
si (n) 1 = 12 4 = 22 9 = 32 16 = 42 25 = 52 ···

Os resultados na tabela sugerem que sp (n) = n(n + 1) e que


si (n) = n2 . Entretanto, isto não constitui por si só uma prova ri-
gorosa destas fórmulas, pois para poder garantir a veracidade delas
utilizando a tabela teríamos que vericar cada valor de n natural,
sendo isto impossível. Provaremos agora que, de fato, as fórmulas
induzidas são válidas usando o método de indução nita.

Exemplo 6.3. Demonstre que para qualquer n ∈ N é válida a igual-


dade:
2 + · · · + 2n = n(n + 1).

Solução. Denamos a proposição

p(n) : 2 + · · · + 2n = n(n + 1)

e observemos que a mesma vale para n = 1 (base da indução); de fato

p(1) : 2 = 1(1 + 1).

Agora partimos para a prova do passo indutivo:


208 6 Indução Matemática

• Hipótese: suponhamos que p(k) é verdadeira para um certo k >


1, k ∈ N.

• Tese: devemos mostrar que p(k + 1) também é verdadeira.

Com efeito, como

2 + · · · + 2k = k(k + 1),
somando 2(k + 1) a ambos os lados desta igualdade, temos que

2 + · · · + 2k + 2(k + 1) = k(k + 1) + 2(k + 1)


= (k + 2)(k + 1).

Esta última igualdade arma que p(k + 1) também é verdadeira. O


Princípio de Indução nos garante que p(n) é verdadeira para qualquer
n ∈ N.

Exemplo 6.4. Demonstre que para qualquer n ∈ N é válida a igual-


dade:
1 + 3 + 5 + · · · + 2n − 1 = n2 .

Solução. Aqui denimos a proposição:

p(n) : 1 + 3 + 5 + · · · + 2n − 1 = n2

e notamos que a mesma é válida se tomarmos, por exemplo, n = 1.


De fato,
p(1) : 1 = 2 · 1 − 1.
Agora só resta provar o passo indutivo:
• Hipótese: suponhamos que p(k) seja verdadeira para um certo
k > 1, k ∈ N.
6.2 Aplicações 209

• Tese: devemos mostrar que p(k + 1) também é verdadeira.

Com efeito, como

1 + 3 + 5 + · · · + 2k − 1 = k 2 ,

somando 2k + 1 a ambos os lados desta igualdade, temos que

1 + 3 + 5 + · · · + 2k − 1 + 2k + 1 = k 2 + 2k + 1
= (k + 1)2 .

O princípio de indução nos garante que p(n) é verdadeira para


qualquer n ∈ N.

Uma consequência imediata do Exemplo 6.3 é a fórmula para a


soma dos n primeiros números naturais, dada por
n(n + 1)
sn = 1 + 2 + 3 + · · · + n = . (6.1)
2
Com efeito, como

2 + 4 + · · · + 2n = n(n + 1),

então dividindo por 2 ambos os membros da igualdade acima, obtemos


a equação (6.1).
Continuando com o mesmo raciocínio, é natural nos perguntarmos
se é possível obter uma fórmula para a soma dos n primeiros quadrados
perfeitos, ou seja, determinar qn onde:

qn = 12 + 22 + 32 + · · · + n2 .

Para induzir a fórmula, consideramos os valores de sn e qn numa tabela:


210 6 Indução Matemática

n 1 2 3 4 5 6 ···
sn 1 3 6 10 15 21 · · ·
qn 1 5 14 30 55 91 · · ·
Aparentemente não existe nenhuma relação entre sn e qn . Mas, se
considerarmos o quociente qn /sn , vejamos o que acontece:

n 1 2 3 4 5 6 ···
qn /sn 3/3 5/3 7/3 9/3 11/3 13/3 ···

Isso nos sugere que vale a relação


qn 2n + 1
= ,
sn 3
logo nosso candidato para valor de qn é
sn (2n + 1) n(n + 1)(2n + 1)
qn = = .
3 6
Convidamos o leitor a provar a veracidade da equação acima utilizando o
Método da Indução no Exercício 1 no nal do capítulo.

6.2.2 Demonstrando Desigualdades

Apresentamos agora alguns exemplos de como usar indução para provar


desigualdades.

Exemplo 6.5. Prove que 3n−1 < 2n2 para todo n ∈ N.


Solução. Denotamos por p(n) a propriedade: 3n−1 < 2n . É claro que p(1)
2

é válida, pois 1 < 2. Agora supondo que P (n) é verdadeira temos que
2 2
3n = 3n−1 · 3 < 2n · 22n+1 = 2(n+1) ,

logo p(n + 1) também vale. Observamos que na desigualdade acima usamos


o fato de que 3 < 22n+1 para qualquer n ∈ N.
6.2 Aplicações 211

Exemplo 6.6. Mostre que para todo número n ∈ N, n > 3, vale que 2n < n!
Demonstração. Para n = 4 a desigualdade é vericada, pois 24 = 16 < 4! =
24. Vamos assumir como hipótese de indução que a desigualdade é válida
para n ≥ 4. Então, precisamos mostrar que a mesma vale também para
n + 1. De fato, por hipótese de indução:

2n < n! (6.2)

Como 2 < n + 1, podemos multiplicar o lado esquerdo da desigualdade


em (6.2) por 2 e o lado direito por n+1, sem alterar o sinal de desigualdade.
Logo, temos que:

2n .2 = 2n+1 < n!(n + 1) = (n + 1)!,

concluindo-se a demonstração.

Exemplo 6.7. Prove que, para todo n ∈ N,


s r q

2+ 2+ 2 + · · · + 2 < 2.
| {z }
n−radicais

Demonstração. Claramente a desigualdade vale para n = 1, pois 2 < 2.
Suponhamos que para certo n ∈ N a desigualdade acontece, então
s r q

2+ 2 + 2 + · · · + 2 < 2.
| {z }
n−radicais

Logo, adicionando 2 em ambos os lados desta desigualdade tem-se


s r q

2+ 2+ 2 + 2 + · · · + 2 < 2 + 2.
| {z }
n−radicais
212 6 Indução Matemática

Tomando raiz quadrada em ambos os lados desta última desigualdade ob-


temos v
u s r
u q
t √
2 + 2 + 2 + 2 + · · · + 2 < 2,
| {z }
n+1−radicais

como desejávamos.

6.2.3 Indução e Problemas de Divisibilidade

Agora damos alguns exemplos de problemas de divisibilidade que podem


ser mostrados utilizando o método da indução:

Exemplo 6.8. Mostre que para qualquer n ∈ N, n3 + 2n é sempre divisível


por 3.

Solução. Para n = 1 a armação é válida, pois 13 +2·1 = 3, que obviamente


é divisível por 3.
Assumamos como hipótese indutiva que a armação vale para algum
k ∈ N, isto é,
Hipótese: k3 + 2k é divisível por 3.

Devemos mostrar que a armação também é verdadeira para k + 1, ou


seja, temos que provar que

Tese: (k + 1)3 + 2(k + 1) é divisível por 3.

Para provar isto último, usamos o fato de que

(k + 1)3 + 2(k + 1) = (k 3 + 3k 2 + 3k + 1) + (2k + 2);


6.2 Aplicações 213

agrupando adequadamente,

(k + 1)3 + 2(k + 1) = (k 3 + 2k) + (3k 2 + 3k + 3)


= (k 3 + 2k) + 3(k 2 + k + 1)
| {z } | {z }
múltiplo de 3 múltiplo de 3

= múltiplo de 3,

concluindo assim a prova.


Exemplo 6.9. Mostre que a soma dos cubos de três números naturais con-
secutivos é divisível por 9.
Solução. Denamos a seguinte proposição:
p(n) : n3 + (n + 1)3 + (n + 2)3 é um múltiplo de nove.

Notemos que P (1) é válida, pois


13 + 23 + 33 = 1 + 8 + 27 = 36 = 9 · 4.

Precisamos provar agora o passo indutivo, isto é,


• Hipótese: P (k) é verdadeira para algum k ∈ N.

• Tese: P (k + 1) também é verdadeira.


Para provar isto, observamos que
(k + 1)3 + (k + 2)3 + (k + 3)3 = (k + 1)3 + (k + 2)3 + (k 3 + 9k 2 + 27k + 27).

Ordenando adequadamente, temos que o lado direito da última igualdade


se escreve como
k 3 + (k + 1)3 + (k + 2)3 + (9k 2 + 27k + 27)
= k 3 + (k + 1)3 + (k + 2)3 + 9(k 2 + 3k + 3)
| {z } | {z }
múltiplo de 9 múltiplo de 9

= múltiplo de 9,

completando assim nossa demonstração.


214 6 Indução Matemática

Muitas vezes, para conseguir mostrar que a hipótese p(n + 1) é verda-


deira, precisamos supor que p(k) é verdadeira para todo n0 ≤ k ≤ n. Isto
é a base do princípio forte da indução nita que enunciamos a seguir:

Teorema 6.10 (Princípio Forte da Indução Finita). Considere n0 um in-


teiro não negativo. Suponhamos que, para cada inteiro n ≥ n0 seja dada
uma proposição p(n) e que valem as propriedades
(a) p(n0 ) é verdadeira;

(b) se para cada inteiro não negativo k, com n0 ≤ k ≤ n, temos que p(k)
é verdadeira, então p(n + 1) é também verdadeira.
Então, a proposição p(n) é verdadeira para qualquer n ≥ n0 .

Utilizando o princípio forte da indução, vamos dar uma prova diferente


do teorema fundamental da aritmética da apresentada no Capítulo 3.

Exemplo 6.11 (Teorema Fundamental da Aritmética). Todo número na-


tural N maior que 1 pode ser escrito como um produto

N = p1 · p2 · p3 · · · pm , (6.3)

onde m ≥ 1 é um número natural e os pi , 1 ≤ i ≤ m são números primos.


Além disso, a fatoração em (6.3) é única se exigirmos que p1 ≤ p2 ≤ · · · ≤
pm .

Solução. Para cada n ∈ N, n ≥ 2, denamos a proposição

p(n) : n é escrito de modo único como um produto de números primos.

Notemos que p(2) é verdadeira, pois 2 é um número primo.


Agora enunciemos o passo indutivo:

• Hipótese indutiva: p(k) é verdade para cada inteiro k tal que 2 ≤ k ≤


n.
6.3 Indução na Geometria 215

• Tese: p(n + 1) é verdade. Em outras palavras, temos que mostrar que


n + 1 é escrito de modo único como um produto de números primos.

Faremos a prova dividindo em dois casos:


(a) Se n + 1 é um número primo, então p(n + 1) é verdade e isto acaba
nossa demonstração.
(b) Se n + 1 não é um número primo, então existem α, β ∈ N com 2 ≤
α ≤ n e 2 ≤ β ≤ n tais que n + 1 = α · β .
Nossa hipótese indutiva é válida para α e β . Isto signica que α se
escreve de modo único como um produto de números primos e que β
se escreve de modo único um produto de números primos. Portanto,
n + 1 = α · β se escreve como um produto de números primos.
Agora mostraremos que n + 1 se escreve de modo único como produto
de primos. Assuma que
p1 p2 . . . pk = q1 q2 . . . qm = n + 1, (6.4)
com p1 ≤ p2 ≤ · · · ≤ pk e q1 ≤ q2 ≤ · · · ≤ qm todos primos. Vamos
mostrar que necessariamente k = m e pi = qi .
De fato, como p1 é primo, ele divide algum qi . Logo, como qi é primo,
p1 = qi ≥ q1 . Analogamente, existe um j tal que q1 = pj ≥ p1 . Logo,
p1 = q1 . Cancelando p1 em ambos os lados da equação (6.4), temos
que (n + 1)/p1 = p2 . . . pk = q2 . . . qm ≤ n. Logo, por hipótese de
indução, k = m e p2 = q2 , . . . , pm = qm , encerrando a demonstração.

6.3 Indução na Geometria

Tratamos aqui alguns exemplos que mostram a utilidade do método de


indução na resolução de problemas geométricos. Vamos começar estudando
216 6 Indução Matemática

duas propriedades importantes dos polígonos. A primeira delas trata da


soma dos ângulos internos de um polígono convexo de n lados (n-ágono).
Um polígono convexo é um polígono tal que qualquer segmento de reta
que liga dois de seus pontos está contido no interior dele. No caso de
polígonos, isto é equivalente ao fato de que todo segmento que liga dois
vértices ou é uma aresta ou está contido no interior do polígono.

Exemplo 6.12. Mostre que a soma dos ângulos internos de um polígono


convexo de n lados (n ≥ 3) é igual a (n − 2)π radianos.

Solução. No caso de n = 3 a propriedade acima é muito bem conhecida.


Desde Tales de Mileto e Euclides se conhecia que a soma dos ângulos internos
de um triângulo é π radianos. Façamos mais um caso, tomando n = 4. Neste
caso, podemos dividir um quadrilátero em dois triângulos, como mostra a
Figura 6.1 (a). Assim, a soma dos ângulos internos de um quadrilátero é
2π radianos.

A4 A4
A3

A5 A3

A1 A2 A1 A2
(a) (b)

Figura 6.1: Dividindo polígonos

Para elucidar o processo de indução e não deixar dúvidas sobre o que


iremos fazer, vamos considerar mais um polígono, o pentágono (n = 5).
Neste caso, para mostrar que a soma dos seus ângulos internos é (5 − 2)π =
3π radianos, iremos dividir o pentágono A1 A2 A3 A4 A5 em um quadrilátero
A1 A2 A3 A4 e um triângulo A1 A4 A5 , como mostra a Figura 6.1 (b). Assim,
6.3 Indução na Geometria 217

a soma dos ângulos internos do pentágono A1 A2 A3 A4 A5 é igual à soma dos


ângulos internos do triângulo A1 A4 A5 (igual a π ) mais a soma dos ângulos
internos do quadrilátero A1 A2 A3 A4 (igual a 2π ), ou seja, é igual a 3π .
Finalmente, vamos assumir como hipótese de indução que para um certo
n ≥ 3 mostramos que a soma dos ângulos internos do n-ágono é dada pela
expressão (n − 2)π . Precisamos mostrar que a soma dos ângulos internos
de um n + 1-ágono é [(n + 1) − 2]π = (n − 1)π . De fato, podemos repetir
o processo anterior. Vamos denominar de A1 , A2 , . . . , An , An+1 os vértices
consecutivos do (n + 1)-ágono. Podemos dividi-lo no n-ágono A1 A2 . . . An e
no triângulo A1 An+1 An . Logo, a soma dos ângulos internos do (n+1)-ágono
é (n − 2)π + π = (n − 1)π .
Exemplo 6.13. Mostre que o número de diagonais de um polígono convexo
n(n−3)
de n-lados é igual a 2 .
Solução. Observe que para n = 3 temos que existem 0 = 3.(3 − 3)/2 dia-
gonais num triângulo. Para n = 4, temos 2 = 4(4 − 3)/2 diagonais num
quadrilátero convexo (veja a Figura 6.2).
Vamos agora assumir como hipótese de indução que se n é um n-
ágono convexo então o seu número de diagonais é n(n − 3)/2 e vamos pro-
var que a fórmula vale para um (n + 1)-ágono convexo. De fato, denote
por A1 , A2 , . . . , An , An+1 os vértices consecutivos do n + 1-ágono. Pode-
mos decompô-lo como a união do n-ágono A1 , A2 , . . . , An e do triângulo
A1 , An , An+1 . Neste caso, para contarmos as diagonais do (n + 1)-ágono
devemos considerar os seguintes casos:
• Diagonais do n-ágono A1 , A2 , . . . , An ; por hipótese de indução, o nú-
mero dessas diagonais é n(n − 3)/2.
• n − 2 diagonais que partem do vértice An+1 mais a diagonal A1 An .
Assim, o número total de diagonais do (n + 1)-ágono é
n(n − 3) n2 − 3n + 2n − 2 n2 − n − 2 (n + 1)(n − 2)
+ (n − 2) + 1 = = = ,
2 2 2 2
218 6 Indução Matemática

como queríamos demonstrar.

A4 A4
A3

A5 A3

A1 A2 A1 A2
(a) (b)

Figura 6.2: Diagonais de polígonos

Exemplo 6.14. Mostre que podemos cobrir os n2 pontos no reticulado a


seguir traçando 2n − 2 segmentos de reta sem tirar o lápis do papel.

• • • • •

• • • • •

• • • • •

• • • • •

• • • • •
| {z }
n×n−pontos

Figura 6.3: O problema de bar n × n


6.3 Indução na Geometria 219

Solução. O caso n = 3 já foi enunciado no Problema 1.12 do Capítulo 1. A


gura a seguir mostra a solução, onde o caminho realizado com as 4 linhas
é o seguinte: A → B → C → D → B .

A
• • •

• • •

• • •
D B

Figura 6.4: Solução do problema de bar 3 × 3

Daremos a prova do problema acima por indução. Para isso, veja que
podemos resolver o caso n = 4 continuando a solução do caso n = 3. Como
paramos num dos vértices do quadrado 3×3, acrescentamos mais uma linha
e uma coluna para obter um reticulado 4 × 4. Assim, conseguimos cobrir os
16 pontos utilizando 4 + 2 = 6 linhas, sem tirar o lápis do papel e cobrindo
dois lados do quadrado, como mostram as linhas descontínuas na Figura
6.5.

• • •C •
A
• • • •

• • • •

• • • •
D B

Figura 6.5: Completando o reticulado


220 6 Indução Matemática

Finalmente, vamos assumir como hipótese de indução que podemos co-


brir n ≥ 2 um reticulado n × n com 2n − 2 linhas, sendo que a última delas
cobre um dos lados do reticulado. Acrescentando 2n+1 pontos como mostra
a Figura 6.5, obtemos um reticulado (n + 1) × (n + 1) que pode ser coberto
com 2n − 2 + 2 = 2(n + 1) − 2 pontos, como queríamos demonstrar.

6.4 Miscelânea

Nesta seção discutiremos alguns exemplos interessantes de como podemos


aplicar o método da indução aos mais variados tipos de problemas. O
primeiro deles é uma generalização do Problema 1.8.

Exemplo 6.15 (A Moeda Falsa). Um rei muito rico possui 3n moedas de


ouro. Porém, uma destas moedas é falsa e seu peso é menor que o peso das
demais. Com uma balança de 2 pratos e sem nenhum peso, mostre que é
possível encontrar a moeda falsa com apenas n pesagens.

Solução. Para resolver este problema, vamos utilizar o Método da Indução.


De fato, se n = 1, procederemos da seguinte forma: pegamos duas moedas
quaisquer e colocamos na balança, deixando uma do lado de fora. Caso a
balança se equilibre, a moeda que está do lado de fora é necessariamente a
que tem menor peso. Caso a balança se desequilibre, a que tem menor peso
está na balança, no prato mais alto. O caso n = 2 foi feito no Problema
1.8.
Vamos agora assumir como hipótese de indução que dadas 3n moedas,
podemos achar a moeda mais leve com n pesagens. Vamos mostrar que
para 3n+1 moedas, é suciente n + 1 pesagens. De fato, dividiremos as
3n+1 moedas em 3 grupos, A, B e C com 3n moedas cada. Colocamos na
balança os grupos A e B . Caso os dois grupos se equilibrem, a moeda mais
leve está no grupo C . Caso o grupo A esteja mais leve, a moeda mais leve se
encontra no grupo A. De qualquer modo, com uma pesagem conseguimos
6.4 Miscelânea 221

determinar em qual grupo de 3n elementos a moeda mais leve se encontra.


Por hipótese de indução, precisamos de mais n pesagens para encontrar a
moeda mais leve, totalizando n + 1 pesagens. Desaamos o leitor a mostrar
que não é possível realizar tal tarefa com menos de n pesagens.

Exemplo 6.16. Mostre que utilizando um balde com 5 litros de capacidade


e outro com 7 litros, é possível separar qualquer quantidade superior ou igual
a 24 litros.

Solução. Novamente, faremos a prova utilizando o Método da Indução.


Neste caso, começaremos o processo de indução a partir de 24. De fato,
podemos separar 24 litros utilizando duas vezes o balde de 7 e duas vezes o
balde de 5 litros. Note que o problema acima equivale a mostrar que

Todo número maior ou igual a 24 pode ser escrito da forma


7x + 5y , onde x e y são números inteiros maiores ou iguais a
zero.

Neste caso, escrevemos 24 como 24 = 2 · 7 + 2 · 5. Por hipótese de


indução, vamos supor que conseguimos escrever um número n ≥ 24 como
n = 7x + 5y , com x e y números inteiros maiores ou iguais a zero. Devemos
mostrar que n + 1 se escreve deste modo também. Para isso, vamos dividir
a análise em dois casos:
Caso 1: y≤3
Logo, x ≥ 2 pois se isso não ocorresse, teríamos 7x + 5y ≤ 22 < 24, o
que é impossível. Assim, podemos escrever:

n + 1 = 7x + 5y + 1 = 7(x − 2) + 5(y + 3),

pois x − 2 ≥ 0.
Caso 2: y≥4
222 6 Indução Matemática

Neste caso, y − 4 ≥ 0. Logo, podemos escrever:

n + 1 = 7x + 5y + 1 = 7(x + 3) + 5(y − 4),

nalizando a nossa prova por indução.

6.4.1 Cuidados ao Usar o Princípio da Indução

Observação 6.17. Quando aplicamos o princípio da indução devemos to-


mar certos cuidados. A seguir damos um exemplo de como o método pode
ser aplicado de forma errada. Vamos mostrar a seguinte armação:

Armação: Num conjunto qualquer de n bolas, todas as bolas


possuem a mesma cor.

Observe que nossa proposição é claramente falsa. Mas, mesmo assim, vamos
dar uma prova por indução.
Para n = 1, nossa proposição é verdadeira pois em qualquer conjunto
com uma bola, todas as bolas têm a mesma cor, pois só existe uma bola. As-
suma por hipótese de indução que a proposição é verdadeira para n e prove-
mos que a proposição é verdadeira para n+1. Ora, seja A = {b1 , . . . , bn , bn+1 }
o conjunto com n + 1 bolas referido. Considere os subconjuntos de B e C
de A com n elementos, construídos como:

B = {b1 , b2 , . . . , bn } e C = {b2 , . . . , bn+1 }

Observe que ambos os conjuntos têm n elementos. Assim, as bolas


b1 , b2 , . . . , bn do conjunto B têm a mesma cor. Do mesmo modo, as bo-
las do conjunto C têm a mesma cor. Em particular, a bola bn tem a mesma
cor da bola bn+1 . Assim, todas as bolas têm a mesma cor. Ache o erro no
argumento! Se você não conseguir, leia a nota de rodapé. 1
1 Uma dica da solução encontra-se no nal do capítulo.
6.5 Indução e Recorrências 223

6.5 Indução e Recorrências

Vamos começar esta seção discutindo um problema muito conhecido e inte-


ressante.

Exemplo 6.18 (As Torres de Hanói2 ). Diz uma antiga lenda que na origem
dos tempos, em um templo de Hanói, foram colocados 64 discos perfurados
de ouro puro e de diâmetros diferentes ao redor de uma de três hastes de
diamante. Muitos sacerdotes moviam os discos, respeitando as seguintes
regras: eles começam empilhados em ordem crescente de acordo com seu
tamanho (ver Figura 6.6). Os discos podem ser deslocados de uma coluna
para qualquer outra, sendo que nunca pode ser colocado um disco maior em
cima de um menor e a cada segundo os sacerdotes movem um disco.
Quando os sacerdotes transportassem todos os discos de uma coluna para
outra, o mundo se acabaria. Suponha que eles começaram esse processo no
ano 2000 e que a lenda é verdadeira, quanto tempo ainda resta para a Terra?

Figura 6.6: Torre de Hanói

Para responder esse problema, consideraremos o problema geral de des-


cobrir quantos movimentos são necessários para mover n anéis de uma haste
para outra. Argumentaremos do seguinte modo: observe que podemos mover
os discos para outra haste se n = 1 ou 2. Com efeito, se temos somente um
anel basta mover este para qualquer outra haste com um único movimento.
2 Este jogo foi inventado, em 1882, pelo matemático Francês Édouard Lucas.
224 6 Indução Matemática

Se temos 2 anéis então movemos o menor deles para a segunda haste, o


maior para a terceira haste e, nalmente, o menor para a terceira haste,
realizando um total de 3 movimentos. Para calcular o caso geral, vamos
empregar um método chamado de método recursivo: o número ak+1 de mo-
vimentos necessários para mover k + 1 anéis será expresso como uma função
de ak .
De fato, se temos k + 1 anéis na primeira haste e sabemos mover k anéis
de uma haste para outra utilizando ak movimentos, então podemos mover
todos os k + 1 anéis para a segunda haste usando 2ak + 1 movimentos.
De fato, movemos todos eles, exceto o maior, para a terceira haste usando
ak movimentos. A seguir, colocamos o maior na segunda haste usando 1
movimento. Imediatamente, deslocamos todos os anéis da terceira haste
para a segunda haste usando mais ak movimentos. Logo, movemos todos os
k + 1 anéis utilizando 2ak + 1 movimentos. Em resumo:

ak+1 = 2ak + 1, (6.5)

onde ak é o número de movimentos necessários para mover k discos de uma


haste para outra. Vamos agora usar indução para provar que ak = 2k − 1.
Uma vez constatada a veracidade da armação para k = 1, 2, para
calcular ak , por hipótese de indução, vamos assumir que ak = 2k − 1. Temos
pela equação (6.5):

ak+1 = 2ak + 1 = 2(2k − 1) − 1 = 2k+1 − 1.

como queríamos demonstrar.


Vamos aproveitar o Exemplo 6.18 para discutir algumas equações que
aparecem em muitas situações em Matemática: as equações de recorrência.
Em geral, uma equação de recorrência é uma equação envolvendo uma
certa quantidade de termos de sequência xn . Para ilustrar isso, observe
a equação (6.5). Aqui, estaremos interessados em um tipo particular de
equação de recorrência, as equações de recorrência lineares.
6.5 Indução e Recorrências 225

Denição 6.19. Uma equação de recorrência linear de grau k é uma ex-


pressão da forma:

xn+1 =rk−1 xn + rk−2 xn−1 + · · · + r0 xn−k+1


(6.6)
x1 = a1 , x2 = a2 , . . . , xk = ak ,

onde r0 , r1 , . . . , rk−1 são números reais e r0 6= 0.

Por exemplo, são equações de recorrência lineares as seguintes equações


2
2xn − 3xn+1 = 0 e − 3xn + xn+1 = 5xn+2
3
e não são equações de recorrência lineares as equações
2
2(xn )3 − 5xn+1 = 0 e − 3xn + xn+1 = 5xn+2 + 3.
3
Exemplo 6.20 (Sequência de Fibonacci). Um exemplo muito interessante
de equação de recorrência é a sequência conhecida por sequência de Fibo-
nacci, devido ao matemático italiano Leonardo di Pisa (1170-1250). Esta
sequência adquiriu muita fama devido a suas conexões com áreas das mais
variadas na cultura humana. Ela aparece em problemas de Biologia, Ar-
quitetura, Engenharia, Física, Química e muitos outras áreas da ciência e
arte.
Denimos a sequência de Fibonacci como sendo a sequência Fn que
satisfaz a seguinte equação de recorrência:

F1 = 1;
F2 = 1;
Fn = Fn−1 + Fn−2 , se n ≥ 3.

Agora vamos utilizar indução para mostrar algumas de suas proprieda-


des.
226 6 Indução Matemática

Exemplo 6.21. Considere Fn a sequência de Fibonacci. Mostre que


 n
7
Fn < .
4
n
Solução. Denamos a proposição p(n) := Fn < 47 . Para n = 1 temos
que F1 = 1 < 74 , de modo que p(1) é verdadeira. Suponhamos que

p(1), p(2), . . . , p(n), ∀n ≥ 2,



7 n+1
sejam todas verdadeiras. Mostraremos que Fn+1 < 4 . Com efeito,

7 n
n−1
Fn+1 = Fn + Fn−1 < 4 + 47
n−1 n−1
< 74 74 + 74
 n−1
< 1 + 74 74 .
 
7 2

7 2

7 n−1
Como 1 + 7
4 < 4 , segue-se que Fn+1 < 4 4 . Portanto,

7 n+1
Fn+1 < 4 .

Exemplo 6.22. Dada a seguinte relação de recorrência

a0 = 8;
a1 = 10;
an = 4an−1 − 3an−2 , ∀n ≥ 2.

Mostre que an = 7 + 3n , para todo n ∈ Z+ .

Solução. Denamos a proposição P (n) : an = 7 + 3n . P (0) é verdadeira,


pois P (0) = 7 + 30 = 7 + 1 = 8. Suponhamos que P (k) é verdadeiro para
cada inteiro k tal que 1 ≤ k ≤ n. Vamos mostrar que P (k) é verdade para
6.5 Indução e Recorrências 227

k = n + 1. Com efeito,
an+1 = 4an − 3an−1
= 4(7 + 3n ) − 3(7 + 3n−1 )
= 7 + 4 × 3n − 3 × 3n−1

= 7 + 3n−1 4 × 3 − 3

= 7 + 3n−1 9 = 7 + 3n−1 × 32
= 7 + 3n+1 .

Vamos agora discutir o caso geral da equação de recorrência linear (6.6).


Para isso, vamos fazer algumas observações preliminares que deixaremos a
cargo do leitor:
• se an e bn são soluções da equação (6.6), então an + bn também é
solução;
• se an é solução da equação (6.6) e α é um número real, então αan
também é solução.
Com isto em mente, vamos descrever agora como obter todas as soluções
xn da equação (6.6) em função de n. Observe que dados os termos iniciais
a1 , a2 , . . . , ak a sequencia xn ca inteiramente determinada pela equação de
recorrência. O interessante aqui é determinar o termo xn+1 sem que seja
preciso o cálculo dos termos xn , xn−1 , . . . , xn−k+1 .
Vamos primeiro procurar o que se chama de solução particular da equa-
ção (6.6). Particular porque ela assume uma forma característica e porque
não assumiremos que as condições x1 = a1 , . . . , xk = ak valham.
Vamos procurar soluções do tipo xn = λn , onde λ é um número real
positivo. Neste caso, temos que:
λn+1 = xn+1 =rk−1 xn + rk−2 xn−1 + · · · + r0 xn−k+1
= rk−1 λn + rk−2 λn−1 + · · · + r0 λn−k+1 .
228 6 Indução Matemática

Passando os termos do lado direito da igualdade e colocando em evi-


dência o termo λn−k+1 temos:


λn−k+1 λk − rk−1 λk−1 − rk−2 λk−2 − · · · − r0 = 0. (6.7)

Assim, como λk 6= 0, pois λ > 0, temos que

λk − rk−1 λk−1 − rk−2 λk−2 − · · · − r0 = 0. (6.8)

O polinômio

p(λ) = λk − rk−1 λk−1 − rk−2 λk−2 − · · · − r0

recebe o nome especial de polinômio característico da equação de recorrên-


cia (6.6). Acabamos de mostrar que qualquer raiz do polinômio caracterís-
tico gera uma solução particular da equação (6.6).
Vamos assumir que a equação (6.8) possui k raízes diferentes, digamos
λ1 > λ2 > · · · > λk . Então vale o seguinte teorema:

Teorema 6.23. Se escolhermos números reais c1 , c2 , . . . , ck , então

xn = c1 λn1 + c2 λn2 + · · · + ck λnk (6.9)

é uma solução da equação de recorrência, onde os termos iniciais ai para


i = 1, 2, . . . , k são:

ai = c1 λi1 + c2 λi2 + · · · + ck λik .

Demonstração. Para mostrar o teorema, como x1 = a1 , . . . xk = ak pela


denição dos ai 's, basta mostrar que xn é uma solução.
Ora, o produto de uma solução por um número real também é uma
solução. Assim, como λni é uma solução para i = 1, 2, . . . , k e ci é um
6.6 Exercícios 229

número real, temos que ci λni é solução para i = 1, 2, . . . , k. Como já vimos


acima, a soma de soluções é também uma solução. Logo,

xn = c1 λn1 + c2 λn2 + · · · + ck λnk

é uma solução.

Neste ponto, voltamos a equação (6.6). Desde o princípio, dados os


números ai buscávamos a solução xn tal que x1 = a1 , . . . , xk = ak . A
Equação (6.9) nos dá uma variedade de soluções, onde podemos escolher
os números ci como bem entendermos. Usando equações lineares, podemos
mostrar que sempre é possível escolher os números ci de modo que x1 =
a1 , . . . , xk = ak . Isso encerra nossa busca. Para complementar esta seção,
recomendamos a leitura do Capítulo 3 de [4].

6.6 Exercícios

1. Se qn denota a soma qn = 12 + 22 + · · · + n2 , prove que para todo


n∈N
n(n + 1)(2n + 1)
qn = .
6
2. Use o princípio da indução para provar as seguintes armações:

(a) 3n+1 + 2n+2 é divisível por 7 para todo n ∈ N;


(b) a soma dos cubos de três números naturais consecutivos é divi-
sível por 9;
(c) 7 + 77 + 777 + · · · + 777
| {z. . . 7} = 7(10n+1 − 9n − 10)/81;
n−vezes

(d) (n + 1)(n + 2) . . . (n + n) = 2n · 1 · 3 · 5 · · · (2n − 1).

3. Use o princípio da indução para provar as seguintes desigualdades:


230 6 Indução Matemática

(a) 2n−1 (an + bn ) > (a + b)n , ∀n ∈ N, com a, b ∈ R, a + b > 0 e


a 6= b;
1 1 1 1 √
(b) √ + √ + √ + · · · + √ > n, para todo n ∈ N;
1 2 3 n
1 1 1 1 13
(c) + + + ··· + > , para todo n ∈ N.
n+1 n+2 n+3 2n 24
4. Mostre a seguinte identidade trigonométrica
(n + 1) cos nx − n cos(n + 1)x − 1
cos x + 2 cos 2x + · · · + n cos nx = .
4 sin2 x2

5. Um torneio de xadrez tem n jogadores. Cada jogador joga uma única


partida com cada um dos outros jogadores. Calcule o número total
de partidas realizadas no torneio.

6. Demonstre que para qualquer n ∈ N é válida a igualdade


 2
3 3 3 n(n + 1)
3
1 + 2 + 3 + ··· + n = .
2

7. Demonstre que para qualquer n ∈ N é valida desigualdade


 
1 n
an = 1 + < 3.
n

8. Prove que, para todo n ∈ N e a > 0,


s r q √
√ 1 + 4a + 1
a+ a + a + ··· + a < .
2
| {z }
n−radicais

9. Mostre que para qualquer número natural n ≥ 0, 11n+2 + 122n+1 é


sempre divisível por 133.

10. Mostre que para todo n ∈ Z+ temos que 32n+1 + 2n+2 é um múltiplo
de 7.
6.6 Exercícios 231

11. Mostre que para todo n ∈ Z+ temos que 32n+2 + 26n+1 é um múltiplo
de 11.

12. Considere Fn a sequência de Fibonacci . Mostre que


√ !n √ !n
1 1+ 5 1 1− 5
Fn = √ −√ .
5 2 5 2

13. Mostre as seguintes propriedades a respeito da sequência de Fibonacci


Fn :
n
X n
X
(a) Fi = Fn+2 − 1; (b) F2i−1 = F2n ;
i=1 i=1
Xn
(c) F2i = F2n+1 − 1; (d) Fn−1 Fn+1 − Fn2 = (−1)n .
i=1

14. De quantas formas diferentes podemos cobrir um tabuleiro de 2 ×


n com peças de dominós que cobrem exatamente duas celas do ta-
buleiro?

15. Calcular o número de regiões em que o plano é dividido por n retas


distintas em cada uma das seguintes situações:

(a) as n retas são concorrentes;


(b) não existem duas retas paralelas nem três retas concorrentes.3

16. Dizemos que uma gura é enquadrável com régua e compasso, se a


partir dela é possível, utilizando apenas régua e compasso, construir
um quadrado de mesma área. Prove que:
3 Até
onde sabemos, este problema é conhecido como a pizza de Steiner, o qual
foi resolvido, em 1826, pelo notável geômetra Jacob Steiner (1796-1863).
232 6 Indução Matemática

(a) um triângulo é sempre enquadrável;


(b) um polígono qualquer é enquadrável.

Sugestão para o item (b): Utilize indução dividindo a gura em tri-


ângulos.

17. Dê uma resposta à situação á Observação 6.17.


Sugestão: Observe a validade do argumento quando o conjunto A tem
2 elementos. Veja que B e C não se intersectam. Ou seja, o passo
indutivo falha de n = 1 para n = 2.
Referências Bibliográcas
[1] AIGNER, M. e ZIEGLER, G. (2002). As Provas estão no
Livro. Edgard Blücher.

[2] GARCIA, A. e LEQUAIN, I. (2003). Elementos de Álgebra.


Projeto Euclides, IMPA.

[3] LIMA, E. L.; CARVALHO, P. C. P.; WAGNER, E. e MOR-


GADO, A.C. (2004). A Matemática do Ensino Médio. Volume
1. Sociedade Brasileira de Matemática.

[4] LIMA, E.L.; CARVALHO, P. C. P.; WAGNER, E. e MOR-


GADO, A.C. (2004). A Matemática do Ensino Médio. Volume
2. Sociedade Brasileira de Matemática.

[5] LIMA,E.L.; CARVALHO,P. C. P.; WAGNER,E. e MOR-


GADO,A.C. (2004). A Matemática do Ensino Médio. Volume
3. Sociedade Brasileira de Matemática.

[6] LIMA, E.L.; CARVALHO, P. C. P.; WAGNER,E. e MOR-


GADO, A.C. (2001). Temas e Problemas. Sociedade Brasileira
de Matemática.

[7] LIMA, E.L. (2001). Álgebra Linear. Sociedade Brasileira de


Matemática.

285
286 REFERÊNCIAS BIBLIOGRÁFICAS

[8] MORAIS FILHO, D. C. (2007). Um Convite à Matemática.


EDUFCG.

[9] MORGADO, A.; CARVALHO, J.; CARVALHO, P.; FER-


NANDEZ, P. (1991). Análise Combinatória e Probabilidade .
Sociedade Brasileira de Matemática.

[10] RIBENBOIM, P. (2001). Números Primos: Mistérios e Re-


cordes. Sociedade Brasileira de Matemática.

[11] SANTOS, J. P. O. (1993) Introdução à Teoria dos Números.


IMPA.

[12] SANTOS, J. P. O.; MELLO, M. P. e MURARI, I. T. C.


(2006). Introdução à Análise Combinatória. Editora Unicamp.

[13] SOARES, M. G. (2005). Cálculo em uma Variável Complexa.


Sociedade Brasileira de Matemática.
Mestrado Profissional
em Matemática em Rede Nacional

Iniciação à Matemática

Autores:

Krerley Oliveira Adán J. Corcho

Unidade IV:

Capítulos VII e VIII


7
Desigualdades

Existem duas formas de fazer ótima Matemáti a. A primeira é ser

mais esperto que todo mundo. A segunda é ser mais estúpido que

todo mundo  mas persistente.

Raoul Bott

Neste capítulo estudaremos algumas desigualdades clássicas que

são usadas frequentemente na resolução de problemas matemáticos,

sendo estas aplicadas em contextos que variam desde o nível mais

simples até o mais complexo.

Uma vez que uma inequação em uma ou mais variáveis é resolvida,

o resultado dá lugar a uma desigualdade que é válida para um certo

conjunto de valores. Alguns exemplos simples de desigualdades são os

seguintes:

(a) x ≤ |x|, para qualquer −1 < x < 1;

(b) x2 < x, se x < 1;

(c) (x − y)2 ≥ 0, para quaisquer x e y reais;

x y
(d)
y
+ x
≥ 2, para quaisquer x, y > 0.

233
234 7 Desigualdades

7.1 Desigualdade Triangular

A desigualdade triangular arma o seguinte

Teorema 7.1 (Desigualdade Triangular). Dado um triângulo ABC o


comprimento de um dos lados é sempre inferior à soma dos compri-
mentos dos outros dois lados, ou seja,

AB < AC + CB, AC < AB + BC e BC < BA + AC.

A B

Figura 7.1: Desigualdade Triangular

Em outras palavras, a desigualdade triangular é a formulação ma-

temática da ideia intuitiva de que o caminho reto é mais curto entre

os pontos A e B.

Em analogia com a geometria plana temos uma versão da desigual-

dade triangular para números reais, que provamos a seguir.

Proposição 7.2. Sejam a e b números reais quaisquer, então


|a + b| ≤ |a| + |b|.

Demonstração. Se a + b ≥ 0, então |a + b| = a + b ≤ |a| + |b|. Caso

contrário, se a + b < 0, então |a + b| = −a − b ≤ |a| + |b|.


7.1 Desigualdade Triangular 235

Corolário 7.3. As seguintes desigualdades valem


|a − b| ≤ |a| + |b| (7.1)

|a − b| ≥ |a| − |b|, (7.2)



|a − b| ≥ |a| − |b| (7.3)

Demonstração. Para a primeira, escrevemos |a − b| = |a + (−b)| ≤


|a| + | − b| = |a| + |b|. A segunda desigualdade decorre de |a| =

|b + (a − b)| ≤ |b| + |a − b|. A última desigualdade é consequência da

segunda, trocando os papéis de a e b.

C
•O
A
•P

Figura 7.2: Problema da central de energia

Exemplo 7.4. Quatro cidades rurais, A, B , C e D, estão situadas


geogracamente formando um quadrilátero convexo. Deseja-se cons-
truir uma central de distribuição de energia para as quatro cidades de
modo que a soma total das distâncias da central a cada uma das quatro
cidades seja a mínima possível. Onde deverá ser construída a central?

Solução. Mostraremos que a central de energia deverá ser colocada

no ponto O de intersecção das diagonais do polígono ABCD. Com


236 7 Desigualdades

efeito, considerando um ponto P, diferente de O, (veja Figura 7.2) a

desigualdade triangular nos garante que

OA + OC = AC < P A + P C

OB + OD = BP < P B + P D,

de onde se segue que

OA + OC + OB + OD < P A + P C + P B + P D,

como esperávamos.

Exemplo 7.5. Duas torres de alturas h1 e h2 , respectivamente, estão


separadas a uma distância d. As torres são amarradas por uma corda
AP B que vai do topo A da primeira torre para um ponto P no chão,
entre as torres, e então até o topo B da segunda torre, como na Figura
7.3. Qual a posição do ponto P que nos dá o comprimento mínimo da
corda a ser utilizada?

A
B

Figura 7.3: Problema das Torres


7.1 Desigualdade Triangular 237

Solução. Imaginemos que a superfície do chão é um espelho e que re-

etimos o ponto através deste, obtendo assim o ponto B0 como mostra

a Figura 7.4.

A
B

C P D

P

B′

Figura 7.4: Solução geométrica do problema das torres

Consideremos o segmento AB 0 que intercepta o chão no ponto P


e para nossa surpresa vericaremos que este é o ponto que nos dá o

comprimento mínimo das cordas. Com efeito, suponhamos que existe

outro P0 situado entre as torres que nos dá um comprimento menor

para a corda, então da Figura 7.4 é fácil ver que os triângulos BP D


0 0
e B PD são congruentes, assim como os triângulos BP D e B0P 0D
também são congruentes. Logo, as seguintes igualdades seguem dire-

tamente das congruências:

BP = B 0 P e BP 0 = B 0 P 0 .

Agora, usando a desigualdade triangular no triângulo AB 0 P 0 e as igual-


dades acima, temos que

AP 0 + P 0 B = AP 0 + P 0 B 0 ≥ AB 0 = AP + P B 0 = AP + P B,
238 7 Desigualdades

chegando assim à conclusão de que AP + P B nos oferece o compri-

mento mínimo desejado.

Agora calcularemos a que distância está P da base D. Lembremos

que AC = h1 , BD = h2 e CD = d e observamos que

h2 h1
tang(]BP D) = = .
PD d − PD
dh2
Daí tem-se PD = .
h1 + h2

7.2 Desigualdade das Médias

Denição 7.6. Sejam a1 , a2 , . . . , an−1 e an números reais positivos.


As quantidades
n
mh (a1 , a2 , . . . , an ) = , (7.4)
1/a1 + 1/a2 + · · · + 1/an


mg (a1 , a2 , . . . , an ) = n
a1 a2 · · · an , (7.5)

a1 + a2 + · · · + an
ma (a1 , a2 , . . . , an ) = , (7.6)
n

r
a21 + a22 + · · · + a2n
mq (a1 , a2 , . . . , an ) = (7.7)
n
são chamadas, respectivamente, de média harmônica, média geomé-
trica, média aritmética e média quadrática dos números ai , i = 1, 2, . . . , n.
A seguir provaremos alguns resultados que estabelecem relações de

desigualdades entre as médias denidas acima.


7.2 Desigualdade das Médias 239

Proposição 7.7 (Desigualdade das Médias Aritmética e Quadrática).


Dados a1 , a2 , . . . , an números reais positivos tem-se
r
a1 + a2 + · · · + an a21 + a22 + · · · + a2n
≤ ,
n n
ou seja, ma (a1 , a2 , . . . , an ) ≤ mq (a1 , a2 , . . . , an ). Além disso, a igual-
dade vale se, e somente se, a1 = a2 = · · · = an .
Demonstração. Usando a igualdade

X n
X X
2
(ai − aj ) = (n − 1) a2i − 2 ai aj (7.8)
1≤i<j≤n i=1 1≤i<j≤n

concluímos que,

X n
X
2 ai aj ≤ (n − 1) a2i , (7.9)
1≤i<j≤n i=1

dado que o termo da esquerda em (7.8) é não negativo. Somando em


n
a2i obtemos
P
ambos os membros de (7.9) a quantidade
i=1

n
X 2 n
X
ai ≤ n a2i ,
i=1 i=1

donde, dividindo por n2 e tomando a raiz quadrada, segue-se a desi-

gualdade desejada. Por último, observamos que a igualdade em (7.9)

(ai − aj )2 = 0,
P
é atingida se, e somente se, o que é verdade se,
1≤i<j≤n
e somente se, a1 = a2 = · · · = an .

Proposição 7.8 (Desigualdade das médias Geométrica e Aritmética).


Dados a1 , a2 , . . . , an números reais positivos tem-se
√ a1 + a2 + · · · + an
n
a1 a2 · · · an ≤ ,
n
240 7 Desigualdades

ou seja, mg (a1 , a2 , . . . , an ) ≤ ma (a1 , a2 , . . . , an ). Além disso, a igual-


dade vale se, e somente se, a1 = a2 = · · · = an .

Demonstração. A prova desta desigualdade é mais técnica e exige um

pouco mais de esforço. Dividiremos a mesma em dois passos.

Passo 1. A desigualdade vale para n = 2m .


Procederemos por indução. Para n = 2 a desigualdade vale. De

fato,
√ √ √
( a1 − a2 )2 = a1 + a2 − 2 a1 a2 ≥ 0.
√ a +a √
Assim, a1 + a2 ≥ 2 a1 a2 e conseqüentemente 1 2 ≥ a1 a2 .
2
Agora provamos que se a desigualdade vale para n = k , então

também vale para n = 2k . Com efeito,

a1 + · · · + a2k a1 +···+ak
k
+ ak+1 +···+a
k
2k

=
2k √ 2 √
(1) k a1 · · · ak + k ak+1 · · · a2k

2
(2) q √ √
≥ k
a1 · · · ak k ak+1 · · · a2k

= 2k a1 · · · a2k ,

onde em (1) e (2) usamos a validade da desigualdade em para n=k


e para n = 2, respectivamente. Logo, como já provamos a validade
para n = 2, é claro que vale também para n = 4, 8, . . . , 2m , . . . , como
esperávamos.

Passo 2. Dado m inteiro positivo, então a desigualdade vale para


m
todo n<2 .

Para vericar isto, denimos o número


L= n
a1 · · · an ,
7.2 Desigualdade das Médias 241

e como a desigualdade vale para n = 2m , temos então que

a1 + · · · + an + L
| + ·{z
· · + L}
2m −n vezes 2m
p
≥ a1 · · · an · L2m −n
2m √
2m
= Ln · L2m −n = L.
Portanto,
a1 + · · · + an + (2m − n)L
≥ L,
2m
logo

a1 + · · · + an ≥ 2m L − (2m − n)L = nL,


obtendo assim que


a1 + · · · + an ≥ nL = n n a1 · · · an ,

o que nos dá a desigualdade desejada.

Como para qualquer inteiro positivo n sempre existe um inteiro


m
positivo m tal que n<2 , a desigualdade ca provada para todo n.
A prova de que a igualdade só ocorre quando a1 = a2 = · · · = an
pode também ser feita por indução e deixamos a cargo do leitor.

Proposição 7.9 (Desigualdade das Médias Harmônica e Geométrica).


Dados a1 , a2 , . . . , an números reais positivos tem-se
n √
≤ n a1 a2 · · · an ,
1/a1 + 1/a2 + · · · + 1/an
ou seja, mh (a1 , a2 , . . . , an ) ≤ mg (a1 , a2 , . . . , an ). Além disso, a igual-
dade vale se, e somente se, a1 = a2 = · · · = an .
Demonstração. Usando a Proposição 7.8 com os números ai substituí-

dos por 1/ai (i = 1, 2 . . . , n) vale que


n n
Y 1 1/n 1X 1
= mg (1/a1 , . . . , 1/an ) ≤ ma (1/a1 , . . . , 1/an ) = .
a
i=1 i
n i=1 ai
242 7 Desigualdades

Invertendo esta última desigualdade, obtemos então

mh (a1 , a2 , . . . , an ) ≤ mg (a1 , a2 , . . . , an ),

concluindo-se assim a prova. Notemos que as igualdades só ocorrem

se 1/a1 = 1/a2 = · · · = 1/an equivalem as igualdades a1 = a2 = · · · =


an .

O próximo resultado resume as relações provadas, nas proposições

7.7, 7.8 e 7.9, para as médias mh , mg , ma e mq .

Teorema 7.10 (Desigualdade das Médias). Para toda coleção de nú-


meros reais positivos a1 , a2 , . . . , an−1 e an se vericam as seguintes
desigualdades:

min(a1 , . . . , an ) ≤ mh (a1 , a2 , . . . , an )
≤ mg (a1 , a2 , . . . , an )
(7.10)
≤ ma (a1 , a2 , . . . , an )
≤ mq (a1 , a2 , . . . , an ) ≤ max(a1 , . . . , an ).

Além disso, em cada caso a igualdade ocorre se, e somente se, a1 =


a2 = · · · = an .

Exemplo 7.11. Num triângulo retângulo a altura relativa à hipote-


nusa é sempre menor ou igual que a metade da hipotenusa. Além
disso, a igualdade só ocorre quando o triângulo retângulo é isósceles
(ou seja, seus catetos são iguais).

Solução. Usando a Figura 7.5, temos que a hipotenusa c é dada por

c = x+y e usando o teorema das alturas para um triângulo retângulo


7.2 Desigualdade das Médias 243


temos que h2 = xy , logo h= xy. A desigualdade entre as médias

geométrica e aritmética nos dá que

√ x+y c
h= xy ≤ = ,
2 2
como queríamos. Além disso, a altura é a metade da hipotenusa se, e

a b
h
x y
c

Figura 7.5: Interpretação geométrica da desigualdade das médias geo-


métrica e aritmética

somente se, a igualdade entre as médias ocorre, ou seja, quando x = y.


Então, os catetos a e b do triângulo são iguais, sendo este isósceles.

Exemplo 7.12 (Desigualdade Isoperimétrica para Triângulos). O pe-


rímetro de um triângulo de lados a, b e c é a soma p = a + b + c.
Entre todos os triângulos com perímetro xado p o de maior área é o
triângulo equilátero.
Solução. Usando a Fórmula de Herón temos que a área de um triân-

gulo com perímetro p é dada pela expressão


q
p p
A= (
2 2
− a)( p2 − b)( p2 − c),

onde a, b e c são os lados do triângulo.

Usando agora a desigualdade mg ≤ ma temos que,


s  3
p p p
p −a+ −b+ −c p2
A≤ 2
2 2
3
2 = √ .
12 3
244 7 Desigualdades

2
p√
Logo a maior área possível é , a qual é atingida quando
12 3
p p p
2
−a= 2
−b= 2
− c ⇔ a = b = c,
ou seja, quando o triângulo é equilátero. Notemos que neste caso,
2

p√ a2 3
12 3
= 4
.

Exemplo 7.13 (Desigualdade Isoperimétrica para Paralelepípedos) .


Entre todos os paralelepípedos com área lateral xada A o de maior
volume é o cubo (ou seja, o paralelepípedo com todos seus lados iguais).

Figura 7.6: A área lateral de um paralelepípedo de lados a, b e c é dada


por AL = 2(ab + bc + ac).

Solução. Denotando por a, b e c as medidas das arestas do paralele-

pípedo sabemos que é a soma das áreas de todas as faces do paralele-

pípedo, ou seja,

AL = 2(ab + ac + bc).
Sendo V o volume do paralelepípedo e usando a desigualdade entre as

médias aritmética e geométrica temos que


 3  3
2 ab + ac + bc AL
V = ab · ac · bc ≤ = . (7.11)
3 6
q
AL 3

Assim, o maior volume possível é V = , obtido quando ab =
6
ac = bc, consequentemente a = b = c.
7.3 Desigualdade de Cauchy-Schwarz 245

7.3 Desigualdade de Cauchy-Schwarz

Teorema 7.14 (Desigualdade de Cauchy-Schwarz) . Dados a1 , . . . ,


an e b1 , . . . , bn números reais tem-se
q q
|a1 b1 + · · · + an bn | ≤ x1 + · · · + an b21 + · · · + b2n
2 2 (7.12)

Além disso, a igualdade só ocorre se existir um número real α, tal que


a1 = αb1 , . . . , an = αbn ou b1 = αa1 , . . . , bn = αan .

Demonstração. Usando a identidade de Lagrange:

n
X n
X n
X 2 X
a2i b2i = ai b i + (ai bj − aj bi )2
i=1 i=1 i=1 1≤i<j≤n

temos que
n
X 2 n
X n
X
ai b i ≤ a2i b2i ,
i=1 i=1 i=1

de onde se obtém diretamente a desigualdade de Cauchy-Schwarz.

Além disso, a igualdade ocorre se, e somente se,

X
(ai bj − aj bi )2 = 0 ⇐⇒ ai bj − aj bi = 0, 1 ≤ i < j ≤ n,
1≤i<j≤n

o que é verdade se, e somente se, existe α tal que ai = αbi ou bi = αai ,
com i = 1, 2, . . . , n.

Exemplo 7.15. Entre todos os triângulos retângulos de catetos a e b


e hipotenusa c xada, o que tem maior soma dos catetos s = a + b é
o triângulo isósceles.
Solução. Usando a desigualdade de Cauchy-Schwarz temos que

√ √ √
a+b=a·1+b·1≤ a2 + b2 12 + 12 = c 2
246 7 Desigualdades

e este máximo é atingido quando a = λ·1 e b = λ·1 ou 1 = λ·a e

1 = λ · b. Em qualquer caso devemos ter a = b.

Exemplo 7.16 (Desigualdade de Minkowski) . Dados ai , bi com 1 ≤


i ≤ n, números reais, tem-se
v v v
u n u n u n
u X u X uX
2
t (ai + bi )2 ≤ t ai + t b2i .
i=1 i=1 i=1

Solução. Partimos da seguinte igualdade:

n
X n
X n
X n
X
(ai + bi )2 = a2i + b2i + 2 ai b i . (7.13)
i=1 i=1 i=1 i=1

Aplicando a desigualdade de Cauchy-Schwarz no lado direito de (7.13)

temos que

v v
n
X n
X n
X
u n u n
uX uX
(ai + bi )2 ≤ a2i + 2
b + 2t
i
2
at b2 i i
i=1 i=1 i=1 i=1 i=1
v v 2 (7.14)
u n u n
uX uX
= t a2 + t b2  .
i i
i=1 i=1

Tomando raiz quadrada em ambos os membros de (7.14) obtemos a

desigualdade de Minkowski.

7.4 Desigualdade de Jensen

A Desigualdade de Jensen está estreitamente relacionada com o con-

ceito de convexidade, o qual explicamos a seguir.


7.4 Desigualdade de Jensen 247

Denição 7.17. Uma função f : [α, β] → R é dita convexa se para


quaisquer a, b ∈ [α, β] e para todo λ ∈ [0, 1] satisfaz


f λa + (1 − λ)b ≤ λf (a) + (1 − λ)f (b).

y
y = f (x)

•(b, f (b))

(a, f (a)) •

a b x

Figura 7.7: Gráco de uma função convexa

Geometricamente, a denição de convexidade signica que para

cada par de pontos a e b escolhidos no intervalo [α, β] o gráco da

função encontra-se abaixo do segmento de reta secante que junta os

pontos (a, f (a)) e (b, f (b)), como mostra a Figura 7.7.

Exemplo 7.18. A função f (x) = x2 é convexa em qualquer intervalo


[α, β].

Solução. Sejam a, b ∈ [α, β] e suponhamos, sem perda de generalidade,


248 7 Desigualdades

que a < b. Então, para todo λ ∈ [0, 1] valem as desigualdades:

(λa + (1 − λ)b)2 = λ2 a2 + (1 − λ)2 b2 + 2λ(1 − λ)ab


(1)
≤ λ2 a2 + (1 − λ)2 b2 + λ(1 − λ)(a2 + b2 ) (7.15)

= a2 [λ2 + λ(1 − λ)] + b2 [(1 − λ)2 + λ(1 − λ)]


= λa2 + (1 − λ)b2 ,

a2 +b2
onde na passagem (1) usamos a desigualdade ab ≤ 2
.

Exemplo 7.19. A função f (x) = 1/x é convexa em qualquer intervalo


[α, β] com α positivo.

Solução. Sendo a, b ∈ [α, β] com a < b, para todo λ ∈ [0, 1] tem-se

1 = (λ + (1 − λ))2
= λ2 + 2λ(1 − λ) + (1 − λ)2
(1) a b 
2
≤λ + + λ(1 − λ) + (1 − λ)2 (7.16)
b a
a b
= λ2 + λ(1 − λ) + λ(1 − λ) + (1 − λ)2
b a
 λ (1 − λ)   λ (1 − λ) 
= λa + +(1 − λ)b +
a b a b
  λ (1 − λ) 
= λa + (1 − λ)b +
a b
onde na passagem (1) usamos que a/b + b/a ≥ 2 para quaisquer nú-

meros positivos a e b. De (7.16) segue-se que

1 1 1
≤ λ + (1 − λ) ,
λa + (1 − λ)b a b
1
mostrando isto a convexidade da função
x
.
7.4 Desigualdade de Jensen 249

Observemos que, usando a desigualdade entre as médias aritmética

e quadrática obtemos

2
a1 + a2 + · · · + an a21 + a22 + · · · + a2n

≤ ,
n n

em outras palavras

(ma (a1 , a2 , . . . , an ))2 ≤ ma (a21 , a22 , . . . , a2n ). (7.17)

Por outro lado, a desigualdade entre as médias harmônica e aritmética

nos garantem que

1
≤ ma (1/a1 , 1/a2 . . . , 1/an ). (7.18)
ma (a1 , a2 , . . . , an )

O seguinte resultado garante que as propriedades (7.17) e (7.18), satis-


1
feitas pelas funções convexas x2 e
x
, são válidas para qualquer função

convexa.

Teorema 7.20 (Desigualdade de Jensen) . Seja f : [α, β] → R uma


n
função convexa e sejam λi ∈ [0, 1] (i = 1, . . . , n) tais que λi = 1.
X

i=0
Então, para quaisquer ai ∈ [α, β] (i = 1, . . . , n) vale

f (λ1 a1 + · · · + λn an ) ≤ λ1 f (a1 ) + · · · + λn f (an ). (7.19)

Observação 7.21. Observemos que, quando λ1 = λ2 = · · · = λn =


1/n, a desigualdade de Jensen nos diz que
 a + a + · · · + a  f (a ) + f (a ) + · · · + f (a )
1 2 n 1 2 n
f ≤ ,
n n
ou seja, f (ma (a1 , . . . , an )) ≤ ma (f (a1 ), . . . , f (an )).
250 7 Desigualdades

Demonstração. Faremos a prova por indução. Para n=2 a validade

decorre diretamente da denição. Suponhamos que dado n natural

(7.19) vale, então temos que provar a validade de

n+1  n+1
(7.20)
X X
f λ j aj ≤ λj f (aj ).
j=1 j=i
Notemos que
n+1
X n
X  n
X 
λj aj = λ j aj + 1 − λj an+1
j=1 j=1 j=1
(7.21)
n
X λj
=α aj + (1 − α)an+1 ,
j=1
α
n n
P P λj
onde α= λj . Assim, usando que α
= 1 e a hipótese de indução,
j=1 j=i
obtemos
n+1 n
X  X λ j

f λj aj ≤ αf aj +(1 − α)f (an+1 )
j=1 j=1
α
n
X λj
≤α f (aj ) + (1 − α)f (an+1 ) (7.22)
j=1
α
n+1
X
= λj f (aj ),
j=1

como queríamos provar.

7.5 Exercícios

1. Provar que em todo triângulo a soma dos comprimentos das

medianas é menor que o perímetro do triângulo e maior que o

semiperímetro deste.
7.5 Exercícios 251

2. Os centros de três círculos que não se intersectam estão sobre

uma reta. Prove que se um quarto círculo toca de forma tangente

os três círculos, então o raio deste é maior que pelo menos um

dos raios dos três círculos dados.


n
X 1 2n
3. Dado n inteiro positivo, provar que ≥ .
j=1
j n+1

4. A soma de três números positivos é 6. Provar que a soma de

seus quadrados não é menor que 12.

5. Determinar as dimensões do paralelepípedo de menor diagonal

possível, sabendo que a soma dos comprimentos de todas suas

arestas é 12.

6. Encontrar todas as soluções positivas do sistema de equações

não lineares 
x 2 + · · · + x 2 = 1
1 10
1 1
 2 + · · · + 2 = 100.
x1 x10

7. Demonstrar que, se a1 , a2 , . . . , an são números positivos tais que

a1 a2 · · · an = 1

então

(1 + a1 )(1 + a2 ) · · · (1 + an ) ≥ 2n .

8. Prove que a média geométrica é super-aditiva, isto é, para nú-

meros não negativos bi , 1 ≤ i ≤ n, tem-se


ai e

v v v
u n u n u n
uY uY uY
n
t ai + t
n
bi ≤ t
n
(ai + bi ).
i=1 i=1 i=1
252 7 Desigualdades

Além disso, estude em que condições ocorre a igualdade.

Sugestão: Use a desigualdade entre as médias geométrica e aritmética.

9. Usar o método de indução para provar a desigualdade de Cauchy-

Schwarz.
n
(ai + λbi )2 ≥ 0.
P
10. Para todo λ real Use este fato para dar outra
i=1
prova da desigualdade de Cauchy-Schwarz.

11. Use a desigualdade de Cauchy-Schwarz para dar uma prova alter-

nativa da desigualdade entre as médias aritmética e quadrática

(ma ≤ mq ).

12. Prove que ( n )


n n
X 1 X X
ai b i ≤ a2i + b2i .
i=1
2 i=1 i=1

13. Prove que a4 + b4 + c4 ≥ abc(a + b + c).

14. Prove que se a ≥ 0, b ≥ 0 e c ≥ 0, então

(a + b)(a + c)(b + c) ≥ 8abc.

15. Prove a desigualdade de Bernoulli: (1 + x)n > 1 + nx, para

qualquer x positivo e n inteiro positivo.

16. Prove que se a, b, c e d são inteiros positivos, então:


 
1 1 1 1
(a + b + c + d) + + + ≥ 16.
a b c d

17. Prove que se a ≥ 0, b ≥ 0 c ≥ 0, então


e
√ √ √
(ab + bc + ca) ≥ a bc + b ac + c ab.
7.5 Exercícios 253

18. Prove que se x ≥ 0, então 3x3 − 6x2 + 4 ≥ 0.


Sugestão: Use a desigualdade entre as médias aritmética e geométrica.


19. Prove que se x ≥ 0, então 2x + 3/8 ≥ 4 x.

20. Sejam C1 e C2 dois círculos concêntricos de raios r1 e r2 , res-

pectivamente, com r1 < r2 . Sobre o círculo C1 se marcam dois

pontos P1 e P2 diametralmente opostos. Deseja-se encontrar o

ponto P sobre o círculo C2 que maximiza a soma

d(P ) = P P1 + P P2 .
254 7 Desigualdades
8
Polinômios

A oisa mais bela que podemos ontemplar é o mistério. Isto é a

fonte da verdadeira arte e iên ia.

Albert Einstein

8.1 Operações com Polinômios

A necessidade de estudar equações polinomiais aparece em problemas

práticos da humanidade desde épocas muito remotas. Indícios arque-

ológicos indicam que os babilônicos já tinha o domínio de técnicas de

resolução de algumas equações do primeiro grau e do segundo grau,

apresentadas em forma de problemas cotidianos. Contudo, o grande

avanço teórico no estudo das equações polinomiais só se iniciou com o

Renascimento na Europa. No início do século XVI, Vièti introduziu o

uso de letras para representar quantidades desconhecidas.

Na mesma época, um outro grande desao estava perturbando as

mentes matemáticas de toda a Europa, em especial as da Itália. A

solução explícita utilizando as operações elementares (soma, subtra-

ção, multiplicação, divisão, radiciação e potenciação) da equação do

255
256 8 Polinômios

terceiro grau não era conhecida e muitos dos melhores matemáticos

da época trabalharam neste problema, destacando-se entre eles Ni-

colo Fontana, o Tartaglia (gago, em italiano). A história da solução

desta equação está repleta de intrigas, disputas e acusações, envol-

vendo Tartaglia e Cardano. Hoje os historiadores atribuem a Tarta-

glia a primazia na descoberta da solução da equação do terceiro grau

como conhecemos. É desta época também a solução da equação do

quarto grau, atribuída a Ludovico Ferrari.

Entretanto, apesar dos muitos esforços empreendidos na direção de

encontrar a solução geral da equação do quinto grau, mais de 200 anos

se passaram sem nenhum sucesso. Até que em 1824, o matemático

norueguês Niels Abel mostrou que é impossível resolver as equações de

grau cinco em sua forma geral. Ou seja, nem todas as equações de grau

cinco podem ser resolvidas com as operações elementares. Mais ainda,

em 1830 o matemático francês Evariste Galois descobriu um método

que determina quando uma equação de grau qualquer é resolúvel com

as operações elementares, encerrando um belíssimo capítulo do estudo

das equações polinomiais e da Matemática.

Neste capítulo iremos estudar um pouco mais formalmente os poli-

nômios e suas propriedades.

Denição 8.1. Um polinômio na variável x é uma expressão do tipo


p(x) = an xn + an−1 xn−1 + · · · + a1 x + a0

onde a0 , a1 , . . . , an são números. Se an 6= 0, dizemos que n é o grau


do polinômio e a0 , a1 , . . . , an são seus coecientes. O coeciente an é
chamado de coeciente líder do polinômio.
Observação 8.2. Não se dene o grau do polinômio nulo, que tem
todos os coecientes iguais a zero.
8.1 Operações com Polinômios 257

Por exemplo,

• p(x) = 3x − 1 é um polinômio de grau 1;

• q(x) = 4x3 + 7x + 1 é um polinômio de grau 3;

π 4
• t(x) = x é um monômio de grau 4;
2
π
• v(x) = − x4 + 5x2 + 1 é um polinômio de grau 4;
2
• u(x) = 7 é um polinômio de grau 0.

Uma equação polinomial de graun, ou simplesmente uma equação


de grau n, é uma sentença p(x) = 0, onde p(x) é um polinômio de

grau n com coecientes reais. Por exemplo, 2x − 1 = 0 é uma equação


5 3
do primeiro grau, enquanto −x + 4x + 5x − 1 = 0 é uma equação de

grau 5. Note que nem todos os coecientes precisam ser diferentes de

zero.

Para obtermos o valor p(x) = an xn + an−1 xn−1 +


do polinômio

· · · + a1 x + a0 no número real r, devemos substituir x por r para obter


o número real

p(r) = an rn + an−1 rn−1 + · · · + a1 r + a0 .

Por exemplo, o valor do polinômio p(x) = 4x3 − 7x + 1 em 2 é p(2) =


4 · 23 − 7 · 2 + 1 = 19.
Dizemos que um número real r é uma raiz para a equação

an xn + an−1 xn−1 + · · · + a1 x + a0 = 0

se o valor de p(x) = an xn + an−1 xn−1 + · · · + a1 x + a0 em r é zero, ou

seja, se r verica

an rn + an−1 rn−1 + · · · + a1 r + a0 = 0.
258 8 Polinômios

Por exemplo, 5 é raiz da equação:

2x − 10 = 0.

Uma das vantagens dos polinômios sobre outros objetos matemá-

ticos é que podemos denir as operações de soma de polinômios e

multiplicação de polinômios. Com estas operações, o conjunto dos po-

linômios possui muitas propriedades similares à dos números inteiros,

tornando prático o seu uso.

Vamos denir agora o que signica a soma de dois polinômios.

Para isso, vamos começar somando dois monômios e depois estender

nossa denição para polinômios em geral.

Para somar dois monômios de mesmo grau p(x) = ak xk e q(x) =


bk xk somamos seus coecientes, obtendo o polinômio t(x) = p(x) +
q(x) = (ak +bk )xk . Em geral, para somar o polinômio p(x) = a0 +a1 x+
a2 x2 + · · · + an xn com o polinômio q(x) = b0 + b1 x + · · · + bm xm , onde
n ≤ m devemos somar todos os monômios de mesmo grau, obtendo o
polinômio:

t(x) = p(x) + q(x) = c0 + c1 x + · · · + cm xm

onde, c i = ai + b i para 0≤i≤n e ci = b i para i > n.


Por exemplo, sendo

• p(x) = 3x − 1,

• q(x) = 4x3 + 7x + 1,

• t(x) = π2 x4 ,

• v(x) = − π2 x4 + 5x2 + 1
8.1 Operações com Polinômios 259

temos que

• p(x) + q(x) = 4x3 + (3 + 7)x − 1 + 1 = 4x3 + 10x,

• v(x) + t(x) = ( π2 − π2 )x4 + 5x2 + 1 = 5x2 + 1.

A seguir, enumeramos algumas propriedades simples e importantes

da soma de polinômios que decorrem da denição dada e das propri-

edades análogas válidas para os números reais.

1. Associatividade . Dados polinômios p(x), q(x) e t(x), vale

(p(x) + q(x)) + t(x) = p(x) + (q(x) + t(x))

2. Elemento neutro . Se 0 denota o polinômio nulo e p(x) é um

polinômio qualquer, então

0 + p(x) = p(x).

3. Elemento simétrico . Se p(x) = a0 + a1 x + · · · + an xn é um


n
polinômio, então o polinômio q(x) = −a0 − a1 x − · · · − an x

satisfaz:

p(x) + q(x) = 0.

4. Comutatividade . Se p(x) e q(x) são polinômios, então

p(x) + q(x) = q(x) + p(x).

Note que os números inteiros possuem propriedades similares para

a operação de soma de números inteiros. Vamos agora denir o produto


de dois polinômios. Para isso, vamos primeiramente denir o produto

de dois monômios, como já zemos no caso de soma de polinômios.


260 8 Polinômios

Se n, m são números naturais, denimos o produto dos monômios

p(x) = an xn e q(x) = bm xm como:

p(x)q(x) = an bm xn+m .

Tendo isto em mente, para efetuarmos o produto do polinômio de

grau n, p(x) = a0 + a1 x + a2 x2 + · · · + an xn pelo polinômio q(x) =


b0 + b1 x + · · · + bm xm de grau m, com n ≤ m, devemos:

• Completamos a escrita de p(x) e de q(x) até o termo n + m


colocando ak = 0 para k > n e bk = 0 para k > m;

• Denimos

t(x) = p(x)q(x) = c0 + c1 x + · · · + cn+m xn+m

onde, ci = a0 bi + a1 bi−1 + · · · + ai−1 b1 + ai b0 para 0 ≤ i ≤ n + m.

Apesar de parecer complicada, a denição não é tão difícil de ser

aplicada. Para tentar visualizar o processo de multiplicação de dois

polinômios vamos pensar que os monômios são seres alienígenas vin-

dos do distante planeta de Algebrum e possuam mãos. Quando dois

monômios se encontram, invariavelmente eles apertam as mãos e desse

aperto aparece o produto desses monômios.

Assim, para multiplicar os polinômios p(x) e q(x), que são forma-

dos por dois grupos de monômios, devemos escolher o primeiro monô-

mio de p(x) e fazê-lo apertar a mão de cada um dos monômios de

q(x), somando os monômios obtidos. Após isso, tomamos o segundo

monômio de p(x) e fazemos ele apertar a mão de cada um dos monô-

mios deq(x), somando os monômios obtidos aos monômios anteriores.


Repetimos o processo até o último monômio de p(x).
8.1 Operações com Polinômios 261

Deste modo, se p(x) = x2 + 2x − 3 e q(x) = −x2 + 5x + 1, para

obter p(x)q(x) fazemos:

p(x)q(x) = −x4 + 5x3 + x2 − 2x3 + 10x2 + 2x + 3x2 − 15x − 3


= −x4 + 3x3 + 14x2 − 13x − 3.

Observe que com a denição de multiplicação de polinômios dada

acima, o coeciente c0 é igual a a0 b 0 . Do mesmo modo, o coeciente


n+m
do termo x cn+m = an bm . Como p(x) tem grau n (isto é, an 6= 0)
é

e q(x) tem grau m (bm 6= 0), o coeciente cn+m = an bm 6= 0. Logo,


o polinômio p(x)q(x) tem grau n + m. Com isso, demonstramos o

seguinte fato:

Proposição 8.3. Se o polinômio p(x) tem grau n e o polinômio q(x)


tem grau m, então o polinômio p(x)q(x) tem grau n + m.

Um caso particular interessante é quando multiplicamos um núme-

ro c, que podemos considerar como sendo um polinômio de grau zero


q(x) = c, por um polinômio p(x) = a0 + a1 x + · · · + an xn . Neste caso,
nós obtemos o polinômio

cp(x) = ca0 + ca1 x + · · · + can xn .

Do mesmo modo em que podemos vericar as propriedades da

soma de polinômios a partir das propriedades similares dos números

reais, podemos também vericar as propriedades abaixo sobre a mul-

tiplicação de polinômios. Deixamos essa vericação como exercício.

1. Associatividade . Dados polinômios p(x), q(x) e t(x), vale

(p(x)q(x))t(x) = p(x)(q(x)t(x))
262 8 Polinômios

2. Elemento neutro . Se 1 denota o polinômio constante e p(x) é

um polinômio qualquer, então

1p(x) = p(x).

3. Comutatividade . Se p(x) e q(x) são polinômios, então

p(x)q(x) = q(x)p(x).

4. Distributividade . Se p(x), q(x) e t(x) são polinômios, então

(p(x) + q(x))t(x) = q(x)t(x) + p(x)t(x).

Note que, assim como nos inteiros, a propriedade de existência de

elementos inversos para a multiplicação de polinômios não vale. De

fato, podemos vericar que se p(x) é um polinômio de grau n maior ou


igual a um, então não existe um polinômio q(x) tal que p(x)q(x) = 1.

De fato, suponha por absurdo, que exista q(x) um polinômio com grau

m ≥ 0 tal que
p(x)q(x) = 1.

Então, utilizando a Proposição 8.3 temos que o grau de p(x)q(x) é

n+m que é maior ou igual que um. Como o grau do polinômio

constante 1 é zero, temos que a igualdade acima não pode valer, onde

chegamos a um absurdo.

Em resumo, os únicos polinômios que podem ter inversos com res-

peito à operação de multiplicação são os polinômios constantes não

nulos. Esta é mais uma das semelhanças entre os inteiros e os polinô-

mios.
8.2 Algoritmo de Euclides 263

8.2 Algoritmo de Euclides

Diremos que um polinômio a(x) divide o polinômio b(x) se existir q(x)


tal que b(x) = q(x)a(x).
Por exemplo, o polinômio a(x) = x2 + x + 1 divide o polinômio

x3 − 1 pois

(x − 1)(x2 + x + 1) = x3 − 1.

Devido à Proposição 8.3, se o polinômio a(x) divide o polinômio

não nulo b(x), então o grau de a(x) é menor ou igual ao grau de b(x).
Agora, vamos enunciar um fato que vale para os inteiros e que vale

também para os polinômios e que será de grande utilidade. Pedimos

que o leitor releia o algoritmo de Euclides, estudado no Capítulo 3.

No conjunto dos polinômios, ainda vale

Teorema 8.4 (Algoritmo de Euclides). Sejam a(x) e b(x) dois polinô-


mios com coecientes reais, b(x) 6= 0. Então, existem polinômios com
coecientes reais q(x) e r(x), com r(x) = 0 ou grau de r(x) menor
que o grau de b(x) tais que:

a(x) = b(x)q(x) + r(x).

Além disso, q(x) e r(x) estão determinados de modo único.

Demonstração. Vamos mostrar primeiro a unicidade. De fato, assuma

que

a(x) = b(x)q1 (x) + r1 (x) = b(x)q2 (x) + r2 (x),

com r1 e r2 de graus menores que o grau de b. Assim,

b(q1 − q2 ) = r2 − r1 .
264 8 Polinômios

Consequentemente, q1 = q2 , já que caso contrário, o polinômio b(q1 −


q2 ) teria grau pelo menos igual ao grau de b e o polinômio r2 − r1 tem
grau menor que o grau de b.
Vamos agora mostrar a existência. Os passos da prova são idênticos

a prova do algoritmo de Euclides para números inteiros, demonstrado

no Capítulo 3. De fato, a ideia é reduzir o grau do dividendo até

que ele se torne menor que o do divisor e a divisão se torne imediata.

Note que se a tem grau menor que b, então tomamos o resto com
sendo r = a e o quociente como sendo q = 0. Suponhamos que

a(x) = an xn +· · ·+a1 x+a0 tenha grau n e b(x) = bm xm +· · ·+b1 x+b0


tenha grau m e que n > m. Dena

an n−m
c1 (x) = a(x) − x b(x).
bm
Observe que o grau de c1 é no máximo n − 1. Se c1 puder se dividido
por b, digamos com c1 (x) = b(x)q(x) + r(x), com grau de r(x) menor
que o grau de b(x), então

an n−m an
a(x) = b(x) x + c1 (x) = b(x)( xn−m + q(x)) + r(x).
bm bm
Logo, reduzimos o problema de dividir o polinômio a(x) por b(x) pelo
problema de dividir o polinômio c1 (x) por b(x), com c1 (x) de grau

menor que a(x). Repetimos o processo, utilizando c1 no lugar de a(x),

obtendo o polinômio c2 (x) de grau menor que o de c1 (x). Como a cada

passo reduzimos o grau do dividendo em pelo menos uma unidade, ao

m de no máximo n − m passos, obteremos um polinômio com grau


menor que o grau de b(x), que é claramente divisível por b(x). Proce-

dendo como antes, achamos q(x) e r(x) tais que a(x) = b(x)q(x)+r(x)

e r(x) com grau menor que o grau de b(x).


8.2 Algoritmo de Euclides 265

Por exemplo, se a(x) = 10x3 − 3x + 2 e b(x) = x2 + 1, tomando

q(x) = 10x e r(x) = −13x + 2 temos que

10x3 − 3x + 2 = (x2 + 1)10x + (−13x + 2).

Note que o grau de r(x) = −13x + 2 é menor que o grau de b(x) =


x2 + 1 .
Se na expressão do polinômio p(x) decidimos substituir a variável

x por um número real s, estaremos avaliando o polinômio p(x) em s


e denotamos este número por p(s).
2
Por exemplo, se p(x) = x + 3x + 1, então substituindo x por 2,
temos que

p(2) = 22 + 3 · 2 + 1 = 11

e fazendo x = −3

p(−3) = (−3)2 + 3 · (−3) + 1 = 1.

Quandop(s) = 0 dizemos que s anula o polinômio não nulo p(x),


ou ainda, que s é uma raiz do polinômio p(x).
3
Por exemplo, para p(x) = x − 8, temos que 2 é uma raiz de p(x)
3
já que p(2) = 2 − 8 = 0.

Um fato muito importante que é consequência do algoritmo de

Euclides é o seguinte teorema:

Teorema 8.5. Se s é uma raiz do polinômio p(x), então o polinômio


x − s divide p(x). Reciprocamente, se x − s divide p(x), então s é raiz
de p(x).

Demonstração. Primeiramente, assuma que x − s divida p(x). Neste


caso, existe um polinômio q(x) tal que p(x) = q(x)(x − s). Avaliando
266 8 Polinômios

o polinômio p(x) em s, temos que:

p(s) = q(s)(s − s) = q(s) · 0 = 0.

Logo s é uma raiz de p(x).


Para provar que se s é uma p(x) então x − s divide p(x),
raiz de

vamos utilizar o algoritmo da divisão, com a(x) = p(x) e b(x) = x − s.

Neste caso, temos que existem q(x) e r(x) de modo que r(x) = 0 ou o

grau de r(x) é menor que o grau de x − s e além disso vale

p(x) = q(x)(x − s) + r(x).

Observe que, com as condições do resto r(x), podemos escrever que


r(x) = c ∈ R. Então, p(x) = q(x)(x−s)+c e 0 = p(s) = q(s)·0+c = c.
Portanto, r(x) = 0 e p(x) = q(x)(x − s), isto é, x − s divide p(x).

A proposição anterior nos permite determinar o número máximo

de raízes reais de um polinômio não nulo. De fato, vamos mostrar.

Proposição 8.6. O número máximo de raízes reais do polinômio não


nulo p(x) = an xn + an−1 xn−1 + · · · + a1 x + a0 é n.

Demonstração. Digamos que s0 < s1 < s2 < · · · < sk sejam raízes

distintas do polinômio p(x). Observe que podemos utilizar a Propo-

sição 8.5 para garantir que existe um polinômio não nulo q1 (x) tal

que

p(x) = q1 (x)(x − s0 ).
Assim, pela Proposição 8.3, o grau de q1 (x) deve ser igual a n − 1.
Note que p(si ) = q1 (si )(si − s0 ). Como para todo i = 1, 2, . . . , k temos
que si > s0 com p(si ) = 0, temos que, necessariamente, q1 (si ) = 0.

Assim, em particular, temos que q1 (s1 ) = 0. Logo, podemos aplicar


8.2 Algoritmo de Euclides 267

a proposição novamente para obter que existe um polinômio não-nulo

q2 (x) tal que

q1 (x) = q2 (x)(x − s1 ).

Assim, como o grau de q1 (x) é n − 1, pela Proposição 8.3, o grau de


q2 (x) deve ser igual a n − 2.
Novamente, temos que q1 (si ) = q2 (si )(si − s1 ), si > s1 e p(si ) = 0

para todo i = 2, . . . , k . Disto segue que, necessariamente, q2 (si ) = 0,

se i = 2, 3, . . . , k. Assim, temos que q2 (s2 ) = 0.

Logo, podemos repetir esse argumento para obter um polinômio

q3 (x) de grau n − 3, de modo que s3 , s4 , . . . , sk q3 (x). Re-


são raízes de

petindo o argumento, encontramos uma sequência q1 (x), q2 (x), q3 (x), . . .

com graus no máximo n − 1, n − 2, n − 3, . . . o que nos leva a con-

cluir que não podemos repetir esse argumento mais que n vezes, já

que os graus dos polinômios q1 (x), q2 (x), q3 (x), . . . estão diminuindo.

Ou seja, não podemos ter mais que n raízes para o polinômio p(x), o

que conclui a prova.

Alertamos que, apesar da Proposição 8.6 nos garantir que existem

no máximo n raízes reais de um polinômio de grau n não nulo, existem


polinômios que não possuem raízes reais. Por exemplo, p(x) = x + 1
2

2
não possui raízes rais, já que x ≥ 0 para todo número real x.

Uma consequência da Proposição 8.6 é a seguinte:

Proposição 8.7. Se dois polinômios p(x) e q(x) de grau n avaliados


em n + 1 números r1 , r2 , . . . , rn+1 coincidem, isto é, p(ri ) = q(ri ) para
i = 1, 2, 3, . . . , n + 1, então p(x) e q(x) são iguais.

Demonstração. Considere o polinômio t(x) = p(x) − q(x). Observe

que se t(x) é não-nulo, o grau de t(x) é no máximo n, já que p(x)


268 8 Polinômios

e q(x) têm graus iguais a n. Observe ainda que t(ri ) = 0, já que

p(ri ) = q(ri ) e
t(ri ) = p(ri ) − q(ri ) = 0.

Logo, t(x) tem grau no máximo n e mais de n raízes, contradizendo a

Proposição 8.6.

No Exercício 23 faremos uma aplicação interessante dessa propo-

sição, propondo que você prove que dados números reais a1 , a2 , . . . ,


an+1 e r1 , r2 , . . . , rn+1 , então existe um único polinômio de grau n tal

que p(ri ) = ai .

8.3 Sempre Existem Raízes de um Polinômio?

Pode parecer frustrante o fato de que um polinômio com coecientes

reais pode não possuir raízes reais. Por exemplo, quando tentamos

aplicar a fórmula de Bhaskara à equação x2 + 1 = 0, encontramos

∆ = −4 e, consequentemente, se fosse possível escrever as soluções,

elas se escreveriam como √


−4
x1 =
2
e √
−4
x2 = −
2
É claro que as expressões acima não têm sentido no conjunto dos

números reais, pois não existe número cujo quadrado seja −4, ou seja,
não é possível extrair a raiz quadrada de −4. Isso tirou o sono de

várias gerações de matemáticos. Desde Herón de Alexandria há dois

mil anos atrás, os matemáticos encontram expressões como a do tipo

acima, envolvendo raízes de números negativos.


8.3 Sempre Existem Raízes de um Polinômio? 269

A primeira reação da comunidade matemática foi rejeitar esses

números complexos e simplesmente desconsiderar raízes de números

negativos. Porém, já no século XVI, Cardano se deu conta de que os

números complexos surgem naturalmente quando desejamos resolver

uma equação do terceiro ou quarto grau, mas relutava quanto ao seu

uso, dizendo que esses números eram tão sutis, quanto inúteis.

No século seguinte, motivado pela sugestão de Albert Girard que

uma equação de grau n possui n raízes, Reneé Descartes observou que


os números reais eram insucientes para representar todas essas raízes

e utilizou o termo imaginárias para as raízes que não são reais.



A notação tradicional i = −1 só veio a ser introduzida um século
mais tarde, com Leonard Euler, que também é o pai do termo número
complexo. Euler e o matemático francês Jean D'Alambert zeram apli-

cações dos números complexos a problemas práticos, como projeção

de mapas e hidrodinâmica. Euler e Lagrange, grandes matemáticos

da história da humanidade, tentaram mostrar a armação de Girard,

de que uma equação de grau n possui n raízes, mas sem sucesso. A

primeira prova correta de tal teorema só apareceu no nal do século

XVIII com os trabalhos de Gauss.

8.3.1 Números Complexos e Raízes de Polinômios


O conjunto dos números complexos, denotado pela letra C, é o con-

junto das expressões

C = {x + iy; x, y ∈ R},

onde i satisfaz i2 = −1. Costuma-se denotar i por −1. Destacamos

que i é meramente um símbolo que nos ajudará a denir as operações

de soma e de multiplicação de números complexos. Essas operações


270 8 Polinômios

terão as mesmas propriedades que as operações de números reais, como

associatividade, comutatividade, elemento neutro, etc. Por exemplo,

são números complexos 2 − 3i, 3 + i e −3i.


Vamos denir a soma e multiplicação de números complexos. Da-

dos dois números complexos a + bi e c + di denimos a soma como:

(a + bi) + (c + di) = (a + b) + (c + d)i

e denimos a multiplicação como

(a + bi)(c + di) = (ac − bd) + (bc + ad)i

Por exemplo, se tomamos os números 2 − 3i e 3 + 4i então

(2 − 3i) + (3 + 4i) = 5 + i

(2 − 3i)(3 + 4i) = (2 · 3 − (−3 · 4)) + (−3 · 3 + 2 · 4)i = 18 − i.

Aqui nós estamos considerando 0 + 3i = 3i e 3 + 0 · i = 3. Isso

nos permite colocar os números reais dentro do conjunto dos números

complexos, considerando cada número real r como sendo um número

complexo da forma r + 0 · i.
Fica para o leitor a vericação de que valem as propriedades de

associatividade, comutatividade, etc. O elemento neutro da soma é o

elemento 0+0·i que simplesmente denotaremos por 0. Do mesmo

modo, o elemento neutro da multiplicação é 1+0·i, que será denotado


por 1. O leitor curioso pode achar mais informações sobre números

complexos e soluções de equações algébricas em [5] ou [13].


8.3 Sempre Existem Raízes de um Polinômio? 271

Assim, dado um número complexo z faz sentido avaliar o polinômio


(de coecientes complexos ou reais) p(x) = an xn + an−1 xn−1 + · · · +
a1 x + a0 em z, obtendo o número complexo

p(z) = an z n + an−1 z n−1 + · · · + a1 z + a0 .

Por exemplo, se p(x) = x2 + 4, então 2i e −2i são raízes deste

polinômio, já que:

p(2i) = (2i)2 + 4 = −4 + 4 = 0.

p(−2i) = (−2i)2 + 4 = 4i2 + 4 = −4 + 4 = 0.

Note que p(x) não possui nenhuma raiz real, mas possui duas raízes
complexas. Como já mencionamos, a grande vantagem em utilizar os

números complexos em vez dos números reais é que, dado um polinô-

mio qualquer com coecientes complexos, ele sempre tem uma raiz

complexa. Isso foi o assunto da tese de doutorado do Príncipe da


Matemática, Johann Carl Friedrich Gauss (1777-1855).

Teorema 8.8 (Teorema Fundamental da Álgebra) . Todo polinômio


não constante com coecientes complexos de grau n possui exatamente
n raízes complexas, contadas com multiplicidade.

Uma demonstração do Teorema Fundamental da Álgebra foge do

objetivo deste livro. Podem ser dadas várias demonstrações diferen-

tes desse teorema, utilizando diversas teorias matemáticas avançadas.

Uma demonstração desse teorema pode ser achada em [13].


272 8 Polinômios

8.4 Exercícios

1. Calcule o quociente e o resto da divisão de p(x) por q(x) para

os polinômios p(x) e q(x) dados:

(a) p(x) = 3x3 − 2x + 1 e q(x) = −7x − 1;


(b) p(x) = x5 − 1 e q(x) = x − 1;
(c) p(x) = 3x5 − 2x3 + 1 e q(x) = x2 + x + 1

2. Encontre os valores de A e B de forma que

x+1 A B
= + .
x −x
2 x x−1

3. Se os polinômios x2 −x+4 e (x−a)2 +(x+b) são iguais, encontre


a + b.

4. Quais os valores de a e b que tornam iguais os polinômios


2
P1 (x) = x − x − 6 e P2 (x) = (x + a)2 − b?

5. A divisão de P (x) por x4 + 1 tem quociente x+2 e resto 1.

Encontre o polinômio P (x).

6. Qual o resto da divisão do polinômio x100 por x + 1?

7. Determine o resto da divisão do polinômio p(x) pelo polinômio

g(x) = x, onde p(x) = (x − 1)(x − 2) . . . (x − n) + b .

8. Mostre que xn − 1 é divisível por x−1 para todo n ≥ 1.

9. Faça os seguintes itens:

(a) encontre o quociente da divisão de xn+1 − 1 por x − 1;


8.4 Exercícios 273

(b) utilize a divisão anterior para calcular a soma 1 + x + x2 +


x3 + · · · + xn dos n primeiros termos de uma progressão

geométrica de razão x.

10. Determine o valor de a para que o polinômio P (x) seja divisível

por x − a, onde P (x) = x3 + (1 − a)x2 + (1 + a)x − 1.

11. Mostre que o polinômio P (x) = x100 − 2x50 + 1 é divisível por


2
x − 1.

12. Mostre que o resto r(x) da divisão do polinômio p(x) por x−s
é r(x) = p(s).
Dado o polinômio p(x) = an xn + an−1 xn−1 + · · · + a1 x + a0
denimos a derivada de p(x) como sendo o polinômio:

p0 (x) = nan xn−1 + (n − 1)an−1 xn−2 + · · · + 2a2 x + a1 .

Por exemplo, a derivada do polinômio x5


5x4 e a
é o polinômio
3 2 2
derivada do polinômio x +5x +2x−1 é o polinômio 3x +10x+2.

13. Usando as informações do Exercício 12, calcule:

(a) a derivada dos polinômios:

(i) x + 1;
(ii) x4 + 3 ;
(ii) 1 + x + x2 + x3 + · · · + xn .
(b) Sabendo que p(0) = 1, calcule também o polinômio p(x)
cuja derivada é

(i) x4 .
(ii) −x2 + 1.
274 8 Polinômios

(ii) x3 + 2x2 + 3.
(c) Prove que se p(x) e q(x) são polinômios, então

(i) (p + q)0 (x) = p0 (x) + q 0 (x)


(ii) (pq)0 (x) = p0 (x)q(x) + p(x)q 0 (x)
Sugestão: Faça primeiro para monômios.

Denimos uma raiz múltipla de um polinômio p(x) como sendo


2
uma raiz a tal que (x − a) divide p(x). Caso a seja uma raiz
que não é raiz múltipla, dizemos que ela é raiz simples.

14. Mostre que a é raiz múltipla de um polinômio p(x) se, e somente


0
se, a é raiz de p(x) e de p (x).

Sugestão: Use o exercício anterior.

15. Para quais valores de n∈N tem-se que

(a) 1 + x2 + x4 + . . . + x2n−2 é divisível por 1 + x + . . . + xn−1 ?


(b) 1 + x3 + x6 + . . . + x3n−3 é divisível por 1 + x + . . . + xn−1 ?
(c) Generalize.

16. (a) Resolva a equação 20x3 − 30x2 + 12x − 1 = 0, sabendo-se


1
que é uma de suas raízes.
2
(b) Uma raiz da equação x3 −(2a+1)x2 +a(a+2)x−a(a+1) = 0
é a + 1, ache as outras duas.

17. Ache os possíveis valores de a∈Z para que o polinômio

a2 x4 + 4x3 + 4ax + 7

seja divisível por x + 1.


8.4 Exercícios 275

Um polinômio com coecientes reais não constante p(x) é dito ir-


redutível se p(x) = a(x)b(x), então a(x) ou b(x) são polinômios
constantes. Quando p(x) não for irredutível, diremos simples-

mente que ele é redutível. Os polinômios irredutíveis desempe-

nham papel análogo no conjunto dos polinômios ao dos números

primos em Z.

18. Prove que todo polinômio de grau 1 é irredutível.

19. Prove que se f (x) é um polinômio de grau ≥ 2 e possui uma raiz


real, então f (x) é redutível.

20. Mostre que todo polinômio f (x) de grau ímpar ≥3 é redutível.

Um polinômio com coecientes inteiros não constante p(x) é dito


irredutível sobre Q se p(x) = a(x)b(x) com a(x) e b(x) polinômios
com coecientes racionais, então a(x) ou b(x) são polinômios

constantes.

Um teorema importante que descreve uma condição para um

polinômio ser irredutível sobre Q é o conhecido critério de Ei-

senstein, que diz:

Teorema 8.9 (Critério de Eisenstein) . Seja f (x) = a0 + a1 x +


· · · + an xn um polinômio com coecientes inteiros. Suponha que
exista um primo p tal que:

(a) p - an ;
(b) p | a0 , p | a1 , . . . , p | an−1 ;
(c) p2 - a0 .

Então, f (x) é irredutível sobre Q.


276 8 Polinômios

Para uma prova desse resultado veja o livro [2]. Faça os seguintes

problemas:

21. Mostre que os seguintes polinômios f (x) são irredutíveis sobre

Q.
Sugestão: Use o critério de Eisenstein .

(a) f (x) = x4 + 2x3 + 2x2 + 2x + 2;


(b) f (x) = x6 + 15;
(c) f (x) = x4 + 10x3 + 20x2 + 30x + 22.

22. Determine quais dos polinômios abaixo são irredutíveis sobre Q.


Sugestão: Use o critério de Eisenstein .

(a) x3 − x + 1
(b) x3 + 2x + 10
(c) x4 − x + 1

O problema a seguir trata do polinômio de interpolação de La-

grange.

23. Demonstre a proposição a seguir:

Polinômio de Interpolação de Lagrange. Sejam ai , bi em


0 0
R, i = 1, 2, . . . , n, com os ai s dois a dois distintos e os bi s nem

todos nulos. Considere os polinômios

(x − ai ) · · · (x − ai−1 )(x − ai+1 ) · · · (x − an )


pi (x) = bi
(ai − a1 ) · · · (ai − ai−1 )(ai − ai+1 ) · · · (ai − an )
8.4 Exercícios 277

para i = 1, 2, . . . , n. Então, o polinômio

n
X
p(x) = pi (x)
i=1

é o único polinômio de grau menor que n, tal que p(ai ) = bi ,


para todos i = 1, 2, . . . , n.

24. Determine o polinômio p(x) de grau 7 tal que

p(1) = p(2) = · · · = p(7) = 8 e p(0) = 1.


278 8 Polinômios
A
Apêndice: Funções

Estamos acostumados a expressões cotidianas que retratam uma

relação entre grandezas, como por exemplo, o quanto João ganha é


função do que ele trabalha, ou ainda a distância que percorremos é
uma função da velocidade e do tempo que viajamos. Essas e outras
expressões ilustram a noção de função como uma relação entre grande-

zas de dois conjuntos dados. Matematicamente, a noção de função foi

melhor entendida muito recentemente, com os avanços teóricos ocorri-

dos no nal do século XIX e início do século XX. Entretanto, o seu uso

como instrumento e os estudos para tornar sua denição um objeto

claro são bem antigos e datam pelo menos desde o início do cálculo

diferencial, onde a noção de função era por vezes entendida como sua

expressão analítica. O entendimento dessa noção foi crucial para o

avanço da Matemática e é importante que o estudante de Matemática

tenha claro seu signicado.

Para iniciar a discussão um pouco mais formalmente da noção

de função, vamos denir intuitivamente uma função como um objeto

matemático composto de três ingredientes: um conjunto não vazio A,

279
280 A Apêndice: Funções

chamado de domínio da função, um conjunto não vazio B , chamado


de contradomínio da função e uma correspondência, que associa a
cada elemento do primeiro conjunto um único elemento do segundo

conjunto. O trio domínio, contradomínio e correspondência damos o

nome de função. Para simplicar o seu uso, foi criada uma notação

que empacota todos os três ingredientes. Denotamos uma função por

f : A→B
x → f (x)
para indicar que A é o domínio, B é o contradomínio e que se x é
um elemento de A então a ele associaremos o elemento f (x) de B .
É importante não confundir uma função com sua expressão analítica,

quando esta é dada. Para caracterizar uma função, precisamos dar

seus três ingredientes: domínio, contradomínio e correspondência, e

não somente a correspondência y = f (x).

Exemplo A.1. Seja a função f denida de modo que o seu domínio


é o conjunto dos números naturais e o contradomínio é o conjunto dos
números naturais, e a correspondência é tal que a cada número natural
n associamos o seu quadrado n2 . Observe que podemos denotar isso
compactamente por:
f : N→N
n → n2
Veja também que se dermos simplesmente a expressão analítica
x → x2 ou y = x2 para nossa função, ela não estaria caracterizada,
pois não saberíamos qual é o domínio e o seu contradomínio.
Em alguns casos onde o domínio e o contradomínio estão xados e

claros para o interlocutor, podemos nos referir a uma função simples-

mente invocando sua correspondência y = f (x).


281

Exemplo A.2. Considere o domínio como sendo o conjunto P for-


mado pelas pessoas do Brasil e o segundo conjunto como sendo o con-
junto L das letras do alfabeto. A correspondência será a seguinte: a
cada pessoa do Brasil, associaremos a primeira letra do seu nome.
Assim, uma pessoa chamada Mário, será associada à letra M. Em
notação de função:
f : P →L
x → f (x)
onde f (x) é a primeira letra do nome de x.

Exemplo A.3. Considere o domínio S como sendo o conjunto dos


pontos de uma sala de aula e o contradomínio como sendo os números
reais. A cada ponto x da sala de aula associamos sua temperatura t(x)
em um dado momento, medida por um termômetro instalado na sala.
Observe que t : S → R assim denida é uma função, pois cada ponto
possui uma única temperatura bem denida no instante xado, que é
um número real. Por outro lado, se trocarmos os papéis do domínio
e contradomínio e a cada número real associamos o ponto da sala que
tem aquela temperatura, não teremos uma função, pois pode haver
mais de um ponto com a dada temperatura ou ainda uma temperatura
que não é atingida por nenhum ponto da sala.

Exemplo A.4. Vamos agora dar outro exemplo em que não temos
uma função, isto é, cuja a nossa aparente correspondência não é de
fato uma correspondência, pois não associa a cada elemento x do do-
mínio um único elemento f (x) do contradomínio. Para tanto, xe o
domínio como sendo o conjunto dos números reais no intervalo [0, 1]
e como contradomínio o conjunto Σ denido pelas sequências de ele-
mentos no conjunto {0, 1, 2, . . . , 9}. Ou seja,
282 A Apêndice: Funções


Σ = (a1 , a2 , a3 , . . . ); ai ∈ {0, 1, 2, 3, . . . , 9} .
Cada elemento x ∈ [0, 1] possui uma expansão decimal x = 0, x1 x2 x3 . . . .
Dena f : [0, 1] → Σ colocando f (x) = (x1 , x2 , x3 , . . . ).
A princípio, parece que f denida desse modo é uma função. Po-
rém, olhando de perto vemos que o número 0, 1 possui mais de uma
representação na base decimal, pois 0, 1 = 0, 09999 . . . . Portanto, f
não está bem denida, isto é, f não associa a cada elemento de [0, 1]
um único elemento de Σ.
Denição A.5. Dada uma função f denida por
f : A→B
x → f (x)
o conjunto imagem de f é o subconjunto f (A) do contradomínio B
formado pelos pontos y do contradomínio tais que existe algum ponto
x no domínio A tal que y = f (x). Ou seja
f (A) = {y ∈ B; existe x ∈ A tal que y = f (x)}.
imagem de um ponto x ∈ A é o ponto f (x). Denimos também
A

a restrição de f a um subconjunto A de seu domínio é a nova função


0

denida considerando-se o domínio como sendo o conjunto A0 e os

demais elementos os mesmos. Denotamos essa nova função por f |A0


ou ainda
f |A0 : A0 → B
x → f (x)
No Exemplo A.2 poderíamos trocar o domínio por um de seus

subconjuntos não vazios. Por exemplo, poderíamos considerar o sub-

conjunto A de P formado pelas pessoas do Brasil que nasceram em

Alagoas.
283

Denição via Relações


Um modo mais formal de denir função é usar a noção de relação

entre dois conjuntos A e B. Uma relação entreA e B é simplesmente


um subconjunto R do produto cartesiano A × B . Uma função é uma
relação R entre A e B que satisfaz duas condições:

• R é unívoca: dados x1 , x2 ∈ A e y∈B tais que (x1 , y) ∈ R e

(x2 , y) ∈ R então x1 = x2 ;

• R é total: dado x ∈ A existe y ∈ B tal que (x, y) ∈ R. de modo


que dado x ∈ A, existe um único y ∈ B tal que (x, y) ∈ R.

Funções Injetoras, Sobrejetoras e Bijetoras


Denição A.6. Uma função f : A → B é dita injetora (ou injetiva )
se a seguinte propriedade vale:

Dados x, y ∈ A tais que f (x) = f (y), então x = y.

Outro modo equivalente de formular tal propriedade é usando sua


forma contrarrecíproca:

Se x, y ∈ A são tais que x 6= y, então f (x) 6= f (y).

Exemplo A.7. Por exemplo, a função f : R → R dada por f (x) = x2


não é injetora, pois f (−1) = (−1)2 = 12 = f (1).
Por outro lado, se g : [0, +∞) → R é dada por g(x) = x2 , então g
é injetora, pois dados dois números não negativos a e b tais que g(a) =
g(b), isto é, a2 = b2 , então a2 − b2 = 0, de onde (a − b)(a + b) = 0,
restando as possibilidades a = b ou a = −b. Como a e b são positivos,
temos que a = b.
284 A Apêndice: Funções

Denição A.8. Uma função f : A → B é dita sobrejetora (ou so-

brejetiva ) se a seguinte propriedade vale:

Dado y ∈ B existe x ∈ A, tal que f (x) = y.

Exemplo A.9. Por exemplo, a função f : R → R dada por f (x) =


x2 do exemplo anterior não é sobrejetora, pois não existe nenhum
número real x tal que f (x) = −1, por exemplo. Por outro lado, se
considerarmos g(x) : R → [0, +∞) dada por g(x) = x2 , então g é
sobrejetora, pois dado qualquer número não negativo b, podemos tomar

a como sendo a = b de modo que g(a) = a2 = b.

Denição A.10. Uma função é dita bijetora (ou ainda bijetiva ) se


ela é injetora e sobrejetora.

Por exemplo, a funçãof : R → R dada por f (x) = x3 é uma função


bijetora, pois é injetora e sobrejetora, já que dado y ∈ R, existe um
3
único x ∈ R tal que y = x .

Quando f : A → B é bijetora, então dado qualquer elemento y ∈

B , existe um elemento x ∈ A tal que f (x) = y (pois f é sobrejetora) e


esse elemento é único (pois f é injetora). Em outros termos, podemos

denir uma nova função: g : B → A associando a cada elemento y ∈ B

o único elemento x em A tal que f (x) = y . Em outras palavras,

g(y) = x, se e somente se, f (x) = y.

g é chamada de função inversa de f .


Quando existe uma bijeção f entre dois conjuntos A e B, dizemos

que A e B têm a mesma quantidade de elementos ou cardinalidade.


Para mais informações sobre funções, recomendamos a leitura de [3].
Referências Bibliográcas
[1] AIGNER, M. e ZIEGLER, G. (2002). As Provas estão
no Livro. Edgard Blücher.

[2] GARCIA, A. e LEQUAIN, I. (2003). Elementos de Ál-


gebra. Projeto Euclides, IMPA.

[3] LIMA, E. L.; CARVALHO, P. C. P.; WAGNER, E. e

MORGADO, A.C. (2004). A Matemática do Ensino Mé-


dio. Volume 1. Sociedade Brasileira de Matemática.

[4] LIMA, E.L.; CARVALHO, P. C. P.; WAGNER, E. e

MORGADO, A.C. (2004). A Matemática do Ensino Mé-


dio. Volume 2. Sociedade Brasileira de Matemática.

[5] LIMA,E.L.; CARVALHO,P. C. P.; WAGNER,E. e

MORGADO,A.C. (2004). A Matemática do Ensino Mé-


dio. Volume 3. Sociedade Brasileira de Matemática.

[6] LIMA, E.L.; CARVALHO, P. C. P.; WAGNER,E. e

MORGADO, A.C. (2001). Temas e Problemas. Socie-

dade Brasileira de Matemática.

[7] LIMA, E.L. (2001). Álgebra Linear. Sociedade Brasileira


de Matemática.

285
286 REFERÊNCIAS BIBLIOGRÁFICAS

[8] MORAIS FILHO, D. C. (2007). Um Convite à Matemá-


tica. EDUFCG.

[9] MORGADO, A.; CARVALHO, J.; CARVALHO, P.;

FERNANDEZ, P. (1991). Análise Combinatória e Pro-


babilidade . Sociedade Brasileira de Matemática.

[10] RIBENBOIM, P. (2001). Números Primos: Mistérios e


Recordes. Sociedade Brasileira de Matemática.

[11] SANTOS, J. P. O. (1993) Introdução à Teoria dos Nú-


meros. IMPA.

[12] SANTOS, J. P. O.; MELLO, M. P. e MURARI, I. T.

C. (2006). Introdução à Análise Combinatória. Editora

Unicamp.

[13] SOARES, M. G. (2005). Cálculo em uma Variável Com-


plexa. Sociedade Brasileira de Matemática.

S-ar putea să vă placă și